You are on page 1of 176

Scilab Code for Unit Operations of Chemical Engineering by Warren L. McCabe, Julian C.

Smith, Peter Harriott1


Created by Prashant Dave Sr. Research Fellow Chem. Engg. Indian Institute of Technology, Bombay College Teacher and Reviewer ............. .............. IIT Bombay 30 October 2010

by a grant from the National Mission on Education through ICT, http://spoken-tutorial.org/NMEICT-Intro. This text book companion and Scilab codes written in it can be downloaded from the Textbook Companion Project Section at the website http://scilab.in/

1 Funded

Book Details
Author: Warren L. McCabe, Julian C. Smith, Peter Harriott Title: Unit Operations of Chemical Engineering Publisher: McGraw-Hill, Inc. Edition: Fifth Year: 1993 Place: New Delhi ISBN: 0-07-112738-0

Contents
List of Scilab Code 1 Denitions and Principles 1.1 Scilab Code . . . . . . . . . . . . . . . . . . . . . . . . . . . 2 Fluid Statics and its Application 2.1 Scilab Code . . . . . . . . . . . . . . . . . . . . . . . . . . . 4 Basic Equations of Fluid Flow 4.1 Scilab Code . . . . . . . . . . . . . . . . . . . . . . . . . . . 5 Flow of Incompressible Fluids in Conduits and Thin Layers 5.1 Scilab Code . . . . . . . . . . . . . . . . . . . . . . . . . . . 6 Flow of Compressible Fluids 6.1 Scilab Code . . . . . . . . . . . . . . . . . . . . . . . . . . . 7 Flow Past Immersed Bodies 7.1 Scilab Code . . . . . . . . . . . . . . . . . . . . . . . . . . . 8 Transportation and Metering of Fluids 8.1 Scilab Code . . . . . . . . . . . . . . . . . . . . . . . . . . . 9 Agitation and Mixing of Liquids 9.1 Scilab Code . . . . . . . . . . . . . . . . . . . . . . . . . . . 10 Heat Transfer by Conduction 10.1 Scilab Code . . . . . . . . . . . . . . . . . . . . . . . . . . . 5 9 9 11 11 13 13 18 18 20 20 25 25 29 29 36 36 44 44

11 Principles of Heat Flow in Fluids 11.1 Scilab Code . . . . . . . . . . . . . . . . . . . . . . . . . . . 12 Heat Transfer to Fluids without Phase Change 12.1 Scilab Code . . . . . . . . . . . . . . . . . . . . . . . . . . . 13 Heat Transfer to Fluids with Phase Change 13.1 Scilab Code . . . . . . . . . . . . . . . . . . . . . . . . . . . 14 Radiation Heat Transfer 14.1 Scilab Code . . . . . . . . . . . . . . . . . . . . . . . . . . . 15 Heat-Exchange Equipment 15.1 Scilab Code . . . . . . . . . . . . . . . . . . . . . . . . . . . 16 Evaporation 16.1 Scilab Code . . . . . . . . . . . . . . . . . . . . . . . . . . . 17 Equilibrium-Stage Operations 17.1 Scilab Code . . . . . . . . . . . . . . . . . . . . . . . . . . . 18 Distillation 18.1 Scilab Code . . . . . . . . . . . . . . . . . . . . . . . . . . . 19 Introduction to Multicomponent Distillation 19.1 Scilab Code . . . . . . . . . . . . . . . . . . . . . . . . . . . 20 Leaching and Extraction 20.1 Scilab Code . . . . . . . . . . . . . . . . . . . . . . . . . . .

48 48 50 50 56 56 60 60 62 62 67 67 71 71 75 75 86 86 93 93

21 Principles of Diusion and Mass Transer between Phases 102 21.1 Scilab Code . . . . . . . . . . . . . . . . . . . . . . . . . . . 102 22 Gas Absorption 108 22.1 Scilab Code . . . . . . . . . . . . . . . . . . . . . . . . . . . 108 23 Humidication Operations 122 23.1 Scilab Code . . . . . . . . . . . . . . . . . . . . . . . . . . . 122

24 Drying of Solids 126 24.1 Scilab Code . . . . . . . . . . . . . . . . . . . . . . . . . . . 126 25 Adsorption 132 25.1 Scilab Code . . . . . . . . . . . . . . . . . . . . . . . . . . . 132 26 Membrane Separation Processes 141 26.1 Scilab Code . . . . . . . . . . . . . . . . . . . . . . . . . . . 141 27 Crystallization 146 27.1 Scilab Code . . . . . . . . . . . . . . . . . . . . . . . . . . . 146 28 Properties, Handling and Mixing of Particulate Soilds 155 28.1 Scilab Code . . . . . . . . . . . . . . . . . . . . . . . . . . . 155 29 Size Reduction 158 29.1 Scilab Code . . . . . . . . . . . . . . . . . . . . . . . . . . . 158 30 Mechanical Separations 162 30.1 Scilab Code . . . . . . . . . . . . . . . . . . . . . . . . . . . 162

List of Scilab Code


1.1 2.1 2.2 4.1 4.2 4.3 4.4 5.1 6.1 6.2 6.3 7.1 7.2 7.3 8.1 8.2 8.3 8.4 8.5 8.6 9.1 9.2 9.3 9.4 9.5 9.6 9.7 9.8 Example Example Example Example Example Example Example Example Example Example Example Example Example Example Example Example Example Example Example Example Example Example Example Example Example Example Example Example 1.1.sce 2.1.sce 2.2.sce 4.1.sce 4.2.sce 4.3.sce 4.4.sce 5.1.sce 6.1.sce 6.2.sce 6.3.sce 7.1.sce 7.2.sce 7.3.sce 8.1.sce 8.2.sce 8.3.sce 8.4.sce 8.5.sce 8.6.sce 9.1.sce 9.2.sce 9.3.sce 9.4.sce 9.5.sce 9.6.sce 9.7.sce 9.8.sce . . . . . . . . . . . . . . . . . . . . . . . . . . . . . . . . . . . . . . . . . . . . . . . . . . . . . . . . . . . . . . . . . . . . . . . . . . . . . . . . . . . . . . . . . . . . . . . . . . . . . . . . . . . . . . . . . . . . . . . . . . . . . . . . . . . . . . . . . . . . . . . . . . . . . . . . . . . . . . . . . . . . . . . . . . . . . . . . . . . . . . . . . . . . . . . . . . . . 5 . . . . . . . . . . . . . . . . . . . . . . . . . . . . . . . . . . . . . . . . . . . . . . . . . . . . . . . . . . . . . . . . . . . . . . . . . . . . . . . . . . . . . . . . . . . . . . . . . . . . . . . . . . . . . . . . . . . . . . . . . . . . . . . . . . . . . . . . . . . . . . . . . . . . . . . . . . . . . . . . . . . . . . . . . . . . . . . . . . . . . . . . . . . . . . . . . . . . . . . . . . . . . . . . . . . . . . . . . . . . . . . . . . . . . . . . . . . . . . . . . . . . . . . . . . . . . . . . . . . . . . . . . . . . . . . . . . . . . . . . . . . . . . . . . . . . . . . . . . . . . . . . . . . . . . . . . . . . . . . . . . . . . . . . . . . . . . . . . . . . . . . . . . . . . . . . . . . . . . . . . . . . . . . . . . . . . . . . . . . . . . . . . . . . . . . . . . . . . . . . . . . . . . . . . . . . . . . . . . . . . . . . . . . . . . . . . . . . . . . . . . . . . . . . . . . . . . . . . . . . . . . . . . . . . . . . . . . . 9 11 11 13 14 15 16 18 20 22 23 25 26 27 29 30 31 32 33 34 36 37 37 38 39 39 41 42

10.1 10.2 10.3 10.4 10.5 11.1 12.1 12.2 12.3 12.4 13.1 13.2 14.1 15.1 15.2 15.3 15.4 16.1 16.2 16.3 17.1 17.2 18.1 18.2 18.3 18.4 18.6 18.7 18.8 19.2 19.3 19.4 19.5 20.1 20.2 20.3 21.1 21.2

Example Example Example Example Example Example Example Example Example Example Example Example Example Example Example Example Example Example Example Example Example Example Example Example Example Example Example Example Example Example Example Example Example Example Example Example Example Example

10.1.sce 10.2.sce 10.3.sce 10.4.sce 10.5.sce 11.1.sce 12.1.sce 12.2.sce 12.3.sce 12.4.sce 13.1.sce 13.2.sce 14.1.sce 15.1.sce 15.2.sce 15.3.sce 15.4.sce 16.1.sce 16.2.sce 16.3.sce 17.1.sce 17.2.sce 18.1.sce 18.2.sce 18.3.sce 18.4.sce 18.6.sce 18.7.sce 18.8.sce 19.2.sce 19.3.sce 19.4.sce 19.5.sce 20.1.sce 20.2.sce 20.3.sce 21.1.sce 21.2.sce

. . . . . . . . . . . . . . . . . . . . . . . . . . . . . . . . . . . . . .

. . . . . . . . . . . . . . . . . . . . . . . . . . . . . . . . . . . . . .

. . . . . . . . . . . . . . . . . . . . . . . . . . . . . . . . . . . . . .

. . . . . . . . . . . . . . . . . . . . . . . . . . . . . . . . . . . . . .

. . . . . . . . . . . . . . . . . . . . . . . . . . . . . . . . . . . . . .

. . . . . . . . . . . . . . . . . . . . . . . . . . . . . . . . . . . . . . 6

. . . . . . . . . . . . . . . . . . . . . . . . . . . . . . . . . . . . . .

. . . . . . . . . . . . . . . . . . . . . . . . . . . . . . . . . . . . . .

. . . . . . . . . . . . . . . . . . . . . . . . . . . . . . . . . . . . . .

. . . . . . . . . . . . . . . . . . . . . . . . . . . . . . . . . . . . . .

. . . . . . . . . . . . . . . . . . . . . . . . . . . . . . . . . . . . . .

. . . . . . . . . . . . . . . . . . . . . . . . . . . . . . . . . . . . . .

. . . . . . . . . . . . . . . . . . . . . . . . . . . . . . . . . . . . . .

. . . . . . . . . . . . . . . . . . . . . . . . . . . . . . . . . . . . . .

. . . . . . . . . . . . . . . . . . . . . . . . . . . . . . . . . . . . . .

. . . . . . . . . . . . . . . . . . . . . . . . . . . . . . . . . . . . . .

. . . . . . . . . . . . . . . . . . . . . . . . . . . . . . . . . . . . . .

. . . . . . . . . . . . . . . . . . . . . . . . . . . . . . . . . . . . . .

. . . . . . . . . . . . . . . . . . . . . . . . . . . . . . . . . . . . . .

. . . . . . . . . . . . . . . . . . . . . . . . . . . . . . . . . . . . . .

. . . . . . . . . . . . . . . . . . . . . . . . . . . . . . . . . . . . . .

. . . . . . . . . . . . . . . . . . . . . . . . . . . . . . . . . . . . . .

. . . . . . . . . . . . . . . . . . . . . . . . . . . . . . . . . . . . . .

44 44 46 46 47 48 50 50 53 54 56 58 60 62 63 64 65 67 69 69 71 73 75 76 79 80 81 84 85 86 88 89 91 93 94 97 102 103

21.3 21.4 21.5 21.6 22.1 22.2 22.3 22.4 22.5 22.6 23.1 23.3 24.1 24.2 24.3 24.4 25.1 25.2 25.3 25.4 26.1 26.4 26.5 27.1 27.2 27.3 27.4 27.5 27.6 28.1 28.2 29.1 29.2 30.1 30.2 30.3 30.4 30.5

Example Example Example Example Example Example Example Example Example Example Example Example Example Example Example Example Example Example Example Example Example Example Example Example Example Example Example Example Example Example Example Example Example Example Example Example Example Example

21.3.sce 21.4.sce 21.5.sce 21.6.sce 22.1.sce 22.2.sce 22.3.sce 22.4.sce 22.5.sce 22.6.sce 23.1.sce 23.3.sce 24.1.sce 24.2.sce 24.3.sce 24.4.sce 25.1.sce 25.2.sce 25.3.sce 25.4.sce 26.1.sce 26.4.sce 26.5.sce 27.1.sce 27.2.sce 27.3.sce 27.4.sce 27.5.sce 27.6.sce 28.1.sce 28.2.sce 29.1.sce 29.2.sce 30.1.sce 30.2.sce 30.3.sce 30.4.sce 30.5.sce

. . . . . . . . . . . . . . . . . . . . . . . . . . . . . . . . . . . . . .

. . . . . . . . . . . . . . . . . . . . . . . . . . . . . . . . . . . . . .

. . . . . . . . . . . . . . . . . . . . . . . . . . . . . . . . . . . . . .

. . . . . . . . . . . . . . . . . . . . . . . . . . . . . . . . . . . . . .

. . . . . . . . . . . . . . . . . . . . . . . . . . . . . . . . . . . . . .

. . . . . . . . . . . . . . . . . . . . . . . . . . . . . . . . . . . . . . 7

. . . . . . . . . . . . . . . . . . . . . . . . . . . . . . . . . . . . . .

. . . . . . . . . . . . . . . . . . . . . . . . . . . . . . . . . . . . . .

. . . . . . . . . . . . . . . . . . . . . . . . . . . . . . . . . . . . . .

. . . . . . . . . . . . . . . . . . . . . . . . . . . . . . . . . . . . . .

. . . . . . . . . . . . . . . . . . . . . . . . . . . . . . . . . . . . . .

. . . . . . . . . . . . . . . . . . . . . . . . . . . . . . . . . . . . . .

. . . . . . . . . . . . . . . . . . . . . . . . . . . . . . . . . . . . . .

. . . . . . . . . . . . . . . . . . . . . . . . . . . . . . . . . . . . . .

. . . . . . . . . . . . . . . . . . . . . . . . . . . . . . . . . . . . . .

. . . . . . . . . . . . . . . . . . . . . . . . . . . . . . . . . . . . . .

. . . . . . . . . . . . . . . . . . . . . . . . . . . . . . . . . . . . . .

. . . . . . . . . . . . . . . . . . . . . . . . . . . . . . . . . . . . . .

. . . . . . . . . . . . . . . . . . . . . . . . . . . . . . . . . . . . . .

. . . . . . . . . . . . . . . . . . . . . . . . . . . . . . . . . . . . . .

. . . . . . . . . . . . . . . . . . . . . . . . . . . . . . . . . . . . . .

. . . . . . . . . . . . . . . . . . . . . . . . . . . . . . . . . . . . . .

. . . . . . . . . . . . . . . . . . . . . . . . . . . . . . . . . . . . . .

103 104 105 107 108 109 110 114 115 118 122 124 126 127 128 129 132 133 136 138 141 143 144 146 147 148 148 149 150 155 156 158 158 162 164 169 170 173

List of Figures
17.1 Diagram for Example 17.1 . . . . . . . . . . . . . . . . . . . 18.1 Results of Example 18.1 . . . . . . . . . . . . . . . . . . . . 20.1 Diagram for Example 20.2 . . . . . . . . . . . . . . . . . . . 20.2 Diagram for Example 20.3 . . . . . . . . . . . . . . . . . . . 22.1 Diagram for Example 22.3 . . . . . . . . . . . . . . . . . . . 22.2 Diagram for Example 22.6 . . . . . . . . . . . . . . . . . . . 25.1 Breakthrough curves for Example 25.2 . . . . . . . . . . . . 27.1 Population density vs. length Example 27.6 . . . . . . . . . 27.2 Size-distribution relations for Example 27.6 . . . . . . . . . . 29.1 Mass-fractions of Example 29.2 . . . . . . . . . . . . . . . . 30.1 30.2 30.3 30.4 30.5 Analysis for Example 30.1 . . . . . . . . . . . . . t/V vs. V for Example 30.2 . . . . . . . . . . . . Rm vs. deltaP for Example 30.2 . . . . . . . . . . alpha vs. deltaP for Example 30.2 . . . . . . . . . Eect of pressure drop and concentration on ux ple 30.4 . . . . . . . . . . . . . . . . . . . . . . . . . . . . . . . for . . . . . . . . . . . . . . . . . . Exam. . . . 73 76 97 100 113 121 136 153 154 161 164 167 168 169 173

Chapter 1 Denitions and Principles


1.1 Scilab Code

Example 1.1 Example 1.1.sce


1 clear all ; 2 clc ; 3 4 // Example 1 . 1 5 6 // S o l u t i o n 7 // ( a ) 8 // U s i n g Eq . ( 1 . 6 , ) , ( 1 . 2 6 ) , and ( 1 . 2 7 ) 9 // L e t N = 1N 10 N = 0.3048/(9.80665*0.45359237*0.3048) ; // [ l b f ] 11 12 // ( b ) 13 // U s i n g ( 1 . 3 8 ) , ( 1 . 1 6 ) , ( 1 . 2 6 ) , and ( 1 . 3 1 ) 14 // L e t B = 1 Btu 15 B = 0.45359237*1000/1.8; // [ c a l ] 16 17 // ( c ) 18 // U s i n g Eq . ( 1 . 6 ) , ( 1 . 1 4 ) , ( 1 . 1 5 ) , ( 1 . 2 6 ) , ( 1 . 2 7 ) , 19

and ( 1 . 3 6 ) // L e t P = 1 atm

20 P = 1 .0 13 2 5* 1 0^ 5* 0 .3 04 8 /( 3 2. 17 4 *0 .4 5 35 9 23 7* 1 2^ 2 ) ; //

[ l b f / in . 2 ]
21 22 23

// ( d ) // U s i n g Eq . ( 1 . 8 ) , ( 1 . 3 3 ) , ( 1 . 3 7 ) , ( 1 . 2 6 ) , and (1.27) 24 // L e t hp = 1 hp 25 hp = 550 *32.174* 0.453592 37*0.304 8^2/1000 ; // [kW]

10

Chapter 2 Fluid Statics and its Application


2.1 Scilab Code

Example 2.1 Example 2.1.sce


1 2 3 4 5 6 7 8 9 10 11 12 13 14 15

clear all ; clc ; // Example 2 . 1 rho_A = 13590; rho_B = 1260; Pa = 14000; gc = 1; // [ f t l b / l b f s 2 ] // U s i n g Eq . ( 2 . 5 ) ; Zb = 250 mmHg Pb = -(250/1000) *(9.80665/1) *13590; // U s i n g Eq . ( 2 . 1 0 ) Rm = (14000+33318) /(9.80665*(13590 -1260) ) disp ( mm ,Rm , The r e a d i n g i n t h e mamometer i s (Rm) = ) Example 2.2 Example 2.2.sce 11

1 2 3 4 5 6 7 8 9 10 11 12 13 14 15 16 17 18

clear all ; clc ; // Example 2 . 2 // ( a ) // U s i n g Eq . ( 2 . 1 5 ) t = (100*1.1) /(1153 -865) rate_each_stream = (1500*42) /(24*60) total_liquid_holdup = 2*43.8*23 vol = total_liquid_holdup /0.95 disp ( g a l ,vol , v e s s e l s i z e = ) // ( b ) t a n k d i a m e t e r Zt = 0.90*4 ZA1 = 1.8 // [ f t ] ; ZA2 = 1.8 + (3.6 -1.8) *(54/72) disp ( f t ,ZA2 , t a n k d i a m e t e r = )

12

Chapter 4 Basic Equations of Fluid Flow


4.1 Scilab Code

Example 4.1 Example 4.1.sce


1 clear all ; 2 clc ; 3 4 // Example 4 . 1 5 6 // ( a ) 7 // d e n s i t y o f t h e f l u i d 8 rho = 0.887*62.37; // [ l b / f t 3 ] 9 // t o t a l v o l u m e t r i c f l o w r a t e 10 q = 30*60/7.48; // [ f t 3/ h r ] 11 // mass f l o w r a t e i n p i p e A and p i p e B i s same 12 mdot = rho * q // [ l b / h r ] 13 // mass f l o w r a t e i n e a c h p i p e o f C i s h a l f o f t h e 14 15 16 17 18 19

total flow mdot_C = mdot /2 // [ l b / h r ] disp ( l b / h r , mdot , mass f l o w r a t e p i p e A = ) disp ( l b / h r , mdot , mass f l o w r a t e p i p e B = ) disp ( l b / h r , mdot_C , mass f l o w r a t e p i p e C = ) // ( b )

13

20 // U s i n g Eq . ( 4 . 4 ) , 21 // v e l o c i t y t h r o u g h p i p e A 22 V_Abar = 240.7/(3600*0.0233) // [ f t / s ] 23 24 // v e l o c i t y t h r o u g h p i p e B 25 V_Bbar = 240.7/(3600*0.0513) // [ f t / s ] 26 27 // v e l o c i t y t h r o u g h e a c h p i p e o f C 28 V_Cbar = 240.7/(2*3600*0.01414) // [ f t / s ] 29 30 disp ( f t / s , V_Abar , v e l o c i t y t h r o u g h p i p e 31 disp ( f t / s , V_Bbar , v e l o c i t y t h r o u g h p i p e 32 disp ( f t / s , V_Cbar , v e l o c i t y t h r o u g h p i p e 33 34 // ( c ) 35 // U s i n g Eq . ( 4 . 8 ) , 36 // mass v e l o c i t y t h r o u g h p i p e A 37 GA = mdot /0.0233 // [ kg /m2 s ] 38 39 // mass v e l o c i t y t h r o u g h p i p e B 40 GB = mdot /0.0513 // [ kg /m2 s ] 41 42 // mass v e l o c i t y t h r o u g h e a c h p i p e o f C 43 GC = mdot /(2*0.01414) // [ kg /m2 s ] 44 45 disp ( kg /m2 s ,GA , mass v e l o c i t y t h r o u g h 46

A = ) B = ) C = )

pipe A =

) disp ( kg /m2 s ,GB , mass v e l o c i t y t h r o u g h p i p e B = ) 47 disp ( kg /m2 s ,GC , mass v e l o c i t y t h r o u g h p i p e C = ) Example 4.2 Example 4.2.sce
1 clear all ; 2 clc ; 3 4 // Example 4 . 2

14

5 // A p p l y i n g Eq . ( 4 . 2 5 ) 6 // Pa = Pb , Ua = 0 7 // Zb = 0 , Za = 5m 8 9 // The v e l o c i t y a t s t r e a m l i n e d i s c h a r g e 10 Ub = sqrt (5*2*9.80665) // [m/ s ] 11 disp ( m/ s ,Ub , s t r e a m l i n e d i s c h a r g e v e l o c i t y

( Ub ) =

) Example 4.3 Example 4.3.sce


1 2 3 4 5 6 7 8 9 10 11 12 13 14 15 16 17 18 19 20 21 22 23 24 25 26 27

clear all ; clc ; // Example 4 . 3 rho = 998; // [ kg /m 3 ] Da = 50; // [mm] Db = 20; // [mm] pa = 100; // [ N/m 2 ] // ( a ) Va_bar = 1.0; // [m/ s ] Vb_bar = Va_bar *( Da / Db ) ^2 // [m/ s ] // U s i n g Eq . ( 4 . 2 9 ) // Za = Zb , h f = 0 pb = pa - rho *( Vb_bar ^2 - Va_bar ^2) /(2*1000) // [ kN/m 2 ] disp ( kN/m2 ,pb , pb = ) // ( b ) // Combining Eqs . ( 4 . 1 4 ) & ( 4 . 1 5 ) // For x d i r e c t i o n , // s i n c e Fg = 0 , we g e t Eq . ( 4 . 3 0 ) theta = %pi /4; Va_xbar = Va_bar ; Sa = ( %pi /4) *( Da /1000) ^2; // [m 2 ] Sax = Sa ; // From FIg 4 . 5 Vb_xbar = Vb_bar * cos ( theta ) ; // [m/ s ] 15

28 Sb = %pi /4*( Db /1000) ^2; // [m 2 ] 29 Sbx = Sb * sin ( theta ) ; // [m 2 ] 30 // U s i n g Eq . ( 4 . 6 ) 31 mdot = Va_bar * rho * Sa ; // [ kg / s ] 32 // S u b s t i t u t i n g i n Eq . ( 4 . 3 0 ) 33 // S o l v i n g f o r Fw , x 34 beta_a = 1; beta_b = 1; 35 Fw_x = mdot *( beta_b * Vb_xbar - beta_a * Va_xbar ) - Sax * pa

*1000+ Sbx * pb *1000 // [ N ]


36 37 // For y d i r e c t i o n , 38 // V a y b a r = 0 , Say = 0 39 Vb_ybar = Vb_bar * sin ( theta ) ; // [m/ s ] 40 Sby = Sb * cos ( theta ) ; // [m 2 ] 41 Va_ybar = 0; // [m/ s ] 42 Say = 0; // [m/ s ] 43 44 Fw_y = mdot *( beta_b * Vb_ybar - beta_a * Va_ybar ) - Say * pa

*1000+ Sby * pb *1000 // [ N ] Example 4.4 Example 4.4.sce


1 2 3 4 5 6 7 8 9 10 11 12 13

clear all ; clc ;

// Example 4 . 4 gc = 32.17; // [ f t l b / l b f s 2 ] rho_w = 62.37; // [ l b / f t 3 ] , d e n s i t y o f w a t e r sp_gravity = 1.84; neta = 0.60; hf = 10; // [ f t l b f / l b ] , f r i c t i o n l o s s e s Va_bar = 3; // [ f t / s ] Da = 3; // [ i n . ] Db = 2; // [ i n . ] // From Appendix c o r s s s e c i o n a l a r e a r e s p e c t i v e t o 3 i n . and 2 i n . d i a m e t e r 14 Sa = 0.0513; // [ f t 2 ] 15 Sb = 0.0233; // [ f t 2 ] 16

16 17 18 19 20 21 22 23 24 25 26 27 28

Za = 0 ; // [ f t ] Zb = 50 ; // [ f t ] Vb_bar = Va_bar *( Sa / Sb ) ; // [ f t / s ] g = gc // U s i n g Eq . ( 4 . 3 2 ) Wp = (( Zb * g / gc ) + Vb_bar ^2/(2* gc ) + hf ) / neta ; // [ f t l b f / lb ] // U s i n g Eq . ( 4 . 3 2 ) on pump i t s e l f // s t a t i o n a i s t h e s u c t i o n c o n n e c t i o n and s t a t i o n b i s the discharge // Za = Zb // Eq . ( 4 . 3 2 ) becomes // t h e p r e s s u r e d e v e l o p e d by pume i s d e l t a P = pbpa deltaP = sp_gravity * rho_w *((( Va_bar ^2 - Vb_bar ^2) /(2* gc ) ) + neta * Wp ) // [ l b f / f t 3 ]

29 30 mdot = Sa * Va_bar * sp_gravity * rho_w ; 31 32 // t h e Power 33 P = mdot * Wp /550 // [ hp ]

17

Chapter 5 Flow of Incompressible Fluids in Conduits and Thin Layers


5.1 Scilab Code

Example 5.1 Example 5.1.sce


1 2 3 4 5 6 7 8 9 10 11 12 13 14 15 16 17 18

clear all ; clc ; // Example 5 . 1 // Given mu = 0.004; // [ kg /m s ] D = 0.0779; // [m] rho = 0.93*998; // [ kg /m 3 ] L = 45; // [m] // For f i t t i n g s , form T a b l e 5 . 1 sum_Kf = 0.9 + 2*0.2; // From Eq . ( 4 . 2 9 ) , a s s u m i n g a l p h a a = 1 , // s i n c e pa = pb , and V a b a r = 0 //A = Vb bar 2 / 2 + h f = g ( ZaZb ) A = 9.80665*(6+9) ; // [m2/ s 2 ] // U s i n g F i g 5 . 9 f = 0.0055;

18

19 20 21 22 23 24

// U s i n g Eq . ( 5 . 6 8 ) , There i s no e x a p n s i o n l o s s and Ke = 0. // From Eq . ( 5 . 6 6 ) , s i n c e Sa i s v e r y l a g e , Kc = 0 . 4 . Hence Vb_bar = sqrt (294.2/(2.7+2311* f ) ) ; // [m/ s ] // From Appendix 5 , c r o s s s e c t i o n a l a r e a o f t h e p i p e S = 0.00477; // [m 2 ] flow_rate = S * Vb_bar *3600 // [m3/ h r ]

19

Chapter 6 Flow of Compressible Fluids


6.1 Scilab Code

Example 6.1 Example 6.1.sce


1 2 3 4 5 6 7 8 9 10 11 12 13 14 15 16 17 18

clear all ; clc ; // Example 6 . 1 // Given gama = 1.4; M = 29; R = 82.0568*10^ -3; // [ atmm3/Kg molK ] Nma = 0.8; gc = 1; // [ f t l b / l b f s 2 ] // At E n t r a n c e p0 = 20; // [ atm ] T0 = 555.6; // [ K ]

// ( a ) // U s i n g Eq . ( 6 . 2 8 ) // P r e s s u r e a t t h r o a t pt = (1/(1+(( gama -1) /2) * Nma ^2) ^(1/(1 -1/ gama ) ) ) * p0 // [ atm ] 19 // From Eq . ( 6 . 1 0 )

20

20 21 22 23 24 25 26 27 28 29 30 31 32 33 34 35 36 37 38 39 40

rho0 = ( p0 * M ) /( R * T0 ) ; // [ kg /m 3 ] // U s i n g Eq . ( 6 . 1 0 ) and Eq . ( 6 . 2 6 ) , t h e v e l o c i t y i n the throat ut = sqrt ((2* gama * gc * R * T0 ) /( M *( gama -1) ) *(1 -( pt / p0 ) ^(1 -1/ gama ) ) ) ; // [m3 am/ kg ] 0 . 5 // I n t e r m s o f [m/ s ] , U s i n g Appendix 2 , 1 atm = 1 . 0 1 3 2 5 1 0 [ N/m 2 ] ut = ut * sqrt (1.01325*10^5) // [m/ s ] // U s i n g Eq . ( 6 . 2 3 ) , d e n s i t y a t t h r o a t rho_t = rho0 *( pt / p0 ) ^(1/ gama ) // [ kg /m 3 ] // The mass v e l o c i t y a t t h e t h r o a t , Gt = ut * rho_t // [ kg /m2 s ] // U s i n g Eq . ( 6 . 2 4 ) , The t e m p e r a t u r e a t t h r o a t Tt = T0 *( pt / p0 ) ^(1 -1/ gama ) // [ K ]

// ( b ) // From Eq . ( 6 . 2 9 ) pstar = ((2/( gama +1) ) ^(1/(1 -1/ gama ) ) ) * p0 // [ atm ] // From Eq . ( 6 . 2 4 ) and ( 6 . 2 9 ) Tstar = T0 *( pstar / p0 ) ^(1 -1/ gama ) // [ K ] // From Eq . ( 6 . 2 3 ) rho_star = rho0 *( pstar / p0 ) ^(1/ gama ) // [ Kg/m 3 ] // From Eq . ( 6 . 3 0 ) G_star = sqrt (2* gama * gc * rho0 * p0 *101.325*10^3/( gama -1) ) *( pstar / p0 ) ^(1/ gama ) * sqrt (1 -( pstar / p0 ) ^(1 -1/ gama ) ) // [ Kgm2/ s ] 41 u_star = G_star / rho_star // [m/ s ]
42 43 44 45 46 47 48 49 50 51

// ( c ) // By c o n t i n u i t y , G i n v e r s e l y p r o p o r t i o n a l t o S , t h e mass v e l o c i t y a t d i s c h a g e i s G_r = G_star /2 // [ Kg/m3 s ] // U s i n g Eq . ( 6 . 3 0 ) // L e t x = p r / p0 err = 1; eps = 10^ -3; x = rand (1 ,1) ;

21

52 while ( err > eps ) 53 xnew = ((0.1294) / sqrt (1 - x ^(1 -1/1.4) ) ) ^1.4; 54 err = x - xnew ; 55 x = xnew ; 56 end 57 58 // U s i n g Eq . ( 6 . 2 7 ) 59 // The Mach Number a t d i s c h a g e i s 60 Nmr = sqrt ((2/( gama -1) ) *(1/ x ^(1 -1/ gama ) -1) )

Example 6.2 Example 6.2.sce


1 2 3 4 5 6 7 8 9 10 11 12 13 14 15 16 17 18 19 20 21 22 23 24

clear all ; clc ; // Example 6 . 2 // Given Tr = 1000; // [ R ] pr = 20; // [ atm ] Ma_a = 0.05; gama = 1.4; gc = 32.174; // [ f t l b / l b f s 2 ] M = 29; R = 1545; // ( a ) // U s i n g Eq . ( 6 . 4 5 ) A = 2*(1+(( gama -1) /2) * Ma_a ^2) /(( gama +1) * Ma_a ^2) ; fLmax_rh = (1/ Ma_a ^2 -1 -( gama +1) * log ( A ) /2) / gama // ( b ) // U s i n g Eq . ( 6 . 2 8 ) , t h e p r e s s u r e a t t h e end o f t h e i s e n t r o p i c n o z z l e pa A = (1+( gama -1) *( Ma_a ^2) /2) ; pa = pr /( A ^( gama /( gama -1) ) ) // [ atm ] // From Example 6 . 1 , t h e d e n s i t y o f a i r a t 20 atm and 1 0 0 0R i s 0 . 7 9 5 l b / f t 3 // U s i n g Eq . ( 6 . 1 7 ) , t h e a c o u s t i c v e l o c i t y Aa = sqrt ( gc * gama * Tr * R / M ) // [m/ s ] 22

25 // The v e l o c i t y a t t h e e n t r a n c e o f t h e p i p e 26 ua = Ma_a * Aa // [m/ s ] 27 //When L b = L max , t h e g a s l e a v e s t h e p i p e a t t h e

a s t e r i s k c o n d i t i o n s , where
28 Ma_b = 1; 29 // U s i n g Eq . ( 6 . 4 3 ) 30 A = ( gama -1) /2; 31 Tstar = Tr *(1+ A * Ma_a ^2) /(1+ A * Ma_b ^2) // [ K ] 32 // U s i n g Eq . ( 6 . 4 4 ) 33 rho_star = 0.795* Ma_a / sqrt (2*(1+( gama -1) * Ma_a ^2/2) 34 35 36 37 38 39 40 41 42 43 44 45 46 47 48 49 50 51 52 53 54 55 56

/(2.4) ) // [ l b / f t 3 ] // U s i n g Eq . ( 6 . 3 9 ) pstar = p0 * Ma_a / sqrt (1.2) // [ atm ] // Mass v e l o c i t y t h r o u g h t h e e n t i r e p i p e G = 0.795* ua // [ l b / f t 2 s ] ustar = G / rho_star // [ f t / s ] // ( c ) // U s i n g Eq . ( 6 . 4 5 ) w i t h f L m a x r h = 400 err = 1; eps = 10^ -3; Ma_ac = rand (1 ,1) ; i =1; while (( err > eps ) ) A = 2*(1+(( gama -1) /2) * Ma_ac ^2) /(( gama +1) * Ma_ac ^2) ; B = gama *400+1+( gama +1) * log ( A ) /2; Ma_anew = sqrt (1/ B ) ; err = Ma_ac - Ma_anew ; Ma_ac = Ma_anew ; end Ma_ac ; uac = Ma_ac * ua / Ma_a // [ f t / s ] Gc = uac *0.795 // [ l b / f t 2 s ] Example 6.3 Example 6.3.sce

clear all ; 23

2 3 4 5 6 7 8 9 10 11 12 13 14 15 16 17 18 19 20 21 22 23 24 25 26 27 28 29 30 31 32 33 34 35 36

clc ; // Example 6 . 3 // Given pa = 2.7; // [ atm ] T = 288; // [ K ] D = 0.075; // [m] L = 70; // [m] Vbar = 60; // [m/ s ] M = 29; rh = D /4; // [m] mu = 1.74*10^ -5 // [ kg /m s ] Appendix 8 rho_a = (29/22.4) *(2.7/1) *(273/288) // [ kg /m 3 ] R = 82.056*10^ -3; G = Vbar * rho_a // [ kg /m2 s ] Nre = D * G / mu ; kbyd = 0.00015*(0.3048/0.075) ; f = 0.0044; // [ from F i g . 5 . 9 ] // U s i n g Eq . ( 6 . 5 2 ) // p b a r = 1 . 9 8 2 ; / / [ atm ] // pb = 1 . 2 6 4 ; / / [ atm ] err = 1; eps = 10^ -3; pb = 1.5; while ( err > eps ) pbar = ( pa + pb ) /2; A = (( f * L /(2* rh ) ) + log ( pa / pb ) ) ; pb_new = pa -( R * T * G ^2/( pbar *29*101325) ) * A ; err = pb - pb_new ; pb = pb_new ; end pb ; // [ atm ] pbar = ( pa + pb ) /2 // [ atm ]

24

Chapter 7 Flow Past Immersed Bodies


7.1 Scilab Code

Example 7.1 Example 7.1.sce


1 2 3 4 5 6 7 8 9 10 11 12 13 14 15 16 17 18 19 20

clear all ; clc ; // Example 7 . 1 // Given rho_p = 2800; // [ kg /m 3 ] g = 9.80665; // [m/ s 2 ] ac = 50* g ; // [m/ s 2 ] // ( a ) // From a p p e n d i x 20 Dp_100 = 0.147; // [mm] Dp_80 = 0.175; // [mm] Dp = ( Dp_100 + Dp_80 ) /2; // [mm] // From Appendix 14 mu = 0.801; // [ cP ] rho = 995.7; // [ kg /m 2 ] // U s i n g Eq . ( 7 . 4 5 ) K = Dp *10^ -3*( g * rho *( rho_p - rho ) /( mu *10^ -3) ^2) ^(1/3) ; // T h i s i s s l i g h t l y a b o v e t h e S t o k e s law r a n g e

25

21 // Assuming 22 N_rep = 4.4; 23 // From F i g . 7 . 6 24 Cd = 7.9; 25 // From Eq . ( 7 . 3 7 ) 26 mu_ta = sqrt (4* g *( rho_p - rho ) * Dp *10^ -3/(3* Cd * rho ) ) //

[m/ s ]
27 28 // ( b ) 29 // U s i n g ac i n p l a c e o f g i n Eq . ( 7 . 4 5 ) 30 K = K *50^(1/3) ; // S i n c e o n l y a c c e l e r a t i o n c h a n g e s 31 // E t i m a t i n g 32 N_rep = 80; // From F i g . ( 7 . 6 ) 33 Cd = 1.2; 34 mu_tb1 = sqrt (4* ac *( rho_p - rho ) * Dp *10^ -3/(3* Cd * rho ) ) 35 36 37 38

// [m/ s ] // For i r r e g u l a r p a r t i c l e s Cd i s a b o u t 20 p e r c e n t greater // t h a n t h a t f o r s p h e r e s Cd = 1.2*1.2; mu_tb2 = sqrt (4* ac *( rho_p - rho ) * Dp *10^ -3/(3* Cd * rho ) ) // [m/ s ] Example 7.2 Example 7.2.sce

1 2 3 4 5 6 7 8 9 10 11 12 13

clear all ; clc ; // Example 7 . 2 // Given g = 32.174; // [ f t l b / l b f s 2 ] eps = 0.8; speg_s = 4.0; speg_c = 1.594; Ds = 0.004; // [ i n . ] rho_w = 62.37; // [ l b f / f t 3 ] delta_speg = speg_s - speg_c ; delta_rho = rho_w * delta_speg ; // [ l b f / f t 3 ] 26

14 rho_c = rho_w * speg_c ; // [ l b f / f t 3 ] 15 // From Appendix 9 16 mu = 1.03; // [ cP ] 17 // U s i n g Eq . ( 7 . 4 5 ) 18 K = Ds /12*( g * rho_c *( delta_rho ) /( mu *6.72*10^ -4) ^2)

^(1/3) ; 19 // U s i n g Eq . ( 7 . 4 0 ) 20 ut = g *( Ds /12) ^2* delta_rho /(18* mu *6.72*10^ -4) // [ f t / s]


21 22 // The t e r m i n a l v e l o c i t y i n h i n d e r e d s e t t l i n g 23 // C a l c u l a t i n g R e y n o l d s Number 24 Nre = ut * rho_c * Ds /(12* mu *6.72*10^ -4) ; 25 // From F i g . ( 7 . 7 ) 26 n = 4.1; 27 // U s i n g Eq . ( 7 . 4 6 ) 28 us = ut * eps ^ n // [ f t / s ]

Example 7.3 Example 7.3.sce


1 clear all ; 2 clc ; 3 4 // Example 7 . 3 5 // The q u a n t i t i e s n e e d e d a r e 6 mu = 0.01; // [ P ] 7 delta_rho = 0.24; // [ g /cm 3 ] 8 // U s i n g Eq . ( 7 . 5 1 ) , s o l v i n g t h e q u a d r a t i c e q u a t i o n 9 10 11 12 13 14 15 16 17

for

Vom bar a = 1.75*1/(0.11*0.4^3) ; b = 150*0.01*0.6/(0.11^2*0.4^3) ; c = - 980*0.24; Vom_bar = ( - b + sqrt ( b ^2 -4* a * c ) ) /(2* a ) ; // [ cm/ s ] // C o r r e s p o n d i n g R e y n o l d s number Nre = 0.11*0.194*0.124/0.01; // From F i g 7 . 1 3 m = 3.9; // For 25 p e r c e n t e x a p n s i o n 27

18 LbyLm = 1.25; 19 eps = 0.52; 20 // From Eq . ( 7 . 5 9 ) 21 Vo_bar = 1.94*(0.52/0.40) ^3.9 // [mm/ s ]

28

Chapter 8 Transportation and Metering of Fluids


8.1 Scilab Code

Example 8.1 Example 8.1.sce


1 2 3 4 5 6 7 8 9 10 11 12 13 14 15 16 17 18

clear all ; clc ; // Example 8 . 1 // Given vdot = 40; // [ g a l / min ] pb = 50; // [ l b f / i n . 2 ] Za = 4; // [ f t ] Zb = 10; // [ f t ] hfs = 0.5; // [ l b f / i n . 2 ] hfd = 5.5; // [ l b f / i n . 2 ] neta = 0.6; rho = 54; // [ l b / f t 3 ] pv = 3.8; // [ l b f / i n . 2 ] g = 9.8; // [m/ s 2 ] gc = 32.17 // [ f t l b / l b f s 2 ] hf = hfs + hfd ; // [ l b f / i n . 2 ] // ( a )

29

Wp_neta = ((14.7+ pb ) *144/ rho ) +( g / gc *10) +( Vb_bar ^2/(2* gc ) ) +( hf *144/54) -(14.7*144/54) ; // [ f t l b f / lb ] 24 delta_H = Wp_neta ;
25 26 // ( b ) 27 mdot = vdot * rho /(7.48*60) ; // [ l b / s ] 28 // U s i n g Eq . ( 8 . 7 ) , t h e i n p u t power i s 29 Pb = mdot * delta_H /(550* neta ) // [ hp ] 30 31 // ( c ) 32 padash = 14.7*144/ rho ; 33 // The v a p o r p r e s s u r e c o r r e s p o n d i n g t o a head 34 hv = pv *144/ rho ; // [ f t l b f / l b ] 35 // f r i c t i o n i n t h e s u c t i o n l i n e 36 hfs = 0.5*144/ rho ; // [ f t l b f / l b ] 37 // U s i n g Eq . ( 8 . 7 ) , v a l u e o f a v a i l a b l e 38 NPSH = padash - hv - hfs - Za // [ f t ]

19 20 21 22 23

// U s i n g d a t a from Appendix 5 Vb_bar = vdot /6.34; // [ f t / s ] // U s i n g Eq . ( 4 . 3 2 )

Example 8.2 Example 8.2.sce


1 2 3 4 5 6 7 8 9 10 11 12 13

clear all ; clc ; // Example 8 2 // Given pa = 29; // [ i n . Hg ] pb = 30.1; // [ i n . Hg ] va = 0; // [ f t / s ] vb = 150; // [ f t / s ] Ta = 200; // [ F ] vdot = 10000; // [ f t 3/ min ] neta = 0.65; M = 31.3; 30

14 R = 29.92; 15 gc = 32.17; // [ f t l b / l b f s 2 ] 16 // a c t u a l s u c t i o n d e n s i t y 17 rho_a = M * pa *(460+60) /(378.7*30*(460+ Ta ) ) ; // [ l b / f t 18 19 20 21 22 23 24 25 26 27 28 29 30 31

3] // a c u a l d i s c h a r g e d e n s i t y rho_b = rho_a * pb / pa ; // [ l b / f t 3 ] // a v e r a g e d e n s i t y o f t h e f l o w i n g g a s rho = ( rho_a + rho_b ) /2; // [ l b / f t 3 ] // mass f l o w r a t e mdot = vdot * M /(378.7*60) // [ l b / s ] // d e v e l o p e d p r e s s u r e dev_p = ( pb - pa ) *144*14.7/( R * rho ) ; // [ f t l b f / l b ] // v e l o c i t y head vel_head = vb ^2/(2* gc ) ; // [ f t l b f / l b ] // U s i n g Eq . ( 8 . 1 ) , a l p h a a = a l p h a b = 1 , va = 0 , Za = Zb , Wp = ( dev_p + vel_head ) / neta // [ f t l b f / l b ] // U s i n g Eq . ( 8 . 4 ) Pb = mdot * Wp /550 // [ hp ] Example 8.3 Example 8.3.sce

1 2 3 4 5 6 7 8 9 10 11 12 13 14 15

clear all ; clc ; // Example 8 . 3 // Given vdot = 180; // [ f t 3/ min ] pa = 14; // [ l b f / i n . 2 ] pb = 900; // [ l b f / i n . 2 ] Ta = 80+460; // [ K ] q0 = 0.063; // [m3/ s ] Cp = 9.3; // [ Btu / lbmol F ] gama = 1.31; delta_Tw = 20; // [ F ] // ( a ) neta = 0.80; 31

16

17 18 19 20 21 22 23 24 25 26 27

// For a m u l t i s t a g e c o m p r e s s o r t h e t o t a l power i s a minimum i f e a c h s t a g e doed t h e same amount o f work // Hence u s i n g same c o p r e s s i o n r a t i o n f o r e a c h s t a g e // U s i n g Eq . ( 8 . 2 5 ) // For one s t a g e comp_ratio = (900/14) ^(1/3) ; // U s i n g Eq . ( 8 . 2 9 ) , t h e power r e q u i r e d by e a c h s t a g e Pb = ( Ta * q0 * gama * vdot ) *( comp_ratio ^(1 -1/ gama ) -1) /(520*( gama -1) * neta ) ; // [ hp ] // T o t a l Power Pt = 3* Pb // [ hp ]

// ( b ) // U s i n g Eq . ( 8 . 2 2 ) , t h e t e m p e r a t u r e a t t h e e x i t o f each s t a g e 28 Tb = Ta * comp_ratio ^(1 -1/ gama ) // [ R ]


29 30 31 32 33 34 35 36 37

// ( c ) S i n c e 1 l b mol = 3 7 8 . 7 s t d f t 3 , t h e f l o w r a t e is vdot = vdot *60/378.7; // [ l b mol / h ] // Heat l o a d i n e a c h c o o l e r i s Hl = vdot * Cp *( Tb - Ta ) // [ Btu / h ] // T o t a l h e a t l o s s Htotal = 3* Hl ; // [ Btu / h ] // C o o l i n g w a t e r r e q u i r e m e n t cwr = Htotal / delta_Tw // [ l b /h ] Example 8.4 Example 8.4.sce

1 2 3 4 5 6 7 8

clear all ; clc ; // Example 8 . 4 // Given q = 75/3600 ; // [m3/ s ] rho = 62.37*16.018; // [ kg /m 3 ] From Appendix 4 Cv = 0.98; 32

9 10 11 12 13 14 15 16 17 18 19 20 21 22 23

g = 9.80665; // [m/ s 2 ] Sw = 1; Sm = 13.6; h = 1.25; // [m] // ( a ) // U s i n g Eq . ( 2 . 1 0 ) delta_p = g * h *( Sm - Sw ) * rho ; // [ N/m 2 ] // U s i n g Eq . ( 8 . 3 6 ) , n e g l e c t i n g t h e e f f e c t o f b e t a Sb = q /( Cv * sqrt (2* delta_p / rho ) ) ; Db = sqrt (4* Sb / %pi ) *100 // [mm] // ( b ) press_loss = 0.1* delta_p ; // [ N/m 2 ] // Power r e q u i r e d a t f u l l f l o w P = q * press_loss /1000 // [kW] Example 8.5 Example 8.5.sce

1 2 3 4 5 6 7 8 9 10 11 12 13 14 15 16 17 18 19 20

clear all ; clc ; // Example 8 . 5 // Given T = 100; // [ F ] mu_O = 5.45; // [ cP ] spg_O = 0.8927; spg_m = 13.6; spg_gl = 1.11; q = 12000; // [ b b l /d ] rho_ratio = 0.984; rho_w = 62.37; // [ l b / f t 3 ] h = 30; // [ i n . ] gc = 32.174; // [ f t l b / l b f s 2 ] // ( a ) // U s i n g Eq . ( 8 . 4 2 ) rhoB_60 = spg_O * rho_w ; // [ l b / f t 3 ] rho_100 = spg_O * rho_w * rho_ratio ; // [ l b / f t 3 ] mdot = q *42* rhoB_60 /(24*3600*7.48) ; // [ l b / s ] 33

21 Da = 4.026/12; // [ f t ] 22 delta_p = h /12*( spg_m - spg_gl ) * rho_w *(1) ; // [ l b f / f t 23 24 25 26 27 28 29 30

2] // U s i n g Eq . ( 8 . 4 2 ) beeta = sqrt (4* mdot /(0.61* %pi * Da ^2* sqrt (2* gc * delta_p * rho_100 ) ) ) ; Do = Da * beeta ; // [ f t ] // t h e o r i f i c e d i a m e t e r D = 12* Do // [ i n . ]

// ( b ) // U s i n g F i g . 8 . 2 0 , t h e f r a c t i o n o f d i f f e r e n t i a l pressure loss is 31 fra_prss_loss = 0.68; 32 //Maximum power c o n s u m p t i o n 33 P = mdot * delta_p * fra_prss_loss /( rho_ratio * rho_w * spg_O *550) // [ hp ] Example 8.6 Example 8.6.sce
1 2 3 4 5 6 7 8 9 10 11 12 13 14 15 16 17

clear all ; clc ; // Example 8 . 4 // Given Cpt = 0.98; Ta = 200; // [ F ] Da = 36; // [ i n . ] pa = 15.25; // [ i n . ] h = 0.54; // [ i n . ] P = 29.92; // [ i n . ] spg_m =13.6; // [ s p e c i f i c g r a v i t y o f m e r c u r y ] rho_w = 62.37; // [ l b / f t 3 ] gc = 32.174; // [ f t l b / l b f s 2 ] // U s i n g Eq . ( 8 . 5 2 ) Pabs = P + pa / spg_m ; // [ i n . ] rho = 29*492*31.04/(359*(200+460) *29.92) ; // [ l b / f t 3] 34

18 19 20 21 22 23 24 25 26 27 28 29 30

// From manometer r e a d i n g delta_p = h /12* rho_w // [ l b f / f t 3 ] // U s i n g Eq . ( 8 . 5 3 , m aximum v e l o c i t y , a s s u m i n g Nma i s negligible umax = Cpt * sqrt (2* gc * delta_p / rho ) // [ f t / s ] // The r e y n o l d s number b a s e d on maximum v e l o c i t y mu_air = 0.022 ; // [ cP ] form Appendix 8 Nre_max = ( Da /12) * umax * rho /( mu_air *0.000672) ; // U s i n g F i g 5 . 7 , t o o b t a i n a v e r a g e v e l o c i t y Vbar = 0.86* umax // [ f t / s ] Nre = Nre_max *0.86; // The v o l u m e t r i c f l o w r a t e q = Vbar *( Da /12) ^2* %pi /4*520/660* Pabs / P *60 // [ f t 3/ min ]

35

Chapter 9 Agitation and Mixing of Liquids


9.1 Scilab Code

Example 9.1 Example 9.1.sce


1 2 3 4 5 6 7 8 9 10 11 12 13 14 15 16 17 18

clear all ; clc ; // Exapmle 9 . 1 // Given Dt = 6; // [ f t ] h = 2; // [ f t ] n = 90/60; // [ r p s ] mu = 12*6.72*10^ -4; // [ l b / f t s ] g = 32.17; // [ f t / s 2 ] rho = 93.5; // [ l b / f t 3 ] Da = 2; // [ f t ] Nre = Da ^2* n * rho / mu ; // From c u r v e A o f F i g . 9 . 1 2 Np = 5.8 // Form Eq . ( 9 . 2 0 ) P = Np * rho * n ^3* Da ^5/ g // [ f t l b f / s ]

36

19 P = P /550 // [ hp ]

Example 9.2 Example 9.2.sce


1 2 3 4 5 6 7 8 9 10 11 12 13 14 15 16 17 18 19 20 21 22 23 24 25 26 27 28 29

clear all ; clc ; // Example 9 . 2 // Given Dt = 6; // [ f t ] h = 2; // [ f t ] n = 90/60; // [ r p s ] mu = 12*6.72*10^ -4; // [ l b / f t s ] g = 32.17; // [ f t / s 2 ] rho = 93.5; // [ l b / f t 3 ] Da = 2; // [ f t ] Nre = Da ^2* n * rho / mu ; // Froude number Nfr = n ^2* Da / g ; // From T a b l e 9 . 1 a = 1; b = 40.0; // U s i n g Eq . ( 9 . 1 9 ) m = (a - log ( Nre ) /2.303) / b ; // U s i n g F i g . 9 . 1 2 , c u r v e D, Np = 1.07; // C o r r e c t e d v a l u s o f Np Np = Np * Nfr ^ m ; // Form Eq . ( 9 . 2 0 ) P = Np * rho * n ^3* Da ^5/ g // [ f t l b f / s ] P = P /550 // [ hp ] Example 9.3 Example 9.3.sce

1 clear all ; 2 clc ;

37

3 4 5 6 7 8 9 10 11 12 13 14 15 16 17 18 19 20

// Example 9 . 3 // Given Dt = 6; // [ f t ] h = 2; // [ f t ] n = 90/60; // [ r p s ] mu = 1200*6.72*10^ -2; // [ l b / f t s ] g = 32.17; // [ f t / s 2 ] rho = 70 // [ l b / f t 3 ] Da = 2; // [ f t ] Nre = Da ^2* n * rho / mu ; // From T a b l e 9 . 3 KL = 65; // From Eq . ( 9 . 2 1 ) Np = KL / Nre ; P = Np * rho * n ^3* Da ^5/ g P = P /550 // [ hp ] Example 9.4 Example 9.4.sce

// [ f t l b f / s ]

1 2 3 4 5 6 7 8 9 10 11 12 13 14 15 16

clear all ; clc ; // Example 9 . 4 // Given Dt = 6; // [ f t ] Da = 2; // [ f t ] n = 80/60; // [ r p s ] T = 70; // [ F ] rho = 62.3; // [ l b / f t 3 ] , From Appendix 14 mu = 6.6*10^ -4; // [ l b / f t s ] , From Appendix 14 Nre = Da ^2* n * rho / mu ; // From F i g . 9 . 1 5 ntT = 36; tT = ntT /1.333 // [ s ]

38

Example 9.5 Example 9.5.sce


1 2 3 4 5 6 7 8 9 10 11 12 13 14 15 16 17

clear all ; clc ;

// Example 9 . 5 // Given Dt = 6; // [ f t ] H = 8; // [ f t ] T = 70; // [ F ] sp_gr = 3.18; w_fr = 0.25; Da = 2; // [ f t ] h = 1.5; // [ f t ] gc = 32.17; // [ f t l b / l b f s 2 ] // ( a ) // U s i n g d a t a o f Buurman e t a l . i n F i g . ( 9 . 1 9 ) // c h a n g e i n nc delta_nc = (104/200) ^0.2*(2.18/1.59) ^0.45*(33.3/11.1) ^0.13; 18 // c h a n g e i n P 19 dalta_P = delta_nc ^3;
20 21 22 23 24 25 26 27 28 29 30 31 32

// U s i n g F i g . 9 . 1 9 V = %pi /4* Dt ^2* H *7.48 ; // [ g a l ] P = 3.3* V /1000 // [ hp ] // ( b ) // From T a b l e 9 . 3 , f o r a c o u r b l a d e t u r b i n e , KT = 1.27; Np = KT ; // s l u r r y d e n s i t y rho_m = 1/(( w_fr / sp_gr ) +(1 - w_fr ) ) *62; // [ l b / f t 3 ] nc = ( P * gc *550/( Np * rho_m * Da ^5) ) ^(1/3) // [ r / s ] Example 9.6 Example 9.6.sce

39

1 2 3 4 5 6 7 8 9 10 11 12 13 14 15 16 17 18 19 20 21 22 23 24 25 26 27 28 29 30 31 32 33 34 35 36

clear all ; clc ; // Example 9 . 6 // Given Dt = 2; // [m] Da = 0.667; // [m] n = 180/60; // [ r p s ] T = 20; // [ C ] qg = 100; // [m3/ h ] rho = 1000; // [ kg /m 3 ] mu = 10^ -3; // [ kg /m s ] ut = 0.2; // [m/ s ] // ( a ) // The power i n p u t i s c a l c u l a t e d and f o l l o w e d by c o r r e c t i o n of gas e f f e c t Nre = n * Da ^2* rho / mu ; // For a f l a t b l a d e t u r b i n e , from T a b l e 9 . 3 KT = 5.75; // U s i n g Eq . ( 9 . 2 4 ) Po = KT * n ^3* Da ^5* rho /1000; // [kW] At = %pi /4* Dt ^2; // [m 2 ] // S u p e r f i c i a l g a s v e l o c i t y Vs_bar = At * qg /3600/10 // [m/ s ] // From F i g . 9 . 2 0 Pg/Po = 0 . 6 0 Pg = Po *0.6; // [kW] // From F i g . 9 . 7 , d e p t h o f l i q u i d i s e q u a l t o d i a m e t e r o f the tank // Hence , l i q u i d volume V = %pi /4* Dt ^2* Dt ; // [m 3 ] // The i n p u t power p e r u n i t volume PgbyV = Pg / V ; // [kW/m 3 ] // ( b ) sigma = rho_L = PgbyV = // U s i n g

72.75; // [ g / s 2 ] 10^ -3; // [ g /mm] PgbyV *10^3 ; // [ g /mm s 2 ] Eq . ( 9 . 4 6 ) 40

37 // L e t x = s h i ( 0 . 5 ) 38 // s o l v i n g t h e e q u a t i o n a s q u a d r a t i c e q u a t i o n 39 a = 1; 40 b = -( Vs_bar / ut ) ^0.5; 41 c = -0.216*(( PgbyV ) ^0.4) *( rho_L ^0.2) /( sigma ^0.6) *(

Vs_bar / ut ) ^(0.5) ; 42 x = ( - b + sqrt ( b ^2 -4* a * c ) ) /(2* a ) ; 43 shi = x ^2;


44 45 46 47 48 49 50 51 52

// ( c ) //To f i n d o u t mean b u b b l e d i a m e t e r // U s i n g Eq . ( 9 . 4 4 ) Ds_bar = 4.15* sigma ^0.6/( PgbyV ^0.4* rho_L ^0.2) * shi ^0.5+0.9 // [mm] // ( d ) // From Eq . ( 9 . 4 0 ) aprime = 6* shi / Ds_bar // [mm 1] Example 9.7 Example 9.7.sce

1 2 3 4 5 6 7 8 9 10 11 12 13 14 15 16 17

clear all ; clc ; // Exapmle 9 . 7 // Given Dt = 2; // [m] Da = 0.667; // [m] n = 180/60; // [ r p s ] T = 20; // [ C ] qg = 100; // [m3/ h ] rho = 1000; // [ kg /m 3 ] mu = 10^ -3; // [ kg /m s ] ut = 0.2; // [m/ s ] At = %pi /4* Dt ^2; // [m 2 ] // U s i n g v a l u e s form Example 7 . 6 // Assuming Pg/Po d e c r e s a e s t o 0 . 2 5 PgbyV = 0.25*20490/6.28; // [W/m 3 ] 41

18 // U s i n g Eq . ( 9 . 4 7 ) 19 Vs_barc = 0.114*( PgbyV ) *( Dt /1.5) ^0.17/1000 // [m/ s ] 20 qg = Vs_barc * At *3600 // [m3/ h ] 21 // The c a l c u l a t e d f l o o d i n g v e l o c i t y i s beyond t h e

r a n g e o f t h e d a t a on which Eq . ( 9 . 4 7 ) 22 // was based , s o i t may n o t be r e l a i b l e . Based on V s b a r c , t h e h i g h e s t m e a s u r e d v a l u e , qg 23 // would be 850 m3/ h . Example 9.8 Example 9.8.sce
1 2 3 4 5 6 7 8 9 10 11 12 13 14 15 16 17 18 19 20 21 22 23

clear all ; clc ; // Example 9 . 8 // Given D1 = 1; // [ f t ] D6 = 6 Nre_i = 10^4; Da = 4; // [ i n . ] t1 = 15; // [ s ] P = 2; // [ hp / g a l ] // ( a ) // U s i n g F i g . 9 . 1 5 // t h e m i x i n g f a c t o r ntT i s c o n s t a n t and t i m e tT i s asumed c o n s t a n t , // s p e e d n w i l l be t h e same i n b o t h v e s s e l s . // U s i n g Eq . ( 9 . 2 4 ) w i t h c o n s a n t d e n s i t y PbyD_ratio = ( D6 / D1 ) ^2; // The Power i n p u t r e q u i r e d i n t h e 6 f t v e s s e l i s then Pin = 2* PbyD_ratio // [ hp / 1 0 0 0 g a l ]

// ( b ) // U s i n g Eq . ( 9 . 5 4 ) w i t h same i n p u t power p e r u n i t volume i n b o t h v e s s e l s 24 n6byn1 = ( D6 / D1 ) ^(2/3) 42

25 // b l e n d i n g i n t h e 6 f t 26 t6 = t1 * n6byn1 // [ s ]

v e s s e l would be

43

Chapter 10 Heat Transfer by Conduction


10.1 Scilab Code

Example 10.1 Example 10.1.sce


1 2 3 4 5 6 7 8 9 10 11 12 13 14 15 16 17

clear all ; clc ; // Exmple 1 0 . 1 // Given T1 = 32; // [ F ] T2 = 200; // [ F ] k1 = 0.021; // [ Btu / f t hF ] k2 = 0.032; // [ Btu / f t hF ] A = 25; // [ f t 2 ] B = 6/12; // [ f t ] // a v e r a g e t e m p e r a t u r e and t h e r m a l c o n d u t i v i t y o f t h e wall Tavg = (40+180) /2; // [ F ] kbar = k1 +( Tavg - T1 ) *( k2 - k1 ) /( T2 - T1 ) ; // [ Btu / f t hF ] delta_T = 180 -40; // [ F ] // U s i n g Eq . ( 1 0 . 5 ) q = kbar * A * delta_T / B // [ Btu / h ] Example 10.2 Example 10.2.sce

44

1 2 3 4 5 6 7 8 9 10 11 12 13 14 15 16 17 18 19 20 21 22 23 24 25 26 27 28 29 30 31

clear all ; clc ; // Example 1 0 . 2 // Given delta1 = 4.5/12 ; // [ f t ] k1 = 0.08; // [ Btu / f t hF ] delta2 = 9/12; // [ f t ] k2 = 0.8; // [ f t ] Tin = 1400 // [ F ] Tout = 170 // [ F ] Rc = 0.5; // [ f t 2 hF/ Btu ] // ( a ) // C o n s i d e r i n g u n i t c r o s s s e c t i o n a l a r e a A = 1; // [ f t 2 ] RA = delta1 / k1 ; // [ f t 2 hF/ Btu ] RB = delta2 / k2 ; // [ f t 2 hF/ Btu ] R = RA + RB ; // [ f t 2 hF/ Btu ] delta_T = Tin - Tout ; // [ F ] o v e r a l l t e m p e r a t u r e d r o p // U s i n g Eq . ( 1 0 . 9 ) q = A * delta_T / R // [ Btu / h ] // ( b ) // The t e m p e r a t u r e d r o p i n one s e r i e s o f r e s i s t a n c e s i s to the // i n d i v i d u a l r e s i s t a n c e a s t h e o v e r a l l t e m p e r a t u r e drop i s to the // o v e r a l l r e s i s t a n c e , o r delta_TA = RA * delta_T / R ; // [ F ] // T e m p e r a t u r e a t t h e i n t e f a c e Tf = Tin - delta_TA // [ F ]

// ( c ) The t o t a l r e s i s t a n c e w i l l now i n c l u d e c o n t a c t resistance 32 R = R + Rc ; // [ f t 2 hF/ Btu ] 33 // t h e h e a t l o s s from u n i t s q u a r e a r e a 34 q = delta_T / R // [ Btu / h ]

45

Example 10.3 Example 10.3.sce


1 2 3 4 5 6 7 8 9 10 11 12 13 14 15 16 17 18 19

clear all ; clc ; // Example 1 0 . 3 // Given r1 = 60/2; // [mm] r2 = (50+ r1 ) ; // [mm] k2 = 0.055; // [W/m C ] r3 = 40+ r2 ; // [mm] k3 = 0.05; // [W/m C ] To = 30; // [ C ] Ti = 150; // [ C ] // L o g r i t h i m i c mean f o r s i l i c a l a y e r and c o r k l a y e r rl_s = ( r2 - r1 ) / log ( r2 / r1 ) // [mm] rl_c = ( r3 - r2 ) / log ( r3 / r2 ) // [mm] // U s i n g Eq . ( 1 0 . 1 5 ) and Eq . ( 1 0 . 1 4 ) s i m u l a t a n e o u s l y //And Adding t h e s e two E q u a t i o n s qbyL = ( Ti - To ) /4.13 // [W/m] Example 10.4 Example 10.4.sce

1 2 3 4 5 6 7 8 9 10 11 12 13 14

clear all ; clc ; // Example 1 0 . 4 // Given k = 0.075; // [ Btu / f t hF ] rho = 56.2; // [ l b / f t 3 ] Cp = 0.40; // [ Btu / l b F ] s = 0.5/12; // [ f t . ] Ts = 250; // [ F ] Ta = 70; // [ F ] Tb_bar = 210; // [ F ] // ( a ) 46

15 Temp_diff_ratio = ( Ts - Tb_bar ) /( Ts - Ta ) ; 16 alpha = k /( rho * Cp ) ; 17 // From F i g . 1 0 . 6 18 N_Fo =0.52; 19 tT = N_Fo * s ^2/ alpha // [ h ] 20 21 // ( b ) 22 // S u b s t i t u t i n g i n Eq . ( 1 0 . 2 3 ) 23 QTbyA = s * rho * Cp *( Tb_bar - Ta ) // [ Btu / f t 2 ]

Example 10.5 Example 10.5.sce


1 2 3 4 5 6 7 8 9 10 11 12 13 14 15 16 17 18 19 20 21

clear all ; clc ; // Example 1 0 . 5 // Given Ts = -20; // [ C ] Ta = 5; // [ C ] T = 0; // [ C ] t = 12; // [ h ] alpha = 0.0011; // [m2/ h ] // ( a ) Temp_diff_ratio = ( Ts - T ) /( Ts - Ta ) ; // From F i g . ( 1 0 . 8 ) , Z = 0.91; // t h e r e f o r e d e p t h x = Z *2* sqrt ( alpha * t ) // [m] // ( b ) // From Eq . ( 1 0 . 2 7 ) , t h e p e n e t r a t i o n d i s t a n c e i s x_rho = 3.64* sqrt ( alpha * t ) // [m]

47

Chapter 11 Principles of Heat Flow in Fluids


11.1 Scilab Code

Example 11.1 Example 11.1.sce


1 2 3 4 5 6 7 8 9 10 11 12 13 14 15 16 17

clear all ; clc ; // Example 1 1 . 1 // From Appendix 5 Di = 1.049/12; // [ f t ] Do = 1.315/12; // [ f t ] xw = 0.133/12; // [ f t ] km = 26; // [ Btu / f t hF ] // U s i n g Eq . ( 1 0 . 1 5 ) f o r L o g r i t h m i c mean d i a m e t e r DL bar DL_bar = ( Do - Di ) / log ( Do / Di ) ; // [ f t ] // From T a b l e 1 1 . 1 hi = 180; // [ Btu / f t 2 hF ] ho = 300; // [ Btu / f t 2 hF ] hdi = 1000; // [ Btu / f t 2 hF ] hdo = 500; // [ Btu / f t 2 hF ]

48

18 // O v e r a l l h e a t t r a n s f e r c o e f f i c i e n t 19 Uo = 1/( Do /( Di * hdi ) + Do /( Di * hi ) +( xw * Do ) /( km * DL_bar )

+1/ ho +1/ hdo ) // [ Btu / f t 2 hF ]

49

Chapter 12 Heat Transfer to Fluids without Phase Change


12.1 Scilab Code

Example 12.1 Example 12.1.sce


1 2 3 4 5 6 7 8 9 10 11 12 13 14

clear all ; clc ; // Example 1 2 . 1 To = 230; // [ F ] Ti = 80; // [ F ] // U s i n g T a b l e 1 2 . 1 hi = 400; // [ Btu / f t 2 hF ] ho = 500; // [ Btu / f t 2 hF ] // From Appendix 6 Di = 0.620; // [ i n . ] Do = 0.750; // [ i n . ] // U s i n g Eq . ( 1 2 . 3 9 ) detla_Tt = (1/ hi ) /(1/ hi +( Di /( Do * ho ) ) ) *( To - Ti ) Example 12.2 Example 12.2.sce

1 clear all ; 2 clc ;

50

3 4 5 6 7 8 9 10 11 12 13 14 15 16 17 18 19 20 21 22 23 24 25 26 27 28 29 30 31 32 33 34 35 36 37 38 39

// Example 1 2 . 2 // Given Tb1 = 141; // [ F ] Tb2 = 79; // [ F ] / Tw1 = 65; // [ F ] Tw2 = 75; // [ F ] Vb_bar = 5; // [ f t / s ] rho_b = 53.1; // [ l b / f t 3 ] mu_b = 1.16; // [ l b / f t h ] , Form Appendix 9 k_b = 0.089; // [ Btu / f t hF ] , From Appendix 13 Cp_b = 0.435; // [ Btu / l b F ] , From Appendix 16 // U s i n g Appndix 14 rho_w = 62.3; // [ l b / f t 3 ] mu_w = 2.34; // [ l b / f t h ] k_w = 0.346; // [ Btu / f t hF ] Cp_w = 1; // [ Btu / l b F ]

// S o u l t i o n Tavg_b = ( Tb1 + Tb2 ) /2; // [ F ] Tavg_w = ( Tw1 + Tw2 ) /2; // [ F ] Dit = 0.745/12; // [ f t ] Dot = 0.875/12; // [ f t ] // U s i n g Appendix 5 // The i n s i d e d i a m e t e r o f t h e j a c k e t Dij = 1.610/12; // [ f t ] // From Appendix 6 , t h e i n s i d e s e c t i o n a l a r e a o f t h e c o p p e r t u b e ( f o r a 7/8 i n . BWG 16 t u b e ) S = 0.00303; // [ f t 2 ] // E q u i v a l e n t d i a m e t e r o f t h e a n n u l a r j a c k e t s p a c e De = 4*( %pi /4*( Dij ^2 - Dot ^2) /( %pi *( Dij + Dot ) ) ) ; // [ f t ] mb_dot = Vb_bar * rho_b * S ; // [ l b / s ] // The r a t e o f h e a t f l o w q = mb_dot * Cp_b *( Tb1 - Tb2 ) ; // [ Btu / s ] // mass f l o w r a t e o f w a t e r mw_dot = q /( Cp_w *( Tw2 - Tw1 ) ) ; // [ l b / s ] // Water v e l o c i t y 51

40 41 42 43 44 45 46 47 48 49 50 51 52 53 54 55 56 57 58 59 60 61 62 63 64 65 66 67 68 69

Vw_bar = mw_dot /( %pi /4*( Dij ^2 - Dot ^2) * rho_w ) ; // [ f t / s ] // R e y n o l d s number f o r b e n z e n e and w a t e r Nre_b = Dit * Vb_bar * rho_b *3600/ mu_b ; Nre_w = De * Vw_bar * rho_w *3600/ mu_w ; // P r a n d t l Number f o r b e n z e n e and w a t e r Npr_b = Cp_b * mu_b / k_b ; Npr_w = Cp_w * mu_w / k_w ; // P r e l i m i n a r y e s t i m a t e s o f t h e c o e f f i c i e n t s a r e o b t a i n e d u s i n g Eq . ( 1 2 . 3 2 ) , o m i t t i n g t h e // c o r r e c t i o n f o r v i s c o s i t y r a t i o : // Benzene hi = 0.023* Vb_bar *3600* rho_b * Cp_b /( Nre_b ^0.2* Npr_b ^(2/3) ) ; // [ Btu / f t 2 hF ] // Water ho = 0.023* Vw_bar *3600* rho_w * Cp_w /( Nre_w ^0.2* Npr_w ^(2/3) ) ; // [ Btu / f t 2 hF ] // U s i n g Eq . ( 1 2 . 3 9 ) // T e m p e r a t u r e d r o p o v e r t h e b e n z e n e r e s i s t a n c e delta_Ti = (1/ hi ) /(1/ hi + Dit /( Dot * ho ) ) *( Tavg_b - Tavg_w ) ; // [ F ] Tw = Tavg_b - delta_Ti ; // [ F ] // The v i s c o s i t i e s o f t h e l i q u i d s a t Tw muw_b = 1.45; // [ l b / f t h ] muw_w = 2.42*0.852; // [ l b / f t h ] // U s i n g Eq . ( 1 2 . 2 4 ) , v i s c o s i t y c o r r e c t i o n f a c t o r s p h i are phi_b = ( mu_b / muw_b ) ^0.14; phi_w = ( mu_w / muw_w ) ^0.14; // The c o r r e c t e d c o e f f i c i e n t s a r e hi = hi * phi_b ; // [ Btu / f t 2 hF ] ho = ho * phi_w ; // [ Btu / f t 2 hF ] // The t e m p e r a t u r e d r o p o v e r t h e b e n z e n e r e s i s t a n c e and t h e w a l l t e m p e r a t u r e delta_Ti = (1/ hi ) /(1/ hi + Dit /( Dot * ho ) ) *( Tavg_b - Tavg_w ) ; // [ F ] 52

70 Tw = Tavg_b - delta_Ti // [ F ] 71 // T h i s i s s o c l o s e t o p r e v i o u s l y 72 73 74 75 76

calculated wall temperature that a second approximation // i s u n n e c e s s a r y // U s i n g Eq . ( 1 1 . 2 9 ) , n e g l e c t i n g t h e r e s i s t a n c e o f t h e tube wall Uo = 1/( Dot /( Dit * hi ) +1/ ho ) ; // [ Btu / f t 2 hF ] disp ( The o v e r a l l c o e f f i c i e n t i s ) ; disp ( Btu / f t 2 hF , Uo ) ;

Example 12.3 Example 12.3.sce


1 2 3 4 5 6 7 8 9 10 11 12 13 14 15 16 17 18 19 20 21

clear all ; clc ;

// Example 1 2 . 3 // Given L = 15; // [ f t ] k = 0.082; // [ Btu / f t hF ] Cp = 0.48; // [ Btu / l b F ] T1 = 150; // [ F ] T2 = 250; // [ F ] Tw = 350; // [ F ] // From T a b l e 1 2 . 3 mu1 = 6; // [ cP ] mu2 = 3.3; // [ cP ] mu_w = 1.37; // [ cP ] mu = ( mu1 + mu2 ) /2; // [ cP ] // From Appendix 5 D = 0.364/12; // [ f t ] // v i s c o s i t y c o r r e c t i o n f a c t o r p h i i s phi = ( mu / mu_w ) ^0.14; // Assuming Laminar f l o w and G r a e t z number l a r g e enough t o a p p l y Eq . ( 1 2 . 2 5 ) 22 // U s i n g Eq . ( 1 2 . 2 5 ) 23 // h = k /D 2 p h i ( Cp mdot / ( k L ) ) ( 1 / 3 ) ; 24 //To u s e Eq . ( 1 2 . 1 8 )

53

25

26 27 28 29 30 31 32 33 34 disp ( Btu / f t 2 hF ,h , E x p e c t e d C o e f f i c i e n t ) 35 Ngz = mdot * Cp /( k * L ) ; 36 // T h i s i s l a r g e enough s o t h a t Eq . ( 1 2 . 2 5 ) a p p l i e s , 37 // R e y n o l d s Number 38 Nre = D * mdot /(( %pi /4* D ^2) * mu *2.42) ; 39 // Nre i s i n Laminar Range

Log_T = (( Tw - T1 ) -( Tw - T2 ) ) / log (( Tw - T1 ) /( Tw - T2 ) ) ; // [ F ] // From Eq . ( 1 2 . 1 8 ) // h = Cp 1 0 0 mdot / ( %pi D L Log T ) // From Eq . ( 1 2 . 2 5 ) and Eq . ( 1 2 . 1 8 ) mdot = (4.69/0.233) ^(3/2) ; // [ l b / h ] // and h = 0.233* mdot ; // [ Btu / f t 2 hF ] disp ( l b / h , mdot , o i l f l o w r a t e )

Example 12.4 Example 12.4.sce


1 2 3 4 5 6 7 8 9 10 11 12 13 14 15 16 17 18 19

clear all ; clc ; // Example 1 2 . 4 // Given P = 1; // [ atm ] Vbar = 1.5; // [ f t / s ] Ti = 68; // [ F ] To = 188; // [ F ] Tw = 220; // [ F ] Tbar = ( Ti + To ) /2; // [ F ] D = 2.067/12; // [ f t ] , from Appendix 5 mu = 0.019; // [ cP ] , a t 1 2 8 [ F ] , from Appendix 8 rho = 29/359*(492/(68+460) ) ; // [ l b / f t 3 ] , a t 6 8 [ F ] G = Vbar * rho *3600; // [ l b / f t 2 h ] Nre = D * G /( mu *2.42) ; g = 32.14; // Hence t h e f l o w i s l a m i n a r // A p p l y i n g Eq . ( 1 2 . 2 5 ) 54

20 21 22 23 24 25 26 27 28 29 30 31 32 33 34 35 36 37 38 39 40 41 42 43 44 45 46 47 48 49 50 51 52

Cp = 0.25; // [ Bu/ l b F ] , a t 1 2 8 [ F ] , Appendix 15 k = 0.0163; // [ Btu / f t hF ] , a t 1 2 8 [ F ] , Appendix 12 //By l i n e a r i n t e r p o l a t i o n mu_w = 0.021; // [ cP ] , Appendix 5 // i n t e r n a l c r o s s s e c t i o n a l a r e a o f p i p e i s S = 0.02330; // [ f t 2 ] , Appendix 5 // mass f l o w r a t e mdot = G * S ; // [ l b / h ] // t h e h e a t l o a d q = mdot * Cp *( To - Ti ) ; // [ Btu / h ] // The l o g r i t h m i c mean t e m p e r a t u r e d i f f e r e n c e i s delta_T1 = Tw - To ; // [ F ] delta_T2 = Tw - Ti ; // [ F ] Log_T = ( delta_T1 - delta_T2 ) / log ( delta_T1 / delta_T2 ) ; // [ F ] // h e a t t r a n s f e r c o e f f i c i e n t h = q /A Log T //A = 0 . 5 4 1 L // A l s o from Eq . ( 1 2 . 2 5 ) , t h e h e a t t r a n s f e r coefficient is // h = 2 k /D ( mdot Cp/ k L ) ( 1 / 3 ) (mu/mu w ) ( 1 / 4 ) // E q u a t i n g t h e two r e a l t i o n s h i p s f o r h L = (6.820/0.9813) ^(3/2) ; // [ f t ] // T h i s r e s u l t i s c o r r e c t e d f o r t h e e f f e c t o f n a t u r a l convection //To u s e Eq . ( 1 2 . 8 0 ) beeta = 1/(460+ Tbar ) ; // [ R 1] , a t 1 2 8 [ F ] delta_T = Tw - Tbar ; // [ F ] rho = 0.0676; // [ l b / f t 3 ] // G r a s h o f number Ngr = D ^3* rho ^2* g * beeta * delta_T /( mu *6.72*10^ -4) ^2; // From Eq . ( 1 2 . 8 0 ) phi_n = 2.25*(1+0.01* Ngr ^(1/3) ) / log10 ( Nre ) ; // t h i s i s f a c t o r i s u s e d t o c o r r e c t t h e v a l u e o f L L = L / phi_n ; // [ f t ] disp ( f t ,L , l e n g h t o f h e a t e d s e c t i o n i s )

55

Chapter 13 Heat Transfer to Fluids with Phase Change


13.1 Scilab Code

Example 13.1 Example 13.1.sce


1 2 3 4 5 6 7 8 9 10 11 12 13 14 15 16 17 18

clear all ; clc ; // Example 1 3 . 1 // Given Pa = 1; // [ atm ] lambda = 139.7; // [ Btu / l b ] L = 5; // [ f t ] Tw = 175; // [ F ] hi = 400; // [ Btu / f t 2 hF ] g = 4.17*10^8; // [ f t / h 2 ] Th = 270; // [ F ] rho_f = 65.4; // [ l b / f t 3 ] kf = 0.083; // [ Btu / f t hF ] , from Appendix 13 muf = 0.726; // [ l b / f t h ] , from Appendix 9 Do = 0.75/12; // [ f t ] Di = Do -(2*0.065) /12; // [ f ] // ( a )

56

19 20 21 22 23 24 25 26 27 28 29 30 31 32 33 34 35 36 37 38 39 40 41 42 43 44 45 46 47 48 49 50 51 52

Twall = 205; // [ F ] err = 50; h = 1.13; while ( err >10) delta_To = Th - Twall ; // from Eq . ( 1 3 . 1 1 ) Tf = Th -3*( Th - Twall ) /4; // [ F ] h = h *( kf ^3* rho_f ^2* g * lambda /( delta_To * L * muf ) ) ^(1/4) ; // [ Btu / f t 2 hF ] // U s i n g Eq . ( 1 2 . 2 9 ) delta_Ti = 1/ hi /(1/ hi + Di /( Do * h ) ) *( Th - Tw ) ; // [ F ] Twall_new = Tw + delta_Ti ; // [ F ] err = Twall_new - Twall ; // [ F ] Twall = Twall_new ; // [ F ] end //To c k e c k w h e t h e r t h e f l o w i s a c t u a l l y l a m i n a r Ao = 0.1963* L ; // [ f t 2 ] , from Appendix 6 // t h e r a t e o f h e a t t r a n s f e r q = h * Ao *( Th - Twall ) ; // [ Btu / h ] mdot = q / lambda ; // [ l b / f t h ] disp ( [ Btu / f t 2 hF ] ,h , c o e f f i c i e n t o f chlorobenzene i s )

// ( b ) // For a h o r i z o n t a l c o n d e n s e r , U s i n g Eq . ( 1 3 . 1 6 ) N =6; Twall = 215; // [ F ] err = 50; h = 0.725; muf = 0.68; // [ l b / f t h ] , from Appendix 6 while ( err >10) delta_To = Th - Twall ; // from Eq . ( 1 3 . 1 1 ) Tf = Th -3*( Th - Twall ) /4; // [ F ] h = h *( kf ^3* rho_f ^2* g * lambda /(6* delta_To * Do * muf ) ) ^(1/4) ; // [ Btu / f t 2 hF ] 53 // U s i n g Eq . ( 1 2 . 2 9 ) 57

54 delta_Ti = 1/ hi /(1/ hi + Di /( Do * h ) ) *( Th - Tw ) ; // [ F ] 55 Twall_new = Tw + delta_Ti ; // [ F ] 56 err = Twall_new - Twall ; // [ F ] 57 Twall = Twall_new ; // [ F ] 58 end 59 disp ( [ Btu / f t 2 hF ] ,h , c o e f f i c i e n t o f

chlorobenzene i s ) Example 13.2 Example 13.2.sce


1 clear all ; 2 clc ; 3 4 // Example 1 3 . 2 5 // Given 6 P = 2; // [ atm ] 7 8 // ( a ) 9 // From F i g . 1 3 . 7 10 // C r i t i c a l p r e s s u r e o f b e n z e n e 11 Pc = 47.7; // [ atm ] 12 PbyPc = P / Pc ; 13 // From F i g . 1 3 . 7 t h e o r d i n a t e ( q /A) max/ Pc i s 14 15 16 17 18

about

19 20 21 22 23 // ( b ) 24 // Given 25 P = 0.2; // [ atm ]

1 9 0 , and qbyA_max = 190* Pc *14.696; // [ Btu /h f t 2 ] disp ( Btu /h f t 2 , qbyA_max , The maximum h e a t f l u x i s ) // A l s o from F i g . 1 3 , 7 delta_Tc = 62; // [ F ] disp ( F , delta_Tc , The c r i t i c a l t e m p e r a t u r e difference is ) // f i l m c o e f f i c i e n t h = qbyA_max / delta_Tc ; // [ Btu /h f t 2 F ] disp ( Btu /h f t 2 F ,h , The f i l m c o e f f i c i e n t i s )

58

26 27 28 29 30 31 32 33

PbyPc = P / Pc ; // U s i n g Eq . ( 1 3 . 2 0 ) // n o t i n g t h a t lambda , s i g m a and r h o L a r e n e a r l y c o n s t a n t and r h o L >rho V // qbyA max rho V ( 1 / 2 ) P ( 1 / 2 ) qbyA_max = qbyA_max *(0.2/2) ^(1/2) ; // [ Btu /h f t 2 ] disp ( Btu /h f t 2 , qbyA_max , The maximum h e a t f l u x i s ) // The c r i t i c a l t e m p e r a t u r e d i f f e r e n c e would be g r e a t e r t h a n 100 [ F ] and // t h e f i l m c o e f f i c i e n t would be l e s s t h a n 410 [ Btu /h f t 2 F ]

59

Chapter 14 Radiation Heat Transfer


14.1 Scilab Code

Example 14.1 Example 14.1.sce


1 2 3 4 5 6 7 8 9 10 11 12 13 14 15 16 17 18 19 20

clear all ; clc ; // Example 1 4 . 1 // Given d = 150; // [mm] T1 = 300+272; // [ K ] T3 = 25+273; // [ K ] eps1 = 0.56; eps2 = 1.0; eps3 = eps1 ; sigma = 5.672 // ( a ) // U s i n g Eq . ( 1 4 . 3 8 ) // q12 = s i g m a A1 F12 ( T14 T2 4 ) // q23 = s i g m a A2 F23 ( T24 T3 4 ) // At e q u i l i b r i u m , q12=q23 // From Eq . ( 1 4 . 3 9 ) F12 = 1/(1/ eps1 +1/ eps2 -1)

60

21 F23 = F12 ; 22 //A1 = A2 23 T2 = (100*(( T1 /100) ^4+( T3 /100) ^4) ^(1/4) ) /2^(1/4) ; // 24 25 26 27 28

[K] disp ( F ,T2 , t h e t e m p e r a t u r e o f l a c q u e r e d s h e e t i s )

// ( b ) // From Eq . ( 1 4 . 3 8 ) , h e a t f l u x q12byA = sigma * F12 *(( T1 /100) ^4 -( T2 /100) ^4) ; // [W/m 2] 29 disp ( W/m2 , q12byA , t h e h e a t f l u x i s )

61

Chapter 15 Heat-Exchange Equipment


15.1 Scilab Code

Example 15.1 Example 15.1.sce


1 2 3 4 5 6 7 8 9 10 11 12 13 14 15 16

clear all ; clc ; // Example 1 5 . 1 // Given Ds = 35/12; // [ f t ] Do = 0.75/12; // [ f t ] p = 1/12; // [ f t ] P = 1; // [ f t ] mdot = 10^5; // [ l b / h ] mu_60 = 0.70; // [ cP ] , a t 60 [ F ] , from Appendix 9 mu_140 = 0.38; // [ cP ] , a t 140 [ F ] , from Appendix 9 Cp = 0.41; // [ Btu / l b F ] , from Appendix 16 k = 0.092; // [ Btu / f t hF ] , from Appendix 13

// S h e l l s i d e c o e f f i c i e n t i s f o u n d u s i n g Donohue Eq .(15.4) 17 // From Eq . ( 1 5 . 2 ) , t h e a r e a f o r c r o s s f l o w i s 18 Sc = 2.9167* P *( P - Do / p ) ; // [ f t 2 ] 19 // The number o f t u b e s i n t h e b a f f l e window i s

62

20 21 22 23 24 25 26 27 28 29 30 31 32

33

approximately equal to the f r a c t i o n a l // a r e a o f t h e window f t i m e s t h e t o t a l nmber o f t u b e s . For a 25 p e r c e n t b a f f l e f = 0.1955 Nb = f *828; //Nb 1 6 1 Nb = 161; // U s i n g Eq . ( 1 5 . 1 ) , a r e a o f t h e b a f f l e window Sb = ( f * %pi * Ds ^2/4) -( Nb * %pi * Do ^2/4) ; // [ f t 2 ] // U s i n g Eq . ( 1 5 . 3 ) , t h e mass v e l o c i t i e s a r e Gc = mdot / Sc ; // [ l b / f t 2 h ] Gb = mdot / Sb ; // [ l b / f t 2 h ] Ge = sqrt ( Gc * Gb ) ; // [ l b / f t 2 h ] // U s i n g Eq . ( 1 5 . 4 ) ho = k / Do *(0.2*( Do * Ge /( mu_60 *2.42) ) ^0.6*( Cp * mu_60 *2.42/ k ) ^0.33*( mu_60 / mu_140 ) ^0.14) ; // [ Btu / f t 2 h F] disp ( Btu / f t 2 hF ,ho , The i n d i v i d u a l h e a t t r a n s f e r c o e f f i c e n t of benzene i s ) Example 15.2 Example 15.2.sce

1 2 3 4 5 6 7 8 9 10 11 12 13 14 15 16

clear all ; clc ; // Example 1 5 . 2 // Given Tca = 70; // [ C ] Tcb = 130; // [ C ] Tha = 240; // [ C ] Thb = 120; // [ C ] // S o l u t i o n // U s i n g Eq . ( 1 5 . 7 ) and ( 1 5 . 8 ) neta_h = ( Tcb - Tca ) /( Tha - Tca ) ; Z = ( Tha - Thb ) /( Tcb - Tca ) ; // From F i g 1 5 . 7 a , t h e c o r r e c t i o n f a c t o r i s f o u n d Fg = 0.735; // t h e t e m p e r a t u r e d r o p s a r e 63

17 18 19 20 21 22 23 24

// At s h e l l i n l e t : deltaT_i = Tha - Tcb ; // [ C ] // At s h e l l o u t l e t : deltaT_o = Thb - Tca ; // [ C ] Log_T = ( deltaT_i - deltaT_o ) / log ( deltaT_i / deltaT_o ) ; // t h e c o r r e c t v a l u e o f Log T i s Log_T = Fg * Log_T ; // [ C ] disp ( C , Log_T , The c o r r e c t mean e m p e r a t u r e d r o p i s ) 25 // B e c a u s e o f low v a l u e o f Fg , a 1 2 h e a t e x c h a n g e r i s n o t s u i t a b l e f o r t h i s duty Example 15.3 Example 15.3.sce

1 2 3 4 5 6 7 8 9 10 11 12 13 14 15 16 17 18 19 20 21 22 23

clear all ; clc ; // Exapmle 1 5 . 3 // Given Tca = 70; // [ C ] Tcb = 130; // [ C ] Tha = 240; // [ C ] Thb = 120; // [ C ] // S o l u t i o n // U s i n g Eq . ( 1 5 . 7 ) and ( 1 5 . 8 ) neta_h = ( Tcb - Tca ) /( Tha - Tca ) ; Z = ( Tha - Thb ) /( Tcb - Tca ) ; // U s i n g F i g 1 5 . 7 b , t h e c o r r e c t i o n f a c t o r i s Fg = 0.945; // t h e t e m p e r a t u r e d r o p s a r e // At s h e l l i n l e t : deltaT_i = Tha - Tcb ; // [ C ] // At s h e l l o u t l e t : deltaT_o = Thb - Tca ; // [ C ] Log_T = ( deltaT_i - deltaT_o ) / log ( deltaT_i / deltaT_o ) ; // t h e c o r r e c t v a l u e o f Log T i s Log_T = Fg * Log_T ; // [ C ]

64

24

disp ( C , Log_T , The c o r r e c t mean e m p e r a t u r e d r o p i s ) Example 15.4 Example 15.4.sce

1 2 3 4 5 6 7 8 9 10 11 12 13 14 15 16 17 18 19 20 21 22 23 24 25 26 27 28 29 30 31

clear all ; clc ;

// Example 1 5 . 4 // Given N = 28; xF = 0.5/12; // [ f t ] yF = 0.035/12; // [ f t ] km = 26; // [ Btu / f t hF ] AT = 2.830; // [ f t 2/ f t ] Ab = 0.416; // [ f t 2/ f t ] hi = 1500; // [ Btu / f t 2 hF ] G = 5000; // [ l b /h f t 2 ] Tavg = 130; // [ F ] Tw = 250; // [ F ] mu = 0.046; // [ l b / f t h ] , from Appendix 8 Cp = 0.25; // [ Btu / l b F ] , from Appendix 15 k = 0.0162; // [ Btu / f t hF ] , from Appendix 12 ID_shell = 3.068/12; // [ f t ] , from Appendix 5 OD_pipe = 1.9/12; // [ f t ] , from Appendix 5 // c r o s s s e c t i o n a l a r e a o f s h e l l s p a c e Ac = %pi /4*( ID_shell ^2 - OD_pipe ^2) -N * xF * yF // [ f t 2 ] // The p e r i m e t e r o f a i r s p a c e Ap = %pi * ID_shell + AT ; // [ f t ] // h y d r a u l i c r a d i u s rh = Ac / Ap ; // [ f t ] // e q u i v a l e n t d i a m e t e r De = 4* rh ; // [ f t ] // R e y n o l d s Number Nre = De * h / mu // I n c o m p u t i n g mu w t h e r e s i s t a n c e o f t h e w a l l and t h e steam f i l m 32 // a r e c o n s i d e r e d n e g l i g i b l e , s o 65

33 mu_w = 0.0528; // [ l b / f t h ] 34 Npr = mu * Cp / k 35 // U s i n g F i g . 1 5 . 1 7 , t h e h e a t t r a n s f e r f a c t o r i s 36 jh = 0.0031; 37 ho = jh * Cp * G *( mu / mu_w ) ^0.14/ Npr ^(2/3) ; // [ Btu / f t 2 h

F ]
38 39 40

41 42 43 44 45 46 47 48 49 50 51 // U s i n g Eq . ( 1 5 . 1 0 ) , t h e o v e r a l l c o e f f i c i e n t 52 Ut = 1/( Ai /( ho *( netaF * AF + Ab ) ) +( xw * Dt /( km * DLbar ) ) +1/

// For r e c t a n g u l a r f i n s , d i s r e a g r d i n g t h e c o n t r i b u t i o n o f the ends o f the f i n s to // t h e p e r i m e t e r , Lp = 2L and S = Lyf , where y f i s t h e f i n t h i c k n e s s and L i s t h e // l e n g t h o f t h e f i n . Then , from Eq . ( 1 5 . 1 1 ) aFxF = xF * sqrt (2* ho /( km * yF ) ) ; // From F i g . 1 5 . 1 6 netaF = 0.93; Dt = 1.610/12; // [ f t ] , from Appendix 5 DLbar = ( OD_pipe - Dt ) / log ( OD_pipe / Dt ) ; // [ f t ] Ai = %pi * Dt *1.0; // [ f t 2 ] AF = AT - Ab ; // [ f t 2/ f t ] xw = ( OD_pipe - Dt ) /2; // [ f t ]

hi ) ; // [ Btu / f t 2 hF ] 53 disp ( Btu / f t 2 hF ,Ut , The o v e r a l l h e a t t r a n s f e r coefficent is )

66

Chapter 16 Evaporation
16.1 Scilab Code

Example 16.1 Example 16.1.sce


1 2 3 4 5 6 7 8 9 10 11 12 13 14 15 16 17 18 19

clear all ; clc ; // Example 1 6 . 1 // Given mdot = 20000; // [ l b / h ] xin = 0.20; xout = 0.50; Pg = 20; // [ l b f / i n . 2 ] Pabs = 1.93; // [ l b f / i n . 2 ] U = 250; // [ Btu / f t 2 hF ] Tf = 100; // [ F ] // S o l u t i o n // t h e amount o f w a t e r i n f e e d and t h i c k l i q u o r , from material balance w_feed = 80/20; // [ l b / p e r pound o f s o l i d ] w_liquor = 50/50; // [ l b / p e r pound o f s o l i d ] // w a t e r e v a p o r a t e d w_eva = w_feed - w_liquor ; // [ l b / p e r pound o f s o l i d ]

67

20 21 22 23 24 25 26 27 28 29 30 31 32 33 34 35 36 37 38 39 40 41 42 43 44 45 46 47 48

// o r w_eva = w_eva * mdot * xin ; // [ l b / h ] // Flow r a y e o f t h i c k l i q u o r i s ml_dot = mdot - w_eva // [ l b / h ] // Steam consumed // S i n c e w i t h s t r o n g s o l u t i o n s o f NaOH t h e h e a t o f d i l u t i o n i s not n e g l i g i b l e , // t h e r a t e o f h e a t t r a n s f e r i s f o u n d from Eq . ( 1 6 . 4 ) and F i g . 1 6 . 8 . // The v a p o r i z t i o n t e m p e r a t u r e o f t h e 50 p e r c e n t s o l u t i o n a t a p r e s s u r e o f 100 mmHg // i s f o u n d a s f o l l o w s Tb_w = 124; // [ F ] , a t 100 mmHg, from Appendix 7 Tb_s = 197; // [ F ] , from F i g . 1 6 . 8 BPE = Tb_s - Tb_w ; // [ F ] // From F i g . 1 6 . 8 , t h e e n t h a l p i e s o f t h e f e e d and t h i c k l i q u o r are found Hf = 55; // [ Btu / l b ] , 20% s o l i d s , 100 [ F ] H = 221; // [ Btu / l b ] , 50% s o l i d s , 197 [ F ] // E n t h a l p y o f t h e l e a v i n g w a t e r v a p o r i s f o u n d from t h e steam t a b l e Hv = 1149; // [ Btu / l b ] , At 197 [ F ] and 1 . 9 3 [ l b f / i n .2] // E n t h a l p y o f t h e v a p o r l e a v i n g t h e e v a p o r a t o r lambda_s = 939; // [ Btu / l b ] , At 20 [ l b f / i n . 2 ] , from Appendix 7 // U s i n g Eq . ( 1 6 . 4 ) , t h e r a t e o f h e a t t r a n s f e r and steam c o n s u m p t i o n q = ( mdot - ml_dot ) * Hv + ml_dot * H - mdot * Hf ; // [ Btu / h ] ms_dot = q / lambda_s ; // [ l b / h ] disp ( l b / h , ms_dot , steam consumed i s ) // Economy Economy = ml_dot / ms_dot disp ( Economy , Economy ) // H e a t i n g S u r f a c e // The c o n d e n s a t i o n t e m p e r a t u r e o f t h e steam i s 259 [ F ] , the heating area required i s 68

49 50 A = q /( U *(259 -197) ) // [ f t 2 ] 51 disp ( f t 2 ,A , h e a t i n g a r e a r e q u i r e d

is )

Example 16.2 Example 16.2.sce


1 2 3 4 5 6 7 8 9 10 11 12 13 14 15 16 17 18 19 20 21 22 23 24

clear all ; clc ; // Example 1 6 . 2 // Given Ti = 108; // [ C ] Tl = 52; // [ C ] U1 = 2500; // [W/m 2 ] U2 = 2000; // [W/m 2 ] U3 = 1000; // [W/m 2 ] // S o l u t i o n // T o t a l t e m p e r a t u r e d r o p delta_T = Ti - Tl ; // [ C ] // From Eq . ( 1 6 . 1 3 ) , t h e t e m p e r a t u r e d r o p s i n s e v e r a l e f f e c t s w i l l be // a p p r o x i m a e l y i n v e r s e l y p r o p o r t i o n a l t o t h e c o e f i c i e n t s . Thus delta_T1 = 1/ U1 /(1/ U1 +1/ U2 +1/ U3 ) * delta_T ; // [ C ] delta_T2 = 1/ U2 /(1/ U1 +1/ U2 +1/ U3 ) * delta_T ; // [ C ] delta_T3 = 1/ U3 /(1/ U1 +1/ U2 +1/ U3 ) * delta_T ; // [ C ] // C o n s e q u e n t l y t h e b o i l i n g p o i n t s w i l l be Tb1 = Ti - delta_T1 ; // [ C ] Tb2 = Tb1 - delta_T2 ; // [ C ] disp ( C ,Tb1 , The b o i l i n g p o i n t i n t h e f i r s t e f f e c t is ) disp ( C ,Tb2 , The b o i l i n g p o i n t i n t h e s e c o n d e f f e c t is ) Example 16.3 Example 16.3.sce

clear all ; 69

2 3 4 5 6 7 8 9 10 11 12 13 14 15 16 17 18 19 20 21 22 23 24 25 26

clc ; // Example 1 6 . 3 // Given mdot_ft = 60000; // [ l b / h ] xin = 0.10; Tin = 180; // [ F ] xout = 0.50 Ps = 50; // [ l b f / i n . 2 ] Tc = 100; // [ F ] // S o l u t i o n // From T a b l e 1 6 . 2 U1 = 700; // [ Btu / f t 2 hF ] U2 = 1000; // [ Btu / f t 2 hF ] U3 = 800; // [ Btu / f t 2 hF ] // The t o t a l r a t e o f e v a p o r a t i o n i s c a l c u l a t e d from an o v e r a l l m a t e r i a l b a l a n c e // a s s u m i n g t h e s o l d s go t h r o u g h t h e e v a p o r a t o r without l o s s // T a b l e 1 6 . 3 mdot_fs = 6000; // [ l b / h ] mdot_fw = 54000; // [ l b / h ] mdot_lt = 12000; // [ l b / h ] mdot_ls = 6000; // [ l b / h ] mdot_lw = 6000; // [ l b / h ] w_evap = mdot_ft - mdot_fs ; // [ l b / h ]

70

Chapter 17 Equilibrium-Stage Operations


17.1 Scilab Code

Example 17.1 Example 17.1.sce


1 2 3 4 5 6 7 8 9 10 11 12 13 14 15 16 17

clear all ; clc ; // Example 1 7 . 1 // Given yb = 0.30;

// L e t Vb = 100; // [ mol ] Ace_in = yb * Vb ; // [ mol ] Air_in = Vb - Ace_in ; // [ mol ] // 97 p e r c e n t a c e t o n e aborbed , A c e t o n e l e a v i n g i s Ace_out = 0.03* Ace_in ; // [ mol ] ya = Ace_out /( Air_in + Ace_out ) ; // A c e t o n e a b s o r b e d Ace_abs = Ace_in - Ace_out ; // [ mol ] // 10 p e r c e n t a c e t o n e i n t h e l e a v i n g s o l u t i o n and no acetone in the e n t e r i n g o i l 18 Lb = Ace_abs /0.1; // [ mol ] 19 La = Lb - Ace_abs ; // [ mol ]

71

20 21 22 23 24 25 26 27 28 29 30 31 32 33 34 35 36 37 38 39

//To f i n d o u t a s i n t e r m e d i a t e p o i n t on t h e o p e r a t i n g l i n e , making an a c e t o n e b a l a n c e // a r o u n d t h e t o p p a r t o f t h e tower , a s s u m i n g a p a r t i c u l a r v a l u e o f yV t h e m o l e s o f // a c e t o n e l e f t i n t h e g a s . for i =1:30 y ( i ) = i /( i + Air_in ) ; // The m o l e s o f a c e t o n e l o s t by t h e g a s i n t h e s e c i o n , must e q u a l t o t h e m o l e s g a i n e d by // t h e l i q u i d Ace_lost = i - Ace_out ; // [ mol ] // Hence x ( i ) = Ace_lost /( La + Ace_lost ) ; end xe = linspace (0.001 ,0.15 ,100) ; ye = 1.9* xe ; plot (x , y ) plot ( xe , ye , r ) xlabel ( x ) ylabel ( y ) legend ( O p e r a t i n g l i n e , E q u i l i b r i u m l i n e ) title ( Diagram Example 1 7 . 1 ) // The number o f i d e a l s t a g e s d e t e r m i n e d from F i g i s 4

72

Figure 17.1: Diagram for Example 17.1 Example 17.2 Example 17.2.sce
1 clear all ; 2 clc ; 3 4 // Example 1 7 . 2 5 // Given 6 Nreal = 7; 7 VbyL = 1.5; 8 m = 0.8; 9 yb = 0;

73

10 11 12 13 14 15 16 17 18 19 20 21 22 23 24 25 26 27 28 29 30 31 32 33 34 35 36 37 38 39 40 41 42 43

xb_star = 0; // xb =0.1 xa ; // ( a ) // S t r i p p i n g F a c t o r S = m * VbyL ; // From an ammonia b a l a n c e , // ya =0.9 xa /VbyL ; // A l s o // x a s t a r = ya /m // U s i n g Eq . ( 1 7 . 2 8 ) //N = l n ( ( xa 0.75 xa ) / ( 0 . 1 xa 0) ) / l n ( S ) N = log (0.25/0.1) / log ( S ) ; disp (N , Number o f i d e a l t r a y s r e q u i r e d a r e ) stage_eff = N / Nreal *100; disp ( % , stage_eff , S t a g e E f f i c i e n c y i s ) // ( b ) VbyL = 2; S = m * VbyL ; // Then , // L e t A = ( xa x a s t a r ) / xb A = exp (5.02) ; // L e t f be t h e f r a c t i o n o f NH3 removed . Then xb = (1 f ) xa . //By a m a t e r i a l b a l a n c e // y = L/V ( xa xb ) = 1 / 2 ( xa (1 f ) xa )= 1/2 f xa // x a s t a r = ya /m = 0 . 5 f xa / 0 . 8 = 0 . 6 2 5 f xa // Thus , // xa x a s t a r = ( 1 0 . 6 2 5 f ) xa // Also , // xa x a s t a r = 1 0 . 5 9 xb = 1 0 . 5 9 ( 1 f ) xa // from t h e s e f = 0.962 disp ( % ,f , p e r c e n t a g e r e m o v a l o b t a i n e d i n t h i s c a s e is )

74

Chapter 18 Distillation
18.1 Scilab Code

Example 18.1 Example 18.1.sce


1 2 3 4 5 6 7 8 9 10 11 12 13 14 15 16 17 18 19 20

clear all ; clc ; // Example 1 8 . 1 // Given xF = 0.50; P = 1; // [ atm ] f =0.0001:0.2:1.2; A = -(1./ f -1) ; x = [0.01:0.01:1]; for i =1: length ( f ) y (i ,:) = - A ( i ) * x + xF / f ( i ) end // From F i g . 1 8 . 2 xB = [0.50 ,0.455 ,0.41 ,0.365 ,0.325 ,0.29]; yD = [0.71 ,0.67 ,0.63 ,0.585 ,0.54 ,0.5]; // From F i g 1 8 . 3 T = [92.2 ,93.7 ,95.0 ,96.5 ,97.7 ,99]; plot (f , T ./100 , f , xB ,f , yD ) xlabel ( f m o l e s v a p o r i z e d p e r mole o f f e e d )

75

21 22

ylabel ( C o n c e n t r a t i o n , mole f r a c t i o n Benzene ) legend ( T e m p e r a t u r e (C) 10 0 , Con . o f Bnzene i n l i q u i d , Con . o f Bnzene i n v a p o r )

Figure 18.1: Results of Example 18.1 Example 18.2 Example 18.2.sce


1 clear all ; 2 clc ; 3 4 // Example 1 8 . 2

76

5 6 7 8 9 10 11 12 13 14 15 16 17 18 19 20 21 22 23 24 25 26 27 28 29 30 31 32 33 34 35 36 37

// Given mdot = 30000; // [ kg /h ] wF_b = 40; wD = 97; wB = 2; R = 3.5; lambda_b = 7360; // [ c a l / g mol ] lambda_t = 7960; // [ c a l / g mol ] alpha = 2.5; TB = 95; // [ C ] TF = 20; // [ C ] P = 1; // [ atm ] Mb = 78; Mt = 92; Cp = 0.44; // [ c a l / gC ] // ( a ) // The c o n c e n t r a t i o n s o f f e e d , o v e r h e a d and b o t t o m s i n mole f r a c t i o n o f b e n z e n e a r e xF = ( wF_b / Mb ) /( wF_b / Mb +((100 - wF_b ) / Mt ) ) ; xD = ( wD / Mb ) /( wD / Mb +((100 - wD ) / Mt ) ) ; xB = ( wB / Mb ) /( wB / Mb +((100 - wB ) / Mt ) ) ; // The a v e r a g e m o l e c u l a r w e i g h t o f t h e f e e d i s Mavg = 100/( wF_b / Mb +(100 - wF_b ) / Mt ) ; // t h e a v e r a g e h e a t o f v a p o r i z a t i o n lambda_avg = xF * lambda_b +(1 - xF ) * lambda_t ; // [ c a l / g mol ] // Feed r a t e F = mdot / Mavg ; // [ kg mol / h ] // U s i n g Eq . ( 1 8 . 5 ) , by o v e r a l l b e n z e n e b a l a n c e D = F *( xF - xB ) /( xD - xB ) ; // [ kg mol / h ] B = F - D ; // [ kg mol / h ] disp ( r e s p e c t i v e l y , kg mol / h ,B , kg mol / h ,F , t h e mole o f o v e r h e a d and bottom p r o d u c t s a r e )

// ( b ) D e t e m i n a t i o n o f number o f i d e a l p l a t e s and position of feed plate 38 // ( i ) 77

39 40 41 42 43

44 45 46 // ( i i ) 47 // The l a t e n t h e a t o f v a p o r i z a t i o n o f t h e f e e d 48 lambda = lambda_avg / Mavg ; // [ c a l / g ] 49 // U s i n g Eq . ( 1 8 . 2 4 ) 50 q = 1+ Cp *( TB - TF ) / lambda ; 51 // From Eq . ( 1 8 . 3 1 ) 52 slope = -q /(1 - q ) ; 53 // From F i g . 1 8 . 1 7 54 // I t i s f o u n d t h a t a r e b o i l e r and 10 i d e a l p l a t e s

// U s i n g F i g . 1 8 . 1 6 // Drawing t h e f e e d l i n e w i t h f = 0 on e q u i l i b r i u m diagram , // P l o t t i n g t h e o p e r a t i n g l i n e s w i t h i n t e r c e p t from Eq . ( 1 8 . 1 9 ) i s 0 . 2 1 6 //By c o u n t i n g t h e r e c t a n g u l a r s t e p s i t i s f o u n d t h a t , b e s i d e s the r e b o i l e r , // 11 i d e a l p l a t e s a r e neded and f e e d s h o u l d be i n t r o d u c e d on t h e 7 t h p l a t e from // t h e t o p .

a r e n e e d e d and f e e d i s t o be i n t r o d u c e d 55 // on t h e f i f t h p l a t e
56 57 // ( i i i ) 58 q = 1/3; 59 slope = -q /(1 - q ) ; 60 // From F i g . 1 8 . 1 8 61 // I t c a l l s f o r a r e b o i l e r and 12 p l a t e s , w i t h t h e

f e e d e n t e r i n g on t h e 7 t h p l a t e
62 63 // ( c ) 64 // v a p o r f l o w i n t h e r e c t i f y i n g s e c t i o n i s 65 V = 4.5* D ; // [ kg mol / h ] 66 lambda_s = 522; // [ c a l / g ] , From Appendix 7 67 q = [1 , 1.37 , 0.333] 68 // U s i n g Eq . ( 1 8 . 2 7 ) 69 Vbar = V - F *(1 - q ) 70 // U s i n g Eq . ( 1 8 . 3 2 ) , steam r e q u i r e d

78

71 72

ms_dot = lambda_t / lambda_s * Vbar ; // [ kg /h ] disp ( r e s p e c t i v e l y , kg /h , ms_dot (3) , kg /h , ms_dot (2) , kg /h , ms_dot (1) , t h e steam c o n s u m p t i o n i n the above t h r e e c a s e s i s )

73 74 75 // ( d ) 76 Tw1 = 25; // [ C ] 77 Tw2 = 40; // [ C ] 78 // The c o o l i n g w a t e r n e e d e d 79 80 81 82

i s same i n a l l c a s e s ,

U s i n g Eq . ( 1 8 . 3 3 ) mw_dot = V * lambda_t /( Tw2 - Tw1 ) ; // [ kg /h ] rho_25 = 62.24*16.018; // [ kg /m 3 ] vw_dot = mw_dot / rho_25 ; // [m3/ h ] disp ( m3/ h , vw_dot , c o o l i n g w a t e r n e e d e d i s ) Example 18.3 Example 18.3.sce

1 2 3 4 5 6 7 8 9 10 11 12 13 14 15 16 17 18 19 20

clear all ; clc ; // Example 1 8 . 3 // Given mdot = 30000; // [ kg /h ] wF_b = 40; wD = 97; wB = 2; R = 3.5; lambda_b = 7360; // [ c a l / g mol ] lambda_t = 7960; // [ c a l / g mol ] alpha = 2.5; TB = 95; // [ C ] TF = 20; // [ C ] P = 1; // [ atm ] Mb = 78; Mt = 92; Cp = 0.44; // [ c a l / gC ] // S o l u t i o n 79

21 xF = ( wF_b / Mb ) /( wF_b / Mb +((100 - wF_b ) / Mt ) ) ; 22 xD = ( wD / Mb ) /( wD / Mb +((100 - wD ) / Mt ) ) ; 23 xB = ( wB / Mb ) /( wB / Mb +((100 - wB ) / Mt ) ) ; 24 // The a v e r a g e m o l e c u l a r w e i g h t o f t h e f e e d i s 25 Mavg = 100/( wF_b / Mb +(100 - wF_b ) / Mt ) ; 26 // t h e a v e r a g e h e a t o f v a p o r i z a t i o n 27 lambda_avg = xF * lambda_b +(1 - xF ) * lambda_t ; // [ c a l / g 28 29 30 31 32 33 34 35 36 37 // ( a ) 38 // U s i n g Eq . ( 1 8 . 4 3 ) 39 RDm = ( xD - yprime ) ./( yprime - xprime ) 40 disp ( r e s p e c t i v e l y , RDm (3) , RDm (2) , RDm (1) , Minimum

mol ] // Feed r a t e F = mdot / Mavg ; // [ kg mol / h ] // U s i n g Eq . ( 1 8 . 5 ) , by o v e r a l l b e n z e n e b a l a n c e D = F *( xF - xB ) /( xD - xB ) ; // [ kg mol / h ] B = F - D ; // [ kg mol / h ] // U s i n g T a b l e 1 8 . 3 , i n a l l t h r e e c a s e s r e s p e c t i v e l y xprime = [0.44 ,0.521 ,0.3]; yprime = [0.658 ,0.730 ,0.513];

Reflux Ratio f o r three c a s e s i s )


41 42 43

// ( b ) // For minimum umber o f p l a t e s the , t h e r e f l u x r a t i o i s i n f i n i t e , the operating l i n e s 44 // c o i n c i d e s w i t h t h e d i a g o n a l , and t h e r e a r e no d i f f e r e n c e s between the t h r e e c a s e s . 45 // The p l o t i s g i v e n by F i g 1 8 . 2 2 . A r e b o i l e r and e i g h t p l a t e s are needed . Example 18.4 Example 18.4.sce
1 clear all ; 2 clc ; 3 4 // Example 1 8 . 4 5 // Given

80

6 7 8 9 10 11 12 13 14 15 16 17 18 19 20 21 22

xa = 0.02; Vbar = 0.2; // [ mol / mol o f Feed ] xb = 0.0001; yb = 0; xe = 0:0.01:1; m = 9 ye = m * xe ; // L e t F = 1; // [ mol ] Lbar = F ; // [ mol ]

// S o l u t i o n ya_star = m * xa ; yb_star = m * xb ; //By o v e r a l l e t h o n a l b a l a n c e ya = Lbar / Vbar *( xa - xb ) + yb // U s i n g Eq . ( 1 7 . 2 7 ) , As b o t h o p e r t i n g l i n e s and equilibrium l i n e s are s t r a i g h t 23 N = log (( ya - ya_star ) /( yb - yb_star ) ) / log (( yb_star ya_star ) /( yb - ya ) ) ;
24 25

disp (N , I d e a l p l a t e s n e e d e d a r e ) Example 18.6 Example 18.6.sce

1 2 3 4 5 6 7 8 9 10 11 12 13

clear all ; clc ; // Example 1 8 . 6 // Given xF = 0.40; P = 1; // [ atm ] D = 5800; // [ kg /h ] R = 3.5; LbyV = R /(1+ R ) ; // S o l u t i o n // P h y s i c a l p r o p e r t i e s o f m e t h a n o l M = 32; 81

14 Tnb = 65; // [ C ] 15 rho_v = M *273/(22.4*338) ; // [ kg / 3 ] 16 rho_l_0 = 810; // [ kg /m 3 ] , At 0C , from P e r r y , 17 18 19 20 21 22 23 24 25 26 27 28 29 30 31 32 33 34 35 36 37 38 39 40

C h e m i c a l E n g i n e e r s Handbook rho_l_20 = 792; // [ kg /m 3 ] , At 20C , from P e r r y , C h e m i c a l E n g i n e e r s Handbook rho_l = 750; // [ kg /m 3 ] , At 65C sigma = 19; // [ dyn /cm ] , from Lange s Handbook o f Chemistry // ( a ) // Vapor v e l o c i t y and column d i a m e t e r // U s i n g F i g . 1 8 . 2 8 , t h e a b s c i s s a i s abscissa = LbyV *( rho_v / rho_l ) ^(1/2) ; // f o r 18 i n . p l a t e s p a c i n g Kv = 0.29; // A l l o w a b l e v a p o r v e l o c i t y uc = Kv *(( rho_l - rho_v ) / rho_v ) ^(1/2) *( sigma /20) ^(0.2) ; // [ f t / s ] // Vapor f l o w r a t e V = D *( R +1) /(3600* rho_v ) ; // [m3/ s ] // C r o s s s e t i o n a l a r e a o f t h e column Bubbling_area = V /2.23; // [m 2 ] // I f t h e b u b b l e a r e a i s 0 . 7 o f t h e t o t a l column a r e a Column_area = Bubbling_area /0.7; // [m 2 ] // Column d i a m e t e r Dc = sqrt (4* Column_area / %pi ) ; // [m] disp ( r e s p e c t i v e l y , m ,Dc , and , f t / s ,uc , t h e a l l o w a b l e v e l o c i t y and colmn d i a m e t e r a r e )

// ( b ) // P r e s s u r e d r o p : // Area o f one u n i t o f t h r e e h o l e s on a t r a n g u l a r 3/4 i n . p i t c h i s 41 // 1 / 2 3 / 4 ( 3 / 4 s q r t ( 3 / 2 ) ) i n . 2 . The h o l e a r e a i n this section ( half a hole ) i s 42 // 1/2 %pi / 4 ( 1 / 4 ) 2 i n . 2 . Thus t h e h o l e a r e a i s %pi /12864/9 s q r t ( 3 ) , or 10.08 percent 43 // o f t h e b u b b l i n g a r e a . 82

44 45 46 47 48 49 50 51 52 53 54 55 56

57 58 59 60 61 62 63 64 65 66 67 68 69 70 // ( c ) 71 // F r o t h h e i g h t i n t h downcomer : 72 // U s i n g Eq . ( 1 8 . 6 2 ) . , E s t i m a t i n g 73 hf_L = 10; // [mm m e t h a n o l ] 74 // Then , 75 Zc = (2* hI ) + hd + hf_L ; // [mm] 76 // from Eq . ( 1 8 . 6 3 ) 77 Z = Zc /0.5; // [mm] 78 disp ( mm m e t h a n o l ,Z , F r o t h h e i g h t i n t h e downcomer

// Vapor v e l o c i t y t h r o u g h h o l e s : uo = 2.23/0.1008; // [m/ s ] // U s i n g Eq . ( 1 8 . 5 8 ) , // From F i g . 1 8 . 2 7 Co = 0.73; hd = 51.0* uo ^2* rho_v /( Co ^2* rho_l ) ; // [mm m e t h a n o l ] // Head o f l i q u i d on p l a t e : // Weir h e i g h t hw = 2*25.4; // [mm] // H e i g h t o f t h e l i q u i d a b o v e w e i r : // Assuming t h e downcomer a r e a i s 15 p e r c e n t o f t h e column // a r e a on e a c h s i d e o f t h column . From P e r r y , t h e chord // l e n g t h f o r s u c h a s e g m e n t a l downcomer i s 1 . 6 2 t i m e s the radius // o f t h e colmn , s o Lw = 1.62*2.23/2; // [m] // L i q i u d Flow r a t e : qL = D *( R +1) /( rho_l *60) ; // [m3/ min ] // From Eq . ( 1 8 . 6 0 ) how = 43.4*( qL / Lw ) ^(2/3) // [mm] // From Eq . ( 1 8 . 5 9 ) , w i t h beeta = 0.6; hI = beeta *( hw + how ) ; // [mm] // T o t a l h e i g h t o f l i q u i d , from Eq . ( 1 8 . 6 2 ) hT = hd + hI ; // [mm] disp ( mm m e t h a n o l ,hT , p r e s s u r e d r o p p e r p l a t e i s )

83

is ) Example 18.7 Example 18.7.sce


1 2 3 4 5 6 7 8 9 10 11 12 13 14 15 16 17 18 19 20 21 22 23 24 25 26 27 28 29 30

clear all ; clc ; // Example 1 8 . 7 // Given xF = 0.40; P = 1; // [ atm ] D = 5800; // [ kg /h ] R = 3.5; LbyV = R /(1+ R ) ; // S o l u t i o n // P h y s i c a l p r o p e r t i e s o f m e t h a n o l M = 32; Tnb = 65; // [ C ] rho_v = M *273/(22.4*338) ; // [ kg / 3 ] rho_l_0 = 810; // [ kg /m 3 ] , At 0C , from P e r r y , C h e m i c a l E n g i n e e r s Handbook rho_l_20 = 792; // [ kg /m 3 ] , At 20C , from P e r r y , C h e m i c a l E n g i n e e r s Handbook rho_l = 750; // [ kg /m 3 ] , At 65C sigma = 19; // [ dyn /cm ] , from Lange s Handbook o f Chemistry // ( a ) // Vapor v e l o c i t y and column d i a m e t e r // U s i n g F i g . 1 8 . 2 8 , t h e a b s c i s s a i s abscissa = LbyV *( rho_v / rho_l ) ^(1/2) ; // f o r 18 i n . p l a t e s p a c i n g Kv = 0.29; // A l l o w a b l e v a p o r v e l o c i t y uc = Kv *(( rho_l - rho_v ) / rho_v ) ^(1/2) *( sigma /20) ^(0.2) /(3.2825112) ; // [ f t / s ] // From Eq . ( 1 8 . 7 1 ) , t h e F f a c t o r i s F = uc * sqrt ( rho_v ) ; disp (F , t h e v a l u e o f F f a c t o r i s ) 84

Example 18.8 Example 18.8.sce


1 2 3 4 5 6 7 8 9 10 11 12 13 14 15 16 17 18 19 20 21 22 23 24 25 26 27

clear all ; clc ; // Example 1 8 . 8 // Given xOA = 0.15; xAi = 0.015; P = 1; // [ atm ] // S o l u t i o n Pv = 3.4; // [ atm ] alpha_o = 3.4; // a t 36 C Tbi = 27; // [ C ] alpha_i = 3.6 alpha = ( alpha_o + alpha_i ) /2; // B a s i s 1 mol Feed nOA = 0.15; // [ mol ] nA = 0.015; // [ mol ] nOB = 0.85; // [ mol ] // U s i n g Eq . ( 1 8 . 7 9 ) nB = nOB *( nA / nOA ) ^(1/ alpha ) ; // [ mol ] n = nA + nB ; // [ mol ] xA = nA / n ; disp ( mol ,nB , p e n t a n e removed i s ) disp ((1 - xA ) , xB ,xA , xA , c o m p o s i t i o n o f t h e remaining l i q u i d i s )

85

Chapter 19 Introduction to Multicomponent Distillation


19.1 Scilab Code

Example 19.2 Example 19.2.sce


1 2 3 4 5 6 7 8 9 10 11 12 13 14 15 16 17 18

clear all ; clc ; // Example 1 9 . 2 // Given P = 1.2; // [ atm ] Tb = 97; // [ C ] Td = 105; // [ C ] f = 0.6; xF (1) = 0.33; xF (2) = 0.37; xF (3) = 0.30; // S o l u t i o n // ( a ) // From F i g . 1 9 . 1

86

19 K (1) = 2.68/ P ; 20 K (2) = 1.21/ P ; 21 K (3) = 0.554/ P ; 22 // I n Eq . ( 1 9 . 1 2 ) , t h e r i g h t hand s i d e 23 24 25 26

27 28

of the equation becomes RHS = ( xF ./( f *( K -1) +1) ) ; RHS2 = sum ( RHS ) disp ( C ,Td , f l a s h t e m p e r a t u r e i s ) ; disp ( p e r c e n t , RHS (3) , n o c t a n e e x a n e , p e r c e n t , RHS (2) , n h e p t a n e , p e r c e n t , RHS (1) , nh e x a n e , Composition o f the l i q u i d product i s ); y = RHS .* K ; disp ( p e r c e n t ,y (3) , n o c t a n e , p e r c e n t ,y (2) , n h e p t a n e , p e r c e n t ,y (1) , nh e x a n e , C o m p o s i t i o n o f the vapor product i s ); // ( b ) //To d e t e r m i n e t h e t e m p e r a t u r e o f t h e f e e d b e f o r e flashing , // an e n t h a l p y b a l a n c e i s made u s i n g 105 C a s t h e r e f e r e n c e temperature . // The h e a t s o f v a p o r i z a t i o n a t 105 C and t h e a v e r a g e heat c a p a c i t i e s of the // l i q u i d from 105 t o 200 C a r e o b t a i n e d from t h e literature . Cp = [62 ,70 ,78] ; // [ c a l / molC ] , Cp ( 1 ) = nhexane , Cp ( 2 ) = nh e p t a n e , and Cp ( 3 ) = n o c t a n e delta_Hv = [6370 ,7510 ,8560] ; // [ c a l / mol ] , d e l t a h v ( 1 ) = nhexane , d e l t a h v ( 2 ) = nh e p t a n e , and d e l t a h v ( 3 ) = n o c t a n e // Based on l i q u i d a t 105 C , t h e e n t h a l p i e s o f t h e product are H_vapor = f * sum (( y .* delta_Hv ) ) // [ c a l ] H_liquid = 0; // For t h e f e e d Cp_bar = sum ( xF .* Cp ) // [ c a l / molC ] T0 = H_vapor / Cp_bar + Td ; disp ( C ,T0 , p r e h e a t t e m p e r a t u r e i s ) 87

29 30 31 32 33 34 35 36

37 38 39 40 41 42 43

Example 19.3 Example 19.3.sce


1 clear all ; 2 clc ; 3 4 // Example 1 9 . 3 5 // Given 6 xF = [0.33 ,0.37 ,0.30] ; // [ mole f r a c t i o n ] xF ( 1 ) = n 7 8 9 10 11 12 13 14 15 16 17 18 19 20 21 22 23 24 25 26 27 28 29 30 31

hexane , xF ( 2 ) = nh e p t a n e , and xF ( 3 ) = n o c t a n e P = 1.2; // [ atm ] f = 0.60; xD_hex = 0.99; // [ mole f r a c t i o n ] xB_hex = [0.01]; // [ mole f r a c t i o n ] K (1) = 2.68/ P ; K (2) = 1.21/ P ; K (3) = 0.554/ P ; // S o l u t i o n // The nh e x a n e i s t h e l i g h t key (LK) , t h e nh e p a n e i s t h e heavy key (HK) , and t h e // n o c t a n e i s a heavy nonkey (HNK) // A p l y i n g mass b a l a n c e and a s s u m i n g no n o c t a n e and 0 . 9 9 mole f r a c t i o n nh e x a n e i n t h e // d i s t i l l a t e . // B a s i s : F = 100; // [ mol / h ] //B+D = 1 0 0 ; // For hexane , //F xF = D xD+B xB // from t h e a b o v e two e q u a i t o n A_BD = [1 ,1; xD_hex xB_hex ]; B_BD = [ F ; F * xF (1) ]; //A BD x BD = B BD x_BD = inv ( A_BD ) * B_BD ; D = x_BD (1) ; B = x_BD (2) ; xD = [0.99 ,0.01 ,0.0] ; 88

32 xB = [0.01 ,0.544 ,0.446] ; 33 comp_D = xD .* D ; 34 comp_B = xB .* B ; 35 36 disp ( mol / h , comp_D (3) , n o c t a n e , mol / h , comp_D (2) ,

n h e p t a n e , mol / h , comp_D (1) , nh e x a n e , The c o m p o s i t i o n o f the overhead product i s ); 37 disp ( mol / h , comp_B (3) , n o c t a n e , mol / h , comp_B (2) , n h e p t a n e , mol / h , comp_B (1) , nh e x a n e , The c o m p o s i t i o n o f t h e bottom p r o d u c t i s ) ;
38 39 40 41 42 43 44

//To f i n d o u t minimum number o f p l a t e s , u s i n g Eq . ( 1 9 . 1 3 ) [ Fenske Equation ] // u s i n g r e l a t i v e v o l a t i v i t y o f t h e l i g h t key t o t h e heavy key , which i s t h e // r a t i o o f t h e K f a c t o r s . The K v a l u e s a t t h e f l a s h t e m p e r a t u e a r e t a k e n from Example 1 9 . 2 alpha_LK_HK = K (1) / K (2) ; Nmin = log (( xD (1) / xD (2) ) /( xB (1) / xB (2) ) ) / log ( alpha_LK_HK ) -1; disp ( p l u s a r e b o i l e r , Nmin , The minimum number o f i d e a l s t a g e s i s ); Example 19.4 Example 19.4.sce

1 clear all ; 2 clc ; 3 4 // Example 1 9 . 4 5 // Given 6 // x ( 1 ) = np e n t a n e , x ( 2 ) = nhexane , x ( 3 ) = n 7 8 9 10 11 12

h e p t a n e and x ( 4 ) = n o c t a n e // xF = f e e d , xD = d i s t i l l a t e and xB = bottom xF = [4 40 50 6] ./100 // [ mole f r a c t i o n ] P = 1; // [ atm ] xD1 (2) = 0.98; xD1 (3) = 0.01;

89

13 14 15

16 17 18 19 20 21 22 23 24 25 26 27 28 29 // ( a ) 30 // The b u b b l e p o i n t i s 80 C , and a t t h i s t e m p e r a t u r e 31 alphaLK_HK = K_80 ./ K_80 (3) ; 32 // For an a p p r o x i m a t e s o l u t i o n , u s i n g Eq . ( 1 9 . 1 5 ) 33 RDm = ( F / D ) *((( D * xD (2) /( F * xF (2) ) ) - alphaLK_HK (2) *( D *

// S o l u t i o n // The k e y s a r e nh e x a n e and nh e p t a n e , and t h e o t h e r c o m p o n e n ts a r e // s u f f i c i e n t l y d i f f e r e n t i n v o l a t i l i t y t o be distributed . // B a s i s : F = 100; // [ mol ] xD1 (1) = 1; xD1 (4) = 0; D = sum ( F * xF .* xD1 ) ; // [ mol ] xD = ( F * xF .* xD1 ) ./( D ) B = F - D ; // [ mol ] xB = ( F * xF - D * xD ) / B ; K_80 = [3.62 ,1.39 ,0.56 ,0.23] ; K_81 = [3.72 ,1.43 ,0.58] ; K_81_2 = [3.74 ,1.44 ,0.584] ; KxF = [0.145 ,0.556 ,0.280 ,0.014] ;

xD (3) /( F * xF (3) ) ) ) /( alphaLK_HK (2) -1) )


34 35 36 37 38 39 40 41 42 43 44 45

//To u s e Underwood method , t h e K v a l u e s a t 80 C a r e converted to r e l a t i v e // v o l a t i l i t i e s and t h e r o o t o f Eq . ( 1 9 . 2 9 ) b e t w e e n 1 and 2 . 4 8 i s f o u n d by t r i a l . // S i n c e q = 1 . 0 , t h e t e r m s must sum t o z e r o . phi = 1.5 f = 0; err = 1; while ( err >0.1) fnew = sum ((( alphaLK_HK .* xD ) ./( alphaLK_HK - phi ) ) ) ; err = abs (f - fnew ) ; if (f > fnew ) phi = phi +0.01; 90

46 else 47 phi = phi -0.01; 48 end 49 f = fnew ; 50 end 51 RDm = f -1; 52 53 // ( b ) 54 //To g e t t h e c o n d i t i o n s 55 56 57 58 59 60 61 62 63 64 65 66 67 68 69 70 71 72 73 74 75

i n t h e u p p e r i n v a r i a n t zone , u s i n g Eq . ( 1 9 . 2 4 ) w i t h VbyD = RDm +1; DbyV = inv ( VbyD ) ; VbyF = VbyD * D / F ; LbyV = RDm /( RDm +1) ; y_80 = DbyV * xD (1:3) ./(1 - LbyV ./ K_80 (1:3) ) y_81_1 = [0.046 ,0.637 ,0.317] ; x_81_1 = y_81_1 ./ K_81 ; // The v a p o r c o m p o s i t i o n f o r l o w e r i n a v a r i a n t z o n e i s // u s i n g Eq . ( 1 9 . 2 8 ) , f o r q = 1 . 0 BbyVb = 0.552; LbbyVb = 1.55; K_83 = [1.52 ,0.618 ,0.258] ; y_83 = BbyVb * xB (2:4) ./( LbbyVb ./ K_83 -1) ; y_83_3 = [0.662 ,0.326 ,0.012] ; x_83_3 = y_83_3 ./ K_83 ;

disp ( r e s p e c t i v e l y , C ,81.1 , C ,83.3 , The t e m p e a t u r e i n Lower z o n e and Upper z o n e i s ) disp ( r e s p e c t i v e l y , y_83_3 (1) , y = , x_83_3 (1) , x , The LK c o m p o s i t i o n i n Lower z o n e i s ) disp ( r e s p e c t i v e l y , y_83_3 (2) , y = , x_83_3 (2) , x The HK c o m p o s i t i o n i n Lower z o n e i s ) disp ( r e s p e c t i v e l y , y_81_1 (2) , y = , x_81_1 (2) , x The LK c o m p o s i t i o n i n Upper z o n e i s ) disp ( r e s p e c t i v e l y , y_81_1 (3) , y = , x_81_1 (3) , x The HK c o m p o s i t i o n i n Upper z o n e i s ) Example 19.5 Example 19.5.sce 91

= =, =, =,

1 2 3 4 5 6 7 8 9 10 11 12 13 14 15 16 17 18 19 20 21 22 23 24 25 26 27 28 29 30 31 32 33 34 35

clear all ; clc ; // Example 1 9 . 5 // Given Nmin = 9.4+1; // From Example 1 9 . 3 xF = [0.33 ,0.37 ,0.30] ; xD = [0.99 ,0.01 ,0] ; K = [2.23 ,1.01 ,0.462] ; alpha = [2.21 ,1.0 ,0.457] ; // For a l i q u i d f e e d q = 1; phi = 1.45; f = 0; err = 1; while ( err >0.1) fnew = sum ((( alpha .* xD ) ./( alpha - phi ) ) ) ; err = abs (f - fnew ) ; if (f > fnew ) phi = phi +0.01; else phi = phi -0.01; end f = fnew ; end RDm = f -1; RD = RDm *1.5; //A = (RDRDm) /RD+1 // from F i g . 1 9 . 5 N = ( Nmin +0.41) /(1 -0.41) ; disp (N , The number o f i d e a l p l a t e r e q u i r e d a r e )

92

Chapter 20 Leaching and Extraction


20.1 Scilab Code

Example 20.1 Example 20.1.sce


1 clear all ; 2 clc ; 3 4 // Example 20 1 5 // Given 6 Fin = 2*10^3; // [ kg / day ] 7 //w ( 1 ) = p a r a f f i n wax , w ( 2 ) = p a p e r p u l p 8 wi = [0.25 ,0.75] ; // [ w i e g h t p e r c e n t ] 9 10 // S o l u t i o n 11 // U s i n g c o n v e n i e n t u n i t s i n Eq . ( 1 7 . 2 4 ) 12 // As t h e r a t i o o f k e r o s e n e t o p u l p i s c o n s t a n t , 13 14 15 16 17

flow

r a t e s s h o u l d be // e x p r e s s e d i n pounds o f k e r o s e n e . Then , a l l t h e c o n c e n t r a t i o n s must // be i n pound o f wax f r e e k e r o s e n e . The u n e x t r a c t e d p a p e r had no k e r o s e n e // s o t h e f i r s t c e l l must be t r e a t e d s e p a r a t e l y . // R e f e r i n g t o t h e F i g . 2 0 . 3 // B a s i s :

93

18 F = 100; // [ l b wax + k e r o s e n e f r e e p u l p ] 19 //By making a mass b a l a n c e o v e r wax 20 // w a x i n = F ( wi ( 1 ) / wi ( 2 ) )+ 0 . 0 0 0 5 s ( s i s 21 22 23 24 25 26 27 28 29 30 31 32 33 34 35 36 37 38 39 40 41 42 43 44

t h e wax

input with s o l v e n t ) // w a x o u t = F ( 0 . 0 0 2 ) +( s 200) 0 . 0 5 // by w a x i n = w a x o u t s_in = (33.33+9.8) /(0.05 -0.0005) ; // [ l b ] // The c o n c e n t r a t i o n o f t h i s s t r e a m i s , t h e r e f o r e s_out = 200; // [ l b ] s_stsol = s_in - s_out ; // [ l b ] wax_sol = s_stsol *0.05; // [ l b ] // The c o n c e n t r a t i o n i n t h e u n d e r f l o w t o t h e s e c o n d unit equals that // o f t h e o v e r f l o w from t h e f i r s t s t a g e , o r 0 . 0 5 l b o f wax p e r pound // o f k e r o s e n e . The wax i n t h e u n d e r f l o w t o u n i t 2 i s wax_uflow_2 = s_out *0.05; // [ l b ] wax_oflow_21 = wax_uflow_2 + wax_sol - F *( wi (1) / wi (2) ) // [ l b ] // The c o n c e n t r a t i o n s o f t h i s s t r e a m i s , t h e r e f o r e , ya = wax_oflow_21 /871; yastar = 0.05; xa = yastar ; ybstar = 0.2/ s_out ; xb = ybstar ; yb = 0.0005;

// S i n c e 1 s t a g e h a s a l r e a d y ben t a k e n i n t o a c c o u n t , // Eq . ( 1 7 . 2 4 ) , w i l l g i v e N1 s t a g e s , Hence N = log (( yb - ybstar ) /( ya - yastar ) ) / log (( yb - ya ) /( ybstar - yastar ) ) ; 45 disp ( N +1 , The t o t a l number o f i d e a l s t a g e s i s ) ; Example 20.2 Example 20.2.sce
1 clear all ; 2 clc ;

94

3 4 // Example 2 0 . 2 5 // Given 6 F = 1000; // [ kg ] 7 solv_O = 10; // [ kg ] 8 solv_B = 655; // [ kg ] 9 w_out = 60; // [ kg ] 10 // S o l u t i o n 11 // L e t s o l u t i o n r e t a i n e d 12 SR =

i s SR , from T a b l e 2 0 . 1

[0.5 ,0.505 ,0.515 ,0.530 ,0.550 ,0.571 ,0.595 ,0.620] ; 13 xb = 0:0.1:0.7; 14 // L e t x and y be t h e mass f r a c t i o n o f o i l i n t h e u n d e r f l o w and 15 // o v e r f l o w s o l u t i o n s .
16 17 18 19 20 21 22 23 24 25 26 27 28 29

30 31 32 33 34 35 36 // At t h e s o l i d i n l e 37 La = 400+25; // [ kg s o l u t i o n s / h ]

// At t h e s o l v e n t i n l e t , Vb = solv_O + solv_B ; // [ kg s o l u t i o n / h ] yb = solv_O / Vb ; err = 1; i = 1; sr = SR (2) ; xb1 = 0.0; while ( err >0.001) Lb = sr * F ; xbnew = w_out / Lb ; err = abs ( xb1 - xbnew ) ; xb1 = xbnew ; sr = SR ( i ) +( xb1 - xb ( i ) ) /( xb ( i +1) - xb ( i ) ) *( SR ( i +1) - SR (i)); i = i +1; end Lb = sr * F ; // Benzene i n t h e u n d e r f l o w a t Lb i s Underlow_B = Lb - w_out ; // [ kg s o l u t i o n s / h ]

95

38 xa = 400/ La ; 39 w_in = 10+400; // [ kg /h ] 40 Extract_O = w_in - w_out ; // [ kg /h ] 41 Extract_B = 655+25 -447; // [ kg /h ] 42 Va = Extract_O + Extract_B ; // [ kg /h ] 43 ya = Extract_O / Va ; 44 45 // The a n s w e r s t o p a r t s ( a ) t o ( d ) a r e 46 // ( a ) 47 disp ( ya , The c o n c e n t r a t i o n o f s t r o n g s o l u t i o n i s ) ; 48 // ( b ) 49 disp ( xb1 , The c o n c e n t r a t i o n o f t h e s o u l t i o n a d h e r i n g 50 51 52 53 54 55 56 57 58 59 60 61 62 63 64 65 66 67 68 69 70 71 72

to the e x t r a c t e d s o l i d s i s ); // ( c ) disp ( kg /h ,Lb , The mass o f s o l u t i o n l e a v i n g w i t h t h e e x t r a c t e d meal i s ) ; // ( d ) disp ( kg /h ,Va , The mass o f e x t r a c t i s ) ; // ( e ) //To d e t e r m i n e an i n t e r m e d i a t e p o i n t on t h e operating line , choosing , xn = 0.5; // S o u l i o n r e t a i n e d Ln = 0.571* F ; // [ kg /h ] //By o v e r a l l b a l a n c e , Eq . ( 2 0 . 1 ) V_n_1 = Va + Ln - La ; // [ kg /h ] //By o i l b a l a n c e y_n_1 = ( Ln * xn + Va * ya - La * xa ) / V_n_1 ; y =0:0.1:1; x = y; plot (x ,y ,[ xb1 , xn , xa ] ,[ yb , y_n_1 , ya ]) xgrid () xlabel ( x ) ylabel ( y ) title ( F i g u r e 2 0 . 4 ) legend ( y=x , o p e r a t i n g l i n e ) // U s i n g F i g u r e 2 0 . 4 , number o f i d e a l s t a g e s r e q u i r e d 96

are 73 N = 4; 74 disp (N , Number o f s t a g e s r e q u i r e d a r e )

Figure 20.1: Diagram for Example 20.2 Example 20.3 Example 20.3.sce
1 clear all ; 2 clc ; 3 4 // Example 2 0 . 3

97

5 // Given 6 T = 25; // [ C ] 7 // x ( 1 ) = Acetone , x ( 2 )= w a t e r and x ( 3 )= MIK 8 //F = f e e d 9 xF = [0.40 , 0.60 ,0.0] ; 10 xMIK_i = [0.0 ,0.0 ,1.0] ; 11 12 // S o l u t i o n 13 // U s i n g d a t a from F i g . 2 0 . 1 0 , t o p l o t e q u i l i b r i u m 14 15 16 17 18 19 20 21 22 23 24 25 26 27 28 29 30 31 32 33 34 35 36 37 38 39

curve // F i g . 2 0 . 1 3 . // B a s i s : F = 100; // [ mass u n i t s / h ] // L e t n = mass f l o w r a t e o f H2O i n e x t a r c t //m = mass f l o w r a t e o f MIK i n r a f f i n a t e // For 99 p e r c e n t r e c o v e r y o f A, t h e e x t a r c t h a s E_A = 0.99* xF (1) * F ; //And t h e R a f f i n a t e h a s R_A = xF (1) *F - E_A ; // The t o t a l f l o w s a r e // At t h e top , // La = F = 40 A+60 H2O //Va = 3 9 . 6 A+n H20+(100 m) MIK = 1 3 9 . 6 + nm // At t h e bottom , Vb = 100; // MIK // Lb = 0 . 4 A+(60 n ) H2O+mMIK = 6 0 . 4 +m n // S i n c e n and m a r e s m a l l and t e n d t o c a n c e l i n t h e summatios f o r Va and La , // t h e t o t a l e x t r a c t f l o w Va i s a b o u t 1 4 0 , which would make yA_a = 39.6/140; xA = 0.4/60; // From F i g 2 0 . 1 0 , f o r yA = 0.283 , yH2O = 0.049 xA = 0.007 , xMIK = 0.02 nm = [6;2]; err = 1; while ( err >0.1) 98

40 41 42 43 44 45 46 47 48 49 50 51 52 53 54 55 56 57 58 59 60 61 62 63 64 65 66 67 68 69 70 71 72

nmold = nm ; nm (1) = yH2O /(1 - yH2O ) *(39.6+100 - nm (2) ) ; nm (2) = xMIK /(1 - xMIK ) *(0.4+60 - nm (1) ) ; err = norm ( nm - nmold ) ; end n = nm (1) ; m = nm (2) ; Va = 139.6+ n - m ; yA_a = 39.6/ Va ; Lb = 60.4+ m - n ; xA_b = 0.4/ Lb ; // For an i n t e r m e d i a t e p o i n t on t h e o p e r a t i n g l i n e , picking yA = 0.12; // From F i g . 2 0 . 1 0 , yH2O = 0.03; yMIK = 0.85; // S i n c e t h e r a f f i n a t e p h a s e h a s o n l y 2 t o 3 p e c e n t MIK , a s s u m i n g // t h a t t h e amount o f MIK i n t h e e x t r a c t i s 1 0 0 , t h e same a s t h e s o l v e n t // f e d : V = 100/ yMIK ; //By an o v e r a l l b a l a n c e from t h e s o l v e n t i n l e t ( bottom ) t o t h e i n t e r m e d i a t e // p o i n t , xb = xA_b ; L = Lb +V - Vb ; yb = 0; //A b a l a n c e on A o v e r t h e same s e c t i o n g i v e s xA ; xA = (0.4+117.6*0.12 -0) / L ; // For xA and xMIK = 0 . 0 3 , A b a l a n c e on MIK from t h e solvent // i n l e t t o t h e i n t e r m e d i a t e p o i n t g i v e s V_revised = 101.1/0.85; L_revised = 54.4+118.9 -100; xA_revised = (0.4+118.9*0.12) /73.3; 99

73 y =0:0.1:1; 74 x = y ; 75 plot (x ,y ,[0.00074 ,0.2 ,0.4 ,] ,[0 ,0.12 ,0.272 ,]) 76 xgrid () 77 xlabel ( x ) 78 ylabel ( y ) 79 title ( F i g u r e 2 0 . 1 3 ) 80 legend ( y=x , o p e r a t i n g l i n e ) 81 82 // From F i g . 2 0 . 1 3 83 disp (3.4 , Number o f s t a g e s )

Figure 20.2: Diagram for Example 20.3 100

101

Chapter 21 Principles of Diusion and Mass Transer between Phases


21.1 Scilab Code

Example 21.1 Example 21.1.sce


1 2 3 4 5 6 7 8 9 10 11 12 13 14 15 16 17 18

clear all ; clc ; // Exapmle 2 1 . 1 // Given yA = 0.20; yAi = 0.10; // S o l u t i o n // ( a ) // L e t A = Dv rho M /BT A = 1; // assumed // U s i n g Eq . ( 2 1 . 1 9 ) , f o r e u i l m o l a l d i f f u s i o n , JA = A *( yA - yAi ) ; // Form Eq . ( 2 1 . 2 4 ) , f o r one way d i f f u s i o n , NA = A * log ((1 - yAi ) /(1 - yA ) ) ; NAbyJA = NA / JA ;

102

19

disp ( I n t h i s c a s e t h e t r a n s f e r r a t e w i t h one way d i f f u s i o n i s , NAbyJA -1 , p e r c e n t g r e a t e r t h a n t h a t with e q u i m o l a l d i f f u s i o n );

20 21 // ( b ) 22 //Whwn, b = BT/2 23 A = A *2; 24 yA = 1 - exp ( NA /2) *(1 - yA ) 25 disp ( yA , The v a l u e o f yA h a l f w a y t h r o u g h t h e l a y e r

f o r one way d i f f u s i o n Example 21.2 Example 21.2.sce


1 2 3 4 5 6 7 8 9 10 11 12 13 14 15 16 17 18 19 20 21

i s );

clear all ; clc ; // Example 2 1 . 2 // Given K = 273.16 T = 100+ K ; // [ K ] P = 10; // [ atm ] // From T a b l e 2 1 . 1 TcA = 198+ K ; // [ K ] TcB = -147+ K ; // [ K ] rho_cA = 0.552; // [ g /cm 3 ] rho_cB = 0.311; // [ g /cm 3 ] MA = 137.5; MB = 28;

// S o l u t i o n VcA = MA / rho_cA // [ cm3/ g mol ] VcB = MB / rho_cB // [ cm3/ g mol ] // S u b s t i t u i n g i n Eq . ( 2 1 . 2 5 ) Dv = (0.01498* T ^1.81*(1/ MA +1/ MB ) ^0.5) /( P *( TcA * TcB ) ^0.1405*( VcA ^0.4+ VcB ^0.4) ^2) ; // [ cm2/ s ] 22 disp ( cm2/ s ,Dv , V o l u m e t r i c D i f f u s i v i t y ( Dv ) = ) Example 21.3 Example 21.3.sce 103

1 2 3 4 5 6 7 8 9 10 11 12 13 14 15 16 17 18 19 20 21 22 23 24

clear all ; clc ; // Example 2 1 . 3 // Given // 1 = b e n z e n e and 2 = t o l u e n e M1 = 78.11; M2 = 92.13; T1_bp = 80.1+273; // [ K ] T2_bp = 110.6+273; // [ K ] VA1 = 96.5; // [ cm3/ mol ] VA2 = 118.3; // [ cm3/ mol ] mu1 = 0.24; // [ cP ] mu2 = 0.26; // [ cP ] T = 110+273; // [ K ] // S o l u t i o n // From Eq . ( 2 1 . 2 6 ) // For b e n z e n e i n t o u l e n e , Dv1 = 7.4*10^ -8*( M2 ) ^0.5* T /( mu2 * VA1 ^0.6) ; // [ cm2/ s ] // For t o l u e n e i n b e n z e n e , Dv2 = 7.4*10^ -8*( M1 ) ^0.5* T /( mu1 * VA2 ^0.6) ; // [ cm2/ s ]

disp ( cm2/ s ,Dv1 , D i f f u s i v i t y o f b e n z e n e i n t o l u e n e i s ); 25 disp ( cm2/ s ,Dv2 , D i f f u s i v i t y o f t o l u e n e i n b e n z e n e i s ); Example 21.4 Example 21.4.sce
1 2 3 4 5 6 7 8

clear all ; clc ; // Example 2 1 . 4 // Given Nre = 20000; T = 40; // [ C ] D = 2; // [ i n . ] 104

9 10 11 12 13 14 15 16 17 18 19 20 21 22

Dv1 = 0.288; // [ cm2/ s ] , f o r water a i r Dv2 = 0.145; // [ cm2/ s ] , f o r e t h a n o l a i r // S o l u t i o n // For a i r a t 40 C rho = 29/22410*273.16/313.16; // [ g /cm 3 ] mu = 0.0186; // [ cP ] , from Appendix 8 mubyrho = mu *10^ -2/ rho ; // [ cm2/ s ]

// ( a ) // For t h e a i r w a t e r system , Nsc = mubyrho / Dv1 ; // Form Eq . ( 2 1 . 5 4 ) Nsh = 0.023*( Nre /2) ^0.81* Nsc ^0.44; // I n t h e f i l m t h e o r y kc = D/BT and s i n c e Nsh = kc D/ Dv 23 BT1 = D / Nsh ; // [ i n . ] 24 disp ( i n . ,BT1 , E f f e c t i v e t h i c k n e s s o f t h e g a s f i l m is )
25 26 // ( b ) 27 // For t h e s y s t e m a i r e t h a n o l , 28 Nsc = mubyrho / Dv2 ; 29 Nsh = 0.023*( Nre /2) ^0.81* Nsc ^0.44; 30 BT2 = D / Nsh ; // [ i n . ] 31 disp ( i n . ,BT2 , E f f e c t i v e t h i c k n e s s

of the gas f i l m

is ) Example 21.5 Example 21.5.sce


1 2 3 4 5 6 7 8 9

clear all ; clc ; // Example 2 1 . 5 // Given T = 110; // [ C ] P = 1; // [ atm ] mu = 0.26; // [ cP ] 105

10 Dvx = 6.74*10^ -5; // [ cm2/ s ] 11 rho_mx = 8.47; // [ mol /L ] 12 Dvy = 0.0494; // [ cm2/ s ] 13 rho_my = 0.0318; // [ mol /L ] 14 15 // ( a ) 16 // U s i n g Eq . ( 2 1 . 7 8 ) 17 kybykx = ( Dvy / Dvx ) ^0.5*( rho_my / rho_mx ) ; 18 // The gas f i l m c o e f f i c i e n t p r e d i c t e d i s o n l y 10

percent 19 // and i f m=1 , 90 p e r c e n t o f t h e o v e r a l l r e s i s t a n c e t o mass 20 // t r a n s f e r would be i n t h e g a s f i l m . 21 disp ( kybykx *100 , f r a c t i o n o f t h e o v e r a l l r e s i s t a n c e i n the gas phase i s );
22 23 24 25 26 27 28 29 30 31 32 33 34 35 36 37 38 39 40 41

// ( b ) // Assuming t h e column i s o p e r a t e d a t t h e same f a c t o r F // Gas f i l m : rho_myprime = 0.00894; // [ mol /L ] Dvyprime = (341/383) ^1.81*( Dvy /0.25) ; deltakyprime = sqrt ( Dvyprime / Dvy ) * rho_myprime / rho_my ; // L i q u i d f i l m : rho_mxprime = 8.93; // [ mol /L ] muprime = 0.35; // [ cP ] Dvxprime =(341/383) *0.26* Dvx / muprime ; deltakxprime = sqrt ( Dvxprime / Dvx ) *( rho_mxprime / rho_mx ) ; // k y p r i m e = d e l t a k y p r i m e ky ; // k x p r i m e = d e l t a k x p r i m e / 0 . 1 0 2 ky ; // At 1 atm and ky = 0 . 1 0 2 kx and Ky = 0 . 9 0 7 / ky // Kyprime = 0 . 4 7 6 ky // For o v e r a l l t r a n s f e r u n i t s NOy = 2*0.476/0.53; neta = 1 - exp ( - NOy ) ; disp ( neta , The e f f i c i e n y w i l l be ) 106

Example 21.6 Example 21.6.sce


1 2 3 4 5 6 7 8 9 10 11 12 13 14 15

clear all ; clc ;

// Example 2 1 . 6 // Given Dvprime = 10^ -7; // [ cm2/ s ] rp = 0.04/2; // [ cm ] t = 30*60; // [ s ] // Then , beeta = Dvprime * t / rp ^2; // form F i g . 1 0 . 6 phi = 0.26; // Murphree e f f i c i e n c y neta_M = 1 - phi ; // Here t h e a v e r a g e e f f i c i e n y i s n e a r l y e q u a l t o t h e Murphree e f f i c i e n c y . 16 disp (4/ neta_M , The a c t u a l number o f s t a g e s i s )

107

Chapter 22 Gas Absorption


22.1 Scilab Code

Example 22.1 Example 22.1.sce


1 2 3 4 5 6 7 8 9 10 11 12 13 14 15 16 17 18 19 20

clear all ; clc ; // Example 2 2 . 1 // Given Dp = 1; // [ i n . ] vdot = 25000; // [ f t 3/ h ] T = 68; // [ F ] P = 1; // [ atm ] ya = 0.02; Mair = 29; Mg = 17; // S o l u t i o n // The a v e r a g e m o l e c u l a r w e i h t o f t h e e n t e r i n g g a s M = (1 - ya ) * Mair + ya * Mg ; rho_y = M *492/(359*(460+68) ) ; // [ l b / f t 3 ] // ( a ) // U s i n g F i g . 2 2 . 5 , when Gy =Gx ; Gy = 0.472; // [ l b / f t 2 s ]

108

21 22 23 24 25 26 27 28 29 30 31 32 33 34 35 36 37 38

Gx = Gy ; // [ l b / f t 2 h ] des_value = Gy /2; // [ l b / f t 2 h ] mdot = vdot * rho_y /3600; // [ l b / s ] // C r o s s s e c t i o n a l a r e a o f t h e t o w e r S = mdot / des_value // [ f t 2 ] // t h e d i a m e t e r o f t h e t o w e r i s Dtower = sqrt (4* S / %pi ) ; // [ f t ] disp ( f t , Dtower , The t o w e r d i a m e t e r i s ) ; // ( b ) h = 20; // [ f t ] // U s i n g F i g 2 2 . 4 , t h e p r e s s u r e d r o p f o r Gy = 850; // [ l b / f 2 h ] Gx = Gy ; delta_P = 0.35; // [ i n . ] (H2O/ f t ) // The t o t a l p r e s s u r e d r o p Pt = delta_P * h ; // [ i n . ] H2O disp ( i n . H2O ,Pt , The p r e s s u r e d r o p would be ) ; Example 22.2 Example 22.2.sce

1 2 3 4 5 6 7 8 9 10 11 12 13 14 15 16 17

clear all ; clc ; // Example 2 2 . 2 // Given Dp = 1; // [ i n . ] vdot = 25000; // [ f t 3/ h ] T = 68; // [ F ] P = 1; // [ atm ] ya = 0.02; Mair = 29; Mg = 17; // S o l u t i o n // The a v e r a g e m o l e c u l a r w e i h t o f t h e e n t e r i n g g a s M = (1 - ya ) * Mair + ya * Mg ; rho_y = M *492/(359*(460+68) ) ; // [ l b / f t 3 ] rho_x = 62.3; // [ l b / f t 3 ] 109

18 19 20 21 22 23 24 25 26 27 28 29 30 31 32 33 34 35 36 37 38 39

// ( a ) // U s i n g F i g . ( 2 2 . 8 ) , from Example 2 2 . 1 A = Gx/Gy = 1 and // L e t A = 1; B = A * sqrt ( rho_y / rho_x ) ; // Form F i g 2 2 . 8 , t h e s u p e r f i c i a l v a p o r v e l o c i t y a t flooding // i s u o f s q r t ( r h o y / ( r h o x r h o y ) ) = 0 . 1 1 , t h e r e f o r e uof = 0.11/ sqrt ( rho_y /( rho_x - rho_y ) ) ; // [m/ s ] // The a l l o w a b l e v a p o r v e l o c i t y uo = uof *0.5; // [m/ s ] uo = uo *3.28; // [ f t / s ] // t h e c o r r e s p o n d i n g mass v e l o c i t y Gy = uo * rho_y ; // [ l b / f t 2 s ] // The a l l o w a b l e mass v e l o c i t y i n t h e e x a m p l e was 0 . 2 3 6 l b / f t 2 s . // The i n c r e a s e by u s i n g s t r u c t u r e d p a c k i n g i s increase = ( Gy /0.236) -1; disp ( increase *100 , The p e r c e n t i n c r e a s e i n mass v e l o c i t y i s );

// ( b ) // The p r e s s u r e d r o p delta_P = 20*1.22*(0.5/0.9) ^1.8; // [ i n . H2O ] // T h i s i s 1 . 2 t i m e s t h e p r e s s u r e d r o p o f 7 i n . H2O i n the I n to la x saddles . 40 disp ( The p r e s s u r e d r o p w i l l be g r e a t e r t h a n I n t o l a x Saddles ) Example 22.3 Example 22.3.sce
1 clear all ; 2 clc ; 3 4 // Example 2 2 . 3 5 // Given 6 vdot = 4500; // [ SCFM ]

110

7 8 9 10 11 12 13 14 15 16 17 18 19 20 21 22 23 24 25 26 27 28 29 30 31 32 33 34 35 36 37 38 39 40 41 42 43

yin = 0.06; yout = 0.0002; P = 1; // [ atm ] Tiy = 20; // [ C ] Tix = 25; // [ C ] // S o l u t i o n // From P e r r y x = [0.0308 ,0.0406 ,0.0503 ,0.0735] ; y20 = [0.0239 ,0.0328 ,0.0417 ,0.0658] ; y30 = [0.0389 ,0.0528 ,0.0671 ,0.1049] ; y40 = [0.0592 ,0.080 ,0.1007 ,0.1579] ; deltaH = -8.31*10^3; // [ c a l / g mol ] , f r o NH3=NH3( aq ) // B a s i s : gas_in = 100; // [ g mol d r y ] air_in = (1 - yin ) * gas_in ; // [ mol ] NH3_in = yin * gas_in ; // [ mol ] H2O_in = 2.4; // [ mol ] air_out = air_in ; // [ mol ] // The m o l e s o f NH3 i n t h e o u t l e t gas , NH3_out = air_out *( yout /(1 - yout ) ) ; // [ mol NH3 ] // The amount o f NH3 a b s o r b e d NH3_abs = NH3_in - NH3_out ; // [ mol ] // Heat E f f e c t s : // The h e a t o f a b s o r p t i o n Qa = - NH3_abs * deltaH ; // [ c a l ] // S e n s i b l e h e a t c h a n g e s i n t h e g a s a r e Qair = air_in *7*5; // [ c a l ] QH2O = H2O_in *8*5; // [ c a l ] Qsy = 3290+96; // [ c a l ] // The amount o f v a p o r i z a t i o n o f w a t e r from t h e liquid pH2O_20 = 17.5; // [mm Hg ] , a t 20C pH2O_25 = 23.7; // [mm Hg ] , a t 25C H2O_inlet = gas_in *( pH2O_20 /742.5) ; // [ mol ] H2O_outlet = 94.02*( pH2O_25 /736.3) ; // [ mol ] // The amount o f w a t e r v a p o r i z e d H2O_vaporized = H2O_outlet - H2O_inlet ; // [ mol ] 111

44 45 46 47 48 49 50 51 52 53 54 55 56 57 58 59 60 61 62 63 64 65 66 67 68 69 70 71 72 73 74 75 76 77 78

deltaHv = 583; // [ c a l / g ] Qv = deltaHv * H2O_vaporized *18.02; // [ c a l ] // From Eq . ( 2 2 . 3 1 ) Qsx = Qa -( Qv + Qsy ) ; // [ c a l ] Cp = 18; // [ c a l / gmolC ] xmax = 0.031; Tb = 40; // [ C ] Ta = 25; // [ C ] err =1; while ( err >0.01) Lb = NH3_abs / xmax ; Tbnew = Qsx /( Lb * Cp ) + Ta ; err = Tb - Tbnew ; Tb = Tbnew ; xmax = xmax +0.002; end Lmin = Lb - NH3_in ; // [ mol H2O ] La = 1.25* Lmin ; // [ mol ] Lb = La + NH3_in ; // [ mol ] // The t e m p e r a t u r e r i s e o f t h e l i q u u i d i s Tb = Qsx /( Lb * Cp ) + Ta ; // [ C ] xb = NH3_in / La ; // [ C ] ystar = 0.044; // Assuming t e m p e r a t u r e t o be l i n e a r f u n c t i o n o f x , so T = 30; // x = 0 . 0 1 3 7 ; // U s i n g t h e d a t a g i v e n f o r 30C and i n t e r p o l a t i n g t o get the // i n i t i a l s l o p e f o r 25 and t h e f i n a l v a l u e y s t a r f o r 35 , the // e u i l i b r i u m l i n e i s drawn y = [0.06 , 0.03 ,0.01 ,0.0002] ; ystar = [0.048 ,0.017 ,0.0055 ,0] ; delta_y = y - ystar ; delta_yL = [0.0125 , 0.0080 ,0.00138] ; delta_NOy = [2.4 ,2.5 ,7.1] ; 112

79 NOy = sum ( delta_NOy ) ; 80 disp ( NOy , The v a l u e o f NOy i s ) ; 81 82 83 84 plot (x , y20 ,x , y30 ,x , y40 ) ; 85 xgrid () ; 86 xlabel ( x ) ; 87 ylabel ( y ) ; 88 legend ( 20C , 30C , 40C ) ; 89 title ( x v s y o f NH3 a t d i f f e r e n t

t e m p e r a t u r e s );

Figure 22.1: Diagram for Example 22.3 113

Example 22.4 Example 22.4.sce


1 2 3 4 5 6 7 8 9 10 11 12 13 14 15 16 17 18 19 20 21 22 23 24 25 26 27 28 29 30

clear all ; clc ; // Example 2 2 . 4 // Given ieee () ; H = 0.0075; // [ TCE ] T = 20; // [ C ] P = 1; // [ atm ] wa = 6*10^ -6; // [ g ] Ca = 6; // [ ppm ] wb = 4.5*10^ -9 // [ g ] M = 18; // S o l u t i o n m = H / P *10^6/ M ; // With t h i s l a r g e v a l u e o f m, t h e d e s o r p t i o n i s l i q u i d p h a s e c o n t r o l l e d . // At t h e minimum a i r r a t e , t h e e x i t g a s w i l l be i n e q u i l i b r i u m with the // i n c o m i n g s o l u t i o n . MTCE = 131.4; j = 1.5; for i = 1:7 xa = wa / MTCE * M ; ya = m * xa ; // Per c u b i c m e t e r o f s o l u t i o n f e d , t h e TCE removed is VTCE = 10^6*( wa - wb ) / MTCE ; // [ mol ] // The t o t a l amount o f g a s l e a v i n g i s V = VTCE / ya ; // [ mol ] Fmin = V *0.0224; // [ s t d m 3 ] , a s 1 gmol = 0 . 0 2 2 4 s t d m3 Vmin = Fmin * j ; 114

31 // D e n s i t y a t t h e s t a n d a r d c o n d i t i o n s , 32 rho = 1.259; // [ kg /m 3 ] , 33 // s o t h e minimum r a t e on a mass b a s i s i s , 34 // L e t A = ( Gy/Gx ) min 35 A = Vmin * rho /1000; // [ kg a i r / kg w a t e r ] 36 // I f t h e a i r r a t e i s 1 . 5 t i m e s t h e minimum v a l u e ,

then
37 ya = ya / j ; 38 xastar = ya / m ; 39 Castar = xastar * MTCE / M *10^6; // [ ppm ] 40 delta_Ca = Ca - Castar ; 41 42 // At bottom 43 Cb = 0.0045; // [ ppm ] 44 Cbstar = 0; // [ ppm ] 45 delta_Cb = Cb - Cbstar ; // [ ppm ] 46 delta_CL = ( delta_Ca - delta_Cb ) / log ( delta_Ca / delta_Cb

) ; // [ ppm ] 47 Nox ( i ) = ( Ca - Cb ) / delta_CL ; 48 j = j +0.5; 49 end


50 51 Hox = 3; // [ f t ] 52 Z = Hox * Nox ; // [ f t ] 53 // Going from 1 . 5 t o 2Vmin o r from 2 t o 3Vmin 54 55 56 57 58 59

d e c r e a s e s the tower height // c o n s i d e r a b l y , and t h e r e d u c t i o n i n pumping work f o r w a t e r i s more t h a n // t h e a d d i t i o n a l e n e r g y n e e d e d t o f o r c e a i r t h r o u g h t h e column . F u r t h e r // i n c r e a s e i n V d o e s n o t c h a n g e Z v e r y much , and t h e optimum a i r r a t e i s // p r o b a b l y i n t h e r a n g e 3 t o 5Vmin . / disp ( Nox , Number o f T r a n s f e r u n i t s w i t h minimum a i r rates ) Example 22.5 Example 22.5.sce 115

1 clear all ; 2 clc ; 3 4 // Example 2 2 . 5 5 // S o l u t i o n 6 // E q u l i b r i u m d a t a a r e shown i n F i g . 2 2 . 2 2 7 //By a h e a t b a l a n c e s i m i l a r t o t h a t o f Eample 2 2 . 3 8 // The t e m p e r a t u r e r i s e o f t h e l i q u i was e s t i m a t e d 9 // t o be 10 delta_T = 12.5; // [ C ] 11 // B a s i s : 12 dry_gas_in = 100; // [ mol ] 13 sol_in = 140; // [ mol ] 14 N2_in = 87; // [ mol ] 15 CO2_in = 10; // [ mol ] 16 EO_in = 3; // [ mol ] 17 N2_out = 87; // [ mol ] 18 CO2_out = 10; // [ mol ] 19 EO_out = 3*0.02; // [ mol ] 20 IN = N2_in + CO2_in + EO_in ; // [ mol ] 21 OUT = N2_out + CO2_out + EO_out ; // [ mol ] 22 // Assuming n e g l i g i b l e CO2 a b s o r p t i o n and n e g l e c t 23 24 25 26 27 28 29 30 31 32 33 34 35 36 37

e f f e c t o f H2O on // g a s c o m p o s i t i o n . // At t o p : xt = 0.004; yt = EO_out / OUT ; // M o l e s o f EO a b s o r b e d EO_abs = 3*0.98; // [ mol ] // M o l e s o f EO a b s o r b e d i n w a t e r EO_H2O = 140*0.0004; // [ mol ] // At bottom : xb = ( EO_abs + EO_H2O ) /(140+ EO_abs ) ; yb = 0.03; // From F i g 2 2 . 2 2 y = [0.03 ,0.015 ,0.005 ,0.0006] ; delta_y1 = [0.008 ,0.0006 ,0.0024 ,0.0003] ;

116

38 for i = 1: length ( y ) -1 39 delta_y = y ( i ) -y ( i +1) ; 40 delta_yL = ( delta_y1 ( i ) - delta_y1 ( i +1) ) / log (

delta_y1 ( i ) / delta_y1 ( i +1) ) ;


41 Noy1 ( i ) = delta_y / delta_yL ; 42 end 43 Noy = sum ( Noy1 ) ; 44 45 // Column d i a m e t e r : 46 // U s i n g g e n e r a l i z e p r e s s u r e d r o p c o r r e l a t i o n , F i g 47 48 49 50 51 52 53 54 55 56 57 58 59 60 61 62 63 64 65 66 67 68 69 70 71 72

.22.6 // Based on t h e i n l e t gas , Mbar = 0.87*28+0.1*44+0.03*44; // At 40C , rho_y = 30.1/359*20*273/313 // [ l b / f t 3 ] rho_x = 62.2; // [ l b / f t 3 ] // L e t A = Gx/Gy s q r t ( r h o y / ( r h o x r h o y ) ) A = 1.4*18/(1*30.1) * sqrt ( rho_y /( rho_x - rho_y ) ) ; // From F i g . 2 2 . 6 , f o r delta_P = 0.5; // [ i n . H2O/ f t ] // L e t B = Gy2 Fp mux 0 . 1 / ( r h o y ( r h o x r h o y ) g c ) B = 0.045; // From T a b l e 2 2 . 1 , Fp = 40; mu = 0.656; // [ cP ] // s o Gy = sqrt ( B *( rho_y ) *( rho_x - rho_y ) *32.2/( Fp * mu ^0.1) ) ; // [ l b / f t 2 h ] // o r Gy = Gy *3600; // [ l b / f t 2 s ] Gx = 1.4*18/(1* Mbar ) * Gy ; // [ l b / f 2 s ] // For a f e e d r a t e F = 10000* Mbar ; // [ l b / h ] S = F / Gx ; // [ f t 2 ] D = sqrt ( S *4/ %pi ) ; // [ f t ] // Column h e i g t h : // From F i g . 2 2 . 2 0 a t Gy = 500 and Gx = 1 5 0 0 Hy_NH3 = 1.4; // [ f t ] 117

73 74 75 76 77 78 79 80 81 82 83 84 85 86 87 88 89 90 91 92 93 94 95 96 97 98

mu_40 =0.0181*10^ -2; // [ P ] , Appendix 8 Dv = 7.01*10^ -3; // [ cm2/ s ] , from Eq . ( 2 1 . 2 5 ) rho = 2.34*10^ -2; // [ l b / f t 3 ] Nsc = mu_40 /( rho * Dv ) ; // Form T a b l e 2 2 . 1 , fp = 1.36; Hy_EO = 1.4*(1.1/0.66) ^0.5*1/1.36*( Gy /500) ^0.3*(1500/ Gx ) ^0.4; // [ f t ] // Form F i g . 2 2 . 1 9 , Hx_O2 = 0.9; // [ f t ] Gx1 = 1500; mu1 = 0.00656; // [ P ] rho1 = 1; // [ l b / f t 3 ] // U s i n g Eq . ( 2 1 . 2 8 ) Dv1 = 2.15*10^ -5; // [ cm2/ s ] Nsc1 = mu1 /( rho1 * Dv1 ) ; // U s i n g Eq . ( 2 2 . 3 5 ) , w i t h t h e c o r r e c t i o n f a c t o r f p and Nsc = 3 8 1 , // f o r O2 i n w a t e r a t 25 C Hx_EO = Hx_O2 *( Gx /( mu1 *100) /( Gx1 /0.894) ) ^0.3*( Nsc1 /381) ^0.5/1.36; // [ f t ] // From F i g 2 2 . 2 2 , t h e a v e r a g e v a l u e o f m m = 1.0; // From Eq . ( 2 2 . 3 0 ) HOy = 1.71+(1*0.96) /1.4; // [ f t ] disp ( NOy , number o f t r a n s f e r u n i t s r e q u i r e d ) disp ( f t ,D , d i a m e t e r o f t h e column ) disp ( f t ,HOy , p a c k i n g h e i g h t ) Example 22.6 Example 22.6.sce

1 clear all ; 2 clc ; 3 4 // Example 2 2 . 6 5 // S o l u t i o n 6 rho_m = 62.2/18; // [ mol / f t 3 ]

118

7 8 9 10 11 12 13 14 15 16 17 18 19 20 21 22 23 24 25 26 27 28 29 30 31 32 33 34 35 36 37 38 39 40 41 42 43

// kya = 0 . 0 2 5 Gy 0 . 7 Gx 0 . 2 5 H2ObySO2 = 2*0.98964/0.01036; // and xb = 1/( H2ObySO2 +1) ; // The m o l a l mass v e l o c i t y o f t h e f e e d g a s Gm i s Gm_in = 200/29*(1/0.8) ; // [ mol / f t 2 h ] SO2_in = Gm_in *0.2; // [ mol / f t 2 h ] Air_in = Gm_in *0.8; // [ mol / f t 2 h ] Air_out = Air_in ; // [ mol / f t 2 h ] SO2_out = Air_out *(0.005/(1 -0.005) ) ; // [ mol / f t 2 h ] SO2_abs = SO2_in - SO2_out ; // [ mol / f t 2 h ] H2O_in = H2ObySO2 * SO2_abs ; // [ mol / f t 2 h ] // O p e r a t i n g l i n e x = 0:6; x = x /10^3; A = x ./(1 - x ) ; B = H2O_in / Air_in * A +(0.005/0.995) ; y = B ./( B +1) ; plot (x , y ) xgrid () ; xlabel ( x ) ; ylabel ( y ) ; // l e g e n d ( 2 0 C , 3 0 C , 4 0 C ) ; title ( x v s y ) ; Gxbar = H2O_in *18.02+ SO2_abs *64.1/2; // [ l b / f t 2 h ] kxa = 0.131* Gxbar ^0.82; // [ mol / f t 3 h ] // The g a s f i l m c o e f f i c i e n t s a r e c a l c u l a t e d f o r t h e bottom // and t h e t o p o f t h e t o w e r : // At bottom : Gy_B = ( Air_in *29) +( SO2_in *64.1) ; // [ l b / f t 2 h ] kya_B = 0.025* Gy_B ^0.7* Gx ^0.25; // [ mol / f t 3 h ] // At t o p : Gy_T = ( Air_out *29) +( SO2_out *64.1) ; // [ l b / f t 2 h ] kya_T = 0.025* Gy_T ^0.7* Gx ^0.25; // [ mol / f t 3 h ] // Assuming yLbar = 0.82 C = kxa * yLbar / kya_B ; 119

44 // a l i n e from ( yb , xb ) w i t h a s l o p e o f C , g i v e s 45 yi = 0.164; 46 yLbar = 0.818; 47 m = 20.1 48 Kya_prime = 1/( yLbar / kya_B + m / kxa ) ; // [ mol / f t 3 h ] 49 // The f r a c t i o n o f t h e t o t a l r e s i s t a n c e t h a t i s i n

the l i q u i d i s
50 Rf = m / kxa /(1/ Kya_prime ) ; 51 // For d i f f e r e n t v a l u e s o f y1 52 y1 =[0.2 ,0.15 ,0.1 ,0.05 ,0.02 ,0.005] ; 53 delta_y1 = [0.103 ,0.084 ,0.062 ,0.034 ,0.015 ,0.005] ; 54 y1i = [0.164 ,0.118 ,0.074 ,0.034 ,0.012 ,0.002] ; 55 delta_yi = y1 - y1i ;

120

Figure 22.2: Diagram for Example 22.6

121

Chapter 23 Humidication Operations


23.1 Scilab Code

Example 23.1 Example 23.1.sce


1 2 3 4 5 6 7 8 9 10 11 12 13 14 15 16 17 18 19 20

clear all ; clc ; // Example 2 3 . 1 // Given T = 320; // [ F ] P = 1 ; // [ atm ] // ( 1 )=CO2 , ( 2 )=H2O , ( 3 )=O2 , ( 4 )=N2 y_in = [0.14 ,0.07 ,0.03 ,0.76] ; Tw = 80; // [ F ] // S o l u t i o n // ( a ) // B a s i s F = 100; // [ mol ] , o f g a s Ts = 120; // [ F ] Cps = [9.72 ,8.11 ,7.14 ,6.98] ; n_in = F * y_in ; // [ mol ] nCp = n_in .* Cps ; // sum_nCp = sum ( nCp ) ; sum_n_in = sum ( n_in ) ; // [ mol ]

122

21 22 23 24 25 26 27 28 29 30 31 32 33 34

35 36 37 38 39 40 41 42 43 // ( b ) 44 // f o r Tin = Ts , by h e a t b a l a n c e 45 z = sum_nCp *( T - Ts ) /( lambda_s ) ; 46 n_out (2) = z + n_in (2) ; // [ mole ] 47 // P a r t i a l p r e s s u r e o f t h e w a t e r i n t h e e x i t g a s 48 PH2O = n_out (2) /107.85*760; // [mm Hg ] 49 // T h i s i s h i g h e r t h a n t h e v a p o r p r e s s u r e o f w a t e r a t

Tavg = ( Ts + T ) /2; // [ F ] lambda_s = 1025.8*18; // [ Btu / l b mol ] , a t Ts , from Appendix 7 // Making a h e a t b a l a n c e f o r z m o l e s o f w a t e r evaporated z = sum_nCp *( T - Ts ) /( lambda_s +18*( Ts - Tw ) ) ; // T o t a l m o l e s o f w a t e r i n e x i t g a s n_out (2) = z + n_in (2) ; // [ mole ] // P a r t i a l p r e s s u r e o f t h e w a t e r i n t h e e x i t g a s PH2O = n_out (2) /107.76*760; // [mm Hg ] // But a t 120 F , PH2Oprime = 8 7 . 5 mm Hg ( Appendix 7 ) . Saturation // t e m p e r a t u r e Ts must be g r e a t e r t h a n 120 F . T r y i n g Ts = 126; // [ F ] Tavg = ( Ts + T ) /2; // [ F ] lambda_s = 1022.3*18; // [ Btu / l b mol ] , a t Ts , from Appendix 7 // Making a h e a t b a l a n c e f o r z m o l e s o f w a t e r evaporated z = sum_nCp *( T - Ts ) /( lambda_s +18*( Ts - Tw ) ) ; // T o t a l m o l e s o f w a t e r i n e x i t g a s n_out (2) = z + n_in (2) ; // [ mole ] // P a r t i a l p r e s s u r e o f t h e w a t e r i n t h e e x i t g a s PH2O = n_out (2) /107.76*760; // [mm Hg ] // T h i s i s c l o s e enough t o t h e v a l u e o f PH2Oprime disp ( F ,Ts , A d i a b a t i c s a t u r a t i o n t e m p e r a t u r e ) ;

126 F ,
50 // 1 0 3 . 2 mm Hg , and Ts > 126 F . T r y i n g 51 Ts = 127; // [ F ] 52 Tavg = ( Ts + T ) /2; // [ F ]

123

53 54 55 56 57 58 59 60 61 62

lambda_s = 1021.7*18; // [ Btu / l b mol ] , a t Ts , from Appendix 7 // Making a h e a t b a l a n c e f o r z m o l e s o f w a t e r evaporated z = sum_nCp *( T - Ts ) /( lambda_s ) ; // T o t a l m o l e s o f w a t e r i n e x i t g a s n_out (2) = z + n_in (2) ; // [ mole ] // P a r t i a l p r e s s u r e o f t h e w a t e r i n t h e e x i t g a s PH2O = n_out (2) /107.76*760; // [mm Hg ] // Thus 127 i s t o o h i g h and 126 i s t o o low . Hence , Ts = (126+127) /2; // [ F ] disp ( F ,Ts , A d i a b a t i c s a t u r a t i o n t e m p e r a t u r e ) ; Example 23.3 Example 23.3.sce

1 2 3 4 5 6 7 8 9 10 11 12 13 14 15 16 17 18 19

clear all ; clc ;

// Example 2 3 . 3 // Given Hair_in = 0.022; Tair_inpre = 70; // [ F ] mdot = 15000; // [ l b / h ] // S o l u t i o n // U s i n g F i g . 2 3 . 1 0 Tair_inreh = 85; // [ F ] Tair_outreh = 130; // [ F ] Hin = 0.0030; hya = 85; Ts = 81; // [ F ] Tair_outpre = 168; // [ F ] humid_heat1 = 0.241; // [ Btu / l b F ] // Heat r e q u i r e d t o p r e h e a t t h e a i r i s Qpre = humid_heat1 * mdot *( Tair_outpre - Tair_inpre ) ; // [ Btu / h ] 20 humid_heat2 = 0.250; // [ Btu / l b F ] 21 // Heat r e q u i r e d i n t h e r e h e a t e r i s

124

22 23 24 25 26 27 28 29 30

Qreh = humid_heat2 * mdot *( Tair_outreh - Tair_inreh ) ; // [ Btu / h ] // T o t a l h e a t r e q u i r e d Qt = Qpre + Qreh ; // [ Btu / h ] //To c a l u c u l a t e t h e volume o f t h e s p r q y chamber , Eq . ( 2 3 . 4 1 ) may // be u s e d . The a v e r a g e humid h e a t i s csbar = ( humid_heat1 + humid_heat2 ) /2; // [ Btu / l b d r y a i r F ] // S u b s t i t u i n g i n Eq . ( 2 3 . 4 1 ) g i v e s VT = log (( Tair_outpre - Ts ) /( Tair_inreh - Ts ) ) * mdot * csbar / hya ; // [ f t 3 ] disp ( f t 3 ,VT , The volume o f t h e s p r a y chamber i s )

125

Chapter 24 Drying of Solids


24.1 Scilab Code

Example 24.1 Example 24.1.sce


1 2 3 4 5 6 7 8 9 10 11 12 13 14 15 16 17 18 19 20

clear all ; clc ; // Example 2 4 . 1 // Given Twb = 80; // [ F ] Tdb = 120; // [ F ] v = 3.5; // [ f t / s ] rho = 120; // [ l b / f t 3 ] Xe = 0; Xc = 0.09; lambda = 1049; // [ Btu / l b ] M = 29; B = 24; // [ i n . ] D = 2; // [ i n . ] Dc = 2; // [ f t ] // S o l u t i o n // ( a ) // mass v e l o c i t y G = v * M *492*3600/(359*(460+120) ) ; // [ l b / f t 2 h ]

126

21 // t h e c o e f f i c e n t , by Eq . ( 2 4 . 1 3 ) , i n f p s u n i t s , i s 22 h = 0.01* G ^0.2/2^0.2; // [ Btu / f t 2 hF ] 23 // S u b s t i t u t i n g i n Eq . ( 2 1 . 1 5 ) g i v e s 24 Rc = 1.94*( Tdb - Twb ) /( lambda ) ; // [ l b / f t 2 h ] 25 disp ( l b / f t 2 h ,Rc , D r y i n g r a t e d u r i n g t h e c o n s t a n t

period is )
26 27 28 29 30 31 32 33 34 35 36 37 38 39 40 41 42

// ( b ) // S i n c e d r y i n g i s from b o t h f a c e s , a r e a A = Dc *( B /12) ^2; // [ f t 2 ] // The r a t e o f d r y i n g mvdot = Rc * A ; // [ l b / h ] // Volume o f t h e c a k e Vc = ( B /12) ^2* D /12; // [ f t 3 ] // mass o f t h e bone d r y s o l i d i s mdot_bd = rho * Vc ; // [ l b ] // The q u a n t i t y o f m o i s t u r e t o be v a p o r i z e d i s X2 = 0.20; X1 = 0.10; Q = mdot_bd *( X2 - X1 ) ; // [ l b ] // D r y i n g t i m e tT = Q / mvdot ; // [ h ] disp ( h ,tT , d r y i n g t i m e ) Example 24.2 Example 24.2.sce

1 2 3 4 5 6 7 8 9 10 11 12

clear all ; clc ; // Example 2 4 . 2 // Given X1 = 0.25; X = 0.05; Dvprime = 8.3*10^ -6; // [ cm2/ s ] D = 25.4; // [mm] // S o l u t i o n s = D /(2*10) ; // [ cm ] 127

13 tT = 4* s ^2/( %pi ^2* Dvprime ) * log (8* X1 /( %pi ^2* X ) ) /3600; 14

// [ h ] disp ( h ,tT , d r y i n g t i m e i s ) Example 24.3 Example 24.3.sce

1 2 3 4 5 6 7 8 9 10 11 12 13 14 15 16 17 18 19 20 21 22 23 24 25 26 27 28 29 30

clear all ; clc ; // Example 2 4 . 3 // Given Tw = 80; // [ F ] Tdb = 120; // [ F ] v = 3.5; // [ f t / s ] rho = 120; // [ l b / f t 3 ] Xe = 0; Xc = 0.09; lambda = 1049; // [ Btu / l b ] M = 29; B = 24; // [ i n . ] D = 2; // [ i n . ] Dc = 2; // [ f t ] X2 = 0.20; X1 = 0.10; Dcyl = 1/4; // [ i n . ] L = 4; // [ i n . ] Vbar = 3.5; // [ f t / s ] Thb = 120; // S o l u t i o n // S i n c e t h e Xc i s l e s s t h a n 10 p e r c e n t , a l l d r y i n g takes place // i n t h e c o n s t a n t r a t e p e r i o d and t h e v a p o r r i z a t i o n temperature , // a s b e f o r e , i s 80 F . // From Exapmle 2 4 . 1 , mass o f w a t e r t o be e v a p o r a t e d mdot = 8*( X2 - X1 ) ; // [ l b ] // The q u a n t i t y o f h e a t t o be t r a n s f e r r e d 128

31 32 33 34 35 36 37 38 39 40 41 42 43 44 45 46 47 48 49 50 51 52 53 54 55 56 57 58 59 60 61

QT = mdot * lambda ; // [ Btu ] // mass o f t h e d r y s o i l d i n one c y l i n d e r i s mp = %pi /4*( Dcyl /12) ^2*( L /12) * rho ; // [ l b ] // s u r f a c e a r e a o f one c y l i n d e r i s Ap = %pi *( Dcyl /12) *( L /12) ; // [ f t 2 ] // T o t a l a r e a e x p o s e d by 8 l b s o l i d s A = 8/ mp * Ap ; // [ f t 2 ] // The h e a t t r a n s f e r c o e f f i c i e n t i s f o u n d from t h e // e q u i v a l e n t form o f Eq . ( 2 1 . 6 2 ) // hDbyk = 1 . 1 7 Nre 0 . 5 8 5 Npr ( 1 / 3 ) // For a i r a t 1 atm and 120F , t h e p r o p e r t i e s a r e rho_a = M /359*492/580; // [ l b / f t 3 ] mu_a = 0.019; // [ cP ] , from Appendix 8 k_a = 0.0162; // [ Btu / f t hF ] , from Appendix 12 Cp_a = 0.25; // [ Btu / l b F ] , from Appendix 15 Nre = 1/48* Vbar * rho_a /( mu_a *6.72*10^ -4) ; Npr = mu_a *2.42* Cp_a / k_a ; // Form Eq . ( 2 1 . 6 2 ) h = ( k_a *1.17* Nre ^0.585* Npr ^(1/3) ) /(1/48) ; // [ Btu / f t 2 hF ] mdot_g = v *3600* rho_a ; // [ l b ] // From F i g . 2 3 . 2 cs = 0.25; delta_Thb = Thb - Tw ; // [ F ] delta_Tha = 8.24; // [ F ] // The h e a t t r a n s f e r r e d form t h e g a s t o a t h i n s e c t i o n o f t h e bed delta_TL = ( delta_Thb - delta_Tha ) / log ( delta_Thb / delta_Tha ) ; // [ F ] // r a t e o f h e a t t r a n s f e r qT = h * A * delta_TL ; // [ Btu / h ] // d r y i n g t i m e tT = QT / qT ; // [ h ] disp ( h ,tT , R e q u i r e d d r y i n g t i m e i s ) Example 24.4 Example 24.4.sce

clear all ; 129

2 3 4 5 6 7 8 9 10 11 12 13 14 15 16 17 18 19 20 21 22 23 24 25 26 27 28 29 30 31 32 33 34 35 36

clc ; // Example 2 4 . 4 // Given msdot = 2800; // [ l b / h ] Xa = 0.15; Xb = 0.005; Ti = 80; // [ F ] To = 125; // [ F ] Thb = 260; // [ F ] Hb = 0.01; // [ l b w a t e r / l b d r y a i r ] G = 700; // [ l b / f t 2 h ] Cps = 0.52; // [ Btu / l b F ] // S o l u t i o n // C o u n t e r c u r r e n t o p e r a t i o n w i l l be u s e d . // Assuming Nt = 1.5; //NTU // From F i g . 2 3 . 2 Twb = 102; // [ F ] // From Eq . ( 2 . 4 8 ) Tha = ( Thb - Twb ) / exp ( Nt ) + Twb ; // [ F ] Tsb = To ; // [ F ] lambda = 1036; // [ Btu / l b ] , a t 102 F , from Appendix 7 Cpv = 0.45; // [ Btu / l b F ] , from Appendix 15 Cpl = 1.0; // [ Btu / l b F ] // From Eq . ( 2 4 . 9 ) mvdot = msdot *( Xa - Xb ) ; // [ l b / h ] // The h e a t duty i s f o u n d form s u b s t i t u t i o n i n Eq .(24.1) qTdot = Cps *( To - Ti ) + Xa * Cpl *( Twb - Ti ) +( Xa - Xb ) * lambda + Xb * Cpl *( To - Twb ) +( Xa - Xb ) * Cpv *( Tha - Twb ) ; // [ Btu / l b ] qT = qTdot * msdot ; // [ Btu / h ] // The f l o w r a t e o f t h e e n t e r i n g a i r i s f o u n d from a h e a t b a l a n c e and t h e humid h e a t c s b . // From F i g . 2 3 . 2 csb = 0.245; // [ Btu / l b F ] , mgdot = qT /( csb *( Thb - Tha ) *(1+ Hb ) ) ; // [ l b / h o f d r y 130

air ] 37 // From Eq . ( 2 4 . 1 0 ) , The o u t l e t h u m i d i t y 38 Ha = Hb + mvdot / mgdot ; // [ l b / l b ]


39 40 41 42 43 44 45 46 47

// For a g i v e n f l o w r a t e , t h e c r o s s s e c t i o n a l a r e a o f t h e d r y e r must be Ac = qT /( csb *( Thb - Tha ) ) / G ; // [ f t 2 ] // The d r y e r d i a m e t e r i s D = (4* Ac / %pi ) ^0.5; // [ f t ] delta_TL = (( Thb - Twb ) -( Tha - Twb ) ) / log (( Thb - Twb ) /( Tha Twb ) ) ; // [ F ] // U s i n g Eq . ( 2 4 . 2 9 ) , t h e d r y e r l e n g t h L = qT /(0.125* %pi * D * G ^0.67* delta_TL ) ; // [ f t ] disp ( r e s p e c t i v e l y , f t ,L , f t ,D , R e q u i r e d d i a m e t e r and l e n g t h o f t h e d r y e r i s )

131

Chapter 25 Adsorption
25.1 Scilab Code

Example 25.1 Example 25.1.sce


1 2 3 4 5 6 7 8 9 10 11 12 13 14 15 16 17 18 19

clear all ; clc ; // Example 2 5 . 1 // Given ya = 0.002; T = 20+273; // [ K ] // S o l u t i o n // ( a ) M = 86.17; // from P e r r y s C h e m i c a l E n g i n e e r s Handbook , 6 t h ed . Pprime = 120; // [mm Hg ] fs = Pprime ; // [mm Hg ] rho_L = 0.615; // [ g /cm 3 ] , a t n o r m a l b o i l i n g p o i n t ( 6 8 . 7 C) P = 760; // [mm Hg ] p = ya * P ; // [mm Hg ] f = p ; // [mm Hg ] V = M / rho_L ; // [ cm3/ g mol ]

132

20 // L e t 21 A = T / V * log10 ( fs / f ) ; 22 // From F i g . 2 5 . 4 , volume a d s o r b e d 23 V_ads = 31/100; // [ cm3 l i q u i d / g c a r b o n ] 24 W = V_ads * rho_L ; // [ g / g c a r b o n ] 25 disp ( g / g c a r b o n ,W , The e q u i l i b r i u m c a p a c i t y

for

t h e bed i s )
26 27 28 29 30 31 32 33 34 35

// ( b ) T = 40+273; // [ K ] Pprime = 276; // [mm Hg ] fs = Pprime ; // [mm Hg ] A = T / V * log10 ( fs / f ) ; // From F i g . 2 5 . 4 , volume a d s o r b e d V_ads = 27/100; // [ cm3 l i q u i d / g c a r b o n ] W = V_ads * rho_L ; // [ g / g c a r b o n ] disp ( g / g c a r b o n ,W , The e q u i l i b r i u m c a p a c i t y f o r t h e bed i s ) Example 25.2 Example 25.2.sce

1 2 3 4 5 6 7 8 9 10 11 12 13 14 15 16 17

clear all ; clc ; // Example 2 5 . 2 // S o l u t i o n cbyc0 =0.05; u0 = 58; // [ cm/ s ] Dv = 0.37; // [m2/ g ] c0 = 365; // [ ppm ] S = 1194; // [m2/ g ] T = 25; // [ C ] rho_b = 0.461; // [ g /cm 3 ] P = 737; // [mm Hg ] M = 74.12; // [ g / mol ] eps = 0.457; t = 1:0.5:8.5; t (4) = 2.4; t (5) = 2.8; t (6) = 3.3; 133

18

cbyc0 =[0.005 ,0.01 ,0.027 ,0.05 ,0.1 ,0.2 ,0.29 ,0.56 ,0.0019 ,0.003 ,0.0079 ,0.01

19 t1 = t (1:8) ; 20 t2 = t (9:16) ; 21 cbyc01 = cbyc0 (1:8) ; 22 cbyc02 = cbyc0 (9:16) ; 23 plot ( t1 , cbyc01 , t2 , cbyc02 ) ; 24 xgrid () ; 25 xlabel ( t , Hours ) ; 26 ylabel ( c / c 0 ) ; 27 title ( B r a k t h r o u g h c u r v e s f o r Example 2 5 . 2 ) ; 28 legend ( L = 8cm , L = 16cm ) ; 29 30 // ( a ) 31 FA = u0 * c0 *10^ -6/22400*273/298*737/760* M *3600; // [ g / 32 33 34 35 36 37 38 39 40 41 42 43 44 45 46 47 48

cm2 h ] // The t o t a l s o l u t e a d s o r b e d i s t h e a r e a a b o v e t h e graph m u l t i p l i e d // by FA . For t h e 8cm bed , t h e a r e a i s Area_bed = 4.79; // [ h ] // T h i s a r e a c o r r e s p o n d s t o t h e i d e a l t i m e t h a t would be r e q u i r e d t o a d s o r b // t h e same amount i f t h e b r e a k t h r o u g h c u r v e w e r e a v e r t i c a l l i n e . The mass // o f c a r b o n p e r u n i t c r o s s s e c t i o n a l a r e a o f t h e bed is Ac = 8* rho_b ; // [ g /cm 2 ] // Thus , Wsol = FA * Area_bed / Ac ; // [ g s o l u t e / g c a r b o n ] // At t h e b r e a k p o i n t , where cbyc0_break = 0.05; // and t_break =2.4; // [ h ] Area_bed_break = 2.37; // [ h ] // The amount a d s o r b e d up t o t h e b r e a k p o i n t i s t h e n Wb = FA * t_break / Ac ; // [ g s o l u t e / g c a r b o n ] ratio_W = Wb / Wsol ; 134

49 50 51 52 53 54 55 56 57 58 59 60 61 62 63 64 65 66 67

// Thus 50 p e r c e n t o f t h e bed c a p a c i t y i s unused , which can be r e p r e s e n t d // by a l e n g t h 4 cm . // For t h e 16 cm bed t h e b r e a k t h r o u g h c u r v e h a s t h e same i n i t i a l s l o p e a s t h e c u v e // f o r 8cm bed , and a l t h o u g h d a t a w e r e n o t t a k e n beyond c b y c 0 = 0 . 2 5 , // t h e c u r v e s a r e assumed t o be p a r a l l e l // For t h e e n t i r e bed , tT = 9.59; // [ h ] Wsat = FA * tT /(16* rho_b ) ; // [ g s o l u t e / g c a r b o n ] // At cbyc0_break = 0.05; t_break =7.1; // [ h ] Area_break = 7.07; // [ h ] Wb = FA * Area_break /(16* rho_b ) ; // [ g s o l u t e / g c a r b o n ] ratio_W = Wb / Wsat ; // At t h e b r e a k p o i n t , 74 p e r c e n t o f t h e bed c a p a c i t y i s used , // which c o r r e s p o n d s t o an u n u s e d s e c t i o n o f l e n g t h 0 . 2 6 1 6 cm . // Within e x p e r i m e n t a l e r r o r , t h e l e n g t h s o f u n u s e d bed a g r e e , // and 4 . 1 cm i s e x p e c t e d v a l u e f o r a s t i l l l o n g e r bed . disp ( cm ,4.2 , l e n g t h o f t h e bed u s e d , p e r c e n t , ratio_W , s a t u r a t i o n c a p a c i t y o f t h e c a r b o n )

68 69 // ( b ) 70 L = 32; // [ cm ] 71 L_exp = L -4.1; // [ cm ] 72 // F r a c t i o n o f t h e bed u s e d 73 fra_bed = L_exp / L ; 74 // The b r e a k p o i n t t i m e i s , 75 tb = L_exp * rho_b * Wsat / FA ; // [ h ] 76 disp ( h ,tb , b r e a k p o i n t t i m e )

135

Figure 25.1: Breakthrough curves for Example 25.2 Example 25.3 Example 25.3.sce
1 2 3 4 5 6 7 8 9

clear all ; clc ; // Example 2 5 . 3 // S o l u t i o n cbyc0 =0.05; u0 = 58; // [ cm/ s ] Dv = 0.37; // [m2/ g ] c0 = 365; // [ ppm ] 136

10 11 12 13 14 15 16 17 18 19 20 21 22 23 24 25 26 27 28 29 30 31 32 33 34 35 36 37 38 39 40 41 42 43 44

S = 1194; // [m2/ g ] T = 25; // [ C ] rho_b = 0.461; // [ g /cm 3 ] P = 737; // [mm Hg ] M = 74.12; // [ g / mol ] eps = 0.457; L = 8; // [ cm ] // ( a ) // From Example 2 5 . 2 ratio_W = 0.495; tou = 0.495; // From F i g . 2 5 . 1 0 N = -1.6/( tou -1) ; // a t c / c 0 = 0 . 0 5 Kca = N * u0 / L ; // [ s 1] disp ( s 1 ,Kca , Kca = ,N , N = ) // p l o t ( t1 , cbyc01 , t2 , c b y c 0 2 ) // ( b ) Dp = 0.37; // [ cm ] mubyrho = 0.152; // [ cm2/ s ] , a t 25C , 1 atm Dv = 0.0861; // [ cm2/ s ] Nre = Dp * u0 / mubyrho ; Nsc = mubyrho / Dv ; // From Eq . ( 2 1 . 6 2 ) , Nsh = 1.17* Nre ^0.585* Nsc ^(1/3) ; kc = Nsh * Dv / Dp ; // [ cm/ s ] a = 6*(1 - eps ) / Dp ; // [ cm2/cm 3 ] kca = kc * a ; // [ s 1] // S i n c e Kca i s s l i g h t l y l e s s t h a n h a l f t h e p r e d i c t e d v a l u e o f kca , // t h e e x t e r n a l r e s i s t a n c e i s c l o s e t o h a l f t h e t o t a l r e s i s t a n c e , and // t h e c a l c u l a t e d v a l u e o f N n e e d n o t be r e v i s e d . The internal // c o e f f i c i e n t can be o b t a i n e d from Kc = Kca / a ; // [ cm/ s ] kc_int1 = 1/(1/ Kc -1/ kc ) ; // [ cm/ s ] 137

45

// I f t h e d i f f u s i o n i n t o t h e p a r t i c l e o c c u r e d o n l y i n t h e g a s phase , t h e 46 //maximum p o s s i b l e v a l u e o f De would be a b o u t Dv / 4 , which l e a d s t o 47 kc_int2 = 10* Dv /(4* Dp ) ; // [ cm/ s ] 48 disp ( Kca i s s l i g h t l y l e s s t h a n h a l f t h e p r e d i c t e d v a l u e o f kca ); Example 25.4 Example 25.4.sce

1 2 3 4 5 6 7 8 9 10 11 12 13 14 15 16 17 18 19 20 21 22 23 24 25 26 27 28

clear all ; clc ; // Example 2 5 . 4 y = 0.0012; vdot = 16000; // [ f t 3/ min ] P = 760; // [mm Hg ] rho_b = 30; // [ l b / f t 3 ] Lun = 0.5; // [ f t ] // S o l u t i o n // ( a ) // Form t h e hand book Pprime = 151; // [mm Hg ] fs = Pprime ; // [mm Hg ] rho_L = 0.805; // [ g /cm 3 ] , a t 20C Tnb = 79.6; // [ C ] rho_e = 0.75; // [ g /cm 3 ] M = 72.1; V = M / rho_e ; p = y * P ; // [mm Hg ] f = p ; // [mm Hg ] // At 35C T = 35+273; // [ K ] A = T / V * log10 ( fs / f ) ; // Form F i g . 2 5 . 4 , // t h e volume a d s o r b e d V_ads = 24; // [ cm 3 / 1 0 0 g c a r b o n ] 138

29 Wsat = V_ads * rho_e ; // [ g / 1 0 0 g c a r b o n ] 30 W0 = 1/3* Wsat ; // [ g / 1 0 0 g c a r b o n ] 31 Working_capacity = Wsat - W0 ; // [ g / 1 0 0 g c a r b o n ] 32 // o r 33 Working_capacity = Working_capacity /100; // [ l b / l b

carbon ] 34 disp ( Working_capacity , Working c a p a c i t y o f t h e bed is ) // ( b ) u0 = 1; // [ f t / s ] A = vdot / u0 ; // [ f t 2 ] D = sqrt (4* A / %pi ) ; // [ f t ] Abed = 10*27; // [ f t 2 ] L1 = 4; // [ f t ] c0 = y /359*273/298*72.1; // [ l b / f t 3 ] // Form Eq . ( 2 5 . 3 ) tstar = L1 * rho_b *( Working_capacity ) /( u0 * c0 *3600) ; // [h] 45 Lu1 = L - Lun ; // [ f t ] 46 tb1 = Lu1 / L * tstar ; // [ h ]
47 48 49 50 51 52 53 54 55 56 57 58 59 60 35 36 37 38 39 40 41 42 43 44

// i f L2 = 3; // [ f t ] Lu2 = L2 - Lun ; tb2 = Lu2 / L * tstar ; // [ h ] // c h e c k i n g f o r d e l t a P // U s i n g Eq . ( 7 . 2 2 ) phi_s = 0.7; // from T a b l e 2 8 . 1 eps = 0.35; // from T a b l e 7 . 1 mu = 1.21*10^ -5; // [ l b / f t s ] rho = 0.074; // [ l b / f t 3 ] // For a 4 10 mesh c a r b o n Dp = 1.108*10^ -2; // [ f t ] deltaPbyL = 150*1* mu *(1 - eps ) ^2/(32.2* phi_s ^2* Dp ^2* eps ^3) +(1.75* rho *1^2*(1 - eps ) /(32.2*0.7* Dp * eps ^3) ) ; // [ l b f / f t 2 f t ] 61 deltaPbyL = deltaPbyL *12/62.4; // [ i n . H2O/ f t ] 139

62 // f o r 63 L = 3; 64 deltaP = 3* deltaPbyL ; // [ i n . H2O ] 65 // which s a t i s f a c t o r y . 66 mc = 2*(10*27*3) *30; // [ l b ] 67 68 disp ( f t ,L2 , A l l o w i n g f o r u n c e r t a i n t i e s

in the c a l c u l a t i o n s , s a t i s f a c t o r y bed l e n g t h w i l l be ) 69 disp ( f t / s ,u0 , g a s v e l o c i t y n e e d e d ) 70 disp ( l b ,mc , c a r b o n n e e d e d )

140

Chapter 26 Membrane Separation Processes


26.1 Scilab Code

Example 26.1 Example 26.1.sce


1 2 3 4 5 6 7 8 9 10 11 12 13 14 15 16 17 18

clear all ; clc ; // Example 2 6 . 1 // Given alpha = 5; per = 0.2; // [ s c f / f t 2 hatm ] Pf = 150; // [ l b f / i n . 2 ] Pp = 15; // [ l b f / i n . 2 ] // S o l u t i o n // ( a ) R = Pp / Pf ; // At t h e f e e d i n l e t xin = 0.209; // U s i n g Eq . ( 2 6 . 1 7 ) A = alpha -1; B = 1 - alpha -1/ R - xin *( alpha -1) / R ;

141

19 20 21 22 23 24 25 26 27 28 29 30 31 32 33 34 35 36 37 38 39 40 41 42 43 44 45 46 47 48 49 50 51 52 53 54

C = alpha * xin / R ; yi_in = ( -B - sqrt ( B ^2 -4* A * C ) ) /(2* A ) ; // At t h e d i s c h a r g e end xd = 0.05; // U s i n g Eq . ( 2 6 . 1 7 ) A = alpha -1; B = 1 - alpha -1/ R - xd *( alpha -1) / R ; C = alpha * xd / R ; yi_d = ( -B - sqrt ( B ^2 -4* A * C ) ) /(2* A ) ; // For an a p p r o x i m a t e s o l u t i o n , t h e s e t e r m i n a l compositions are // a v e r a g e d t o g i v e ybar = ( yi_in + yi_d ) /2; // From an o v e r a l l m a t e r i a l b a l a n c e // B a s i s Lin = 100; // [ s c f h ] V = ( Lin * xin - Lin * xd ) /( ybar - xd ) ; // d i s p ( ybar , and p e r m e a t e c o m p o s i t i o n i s , p e r c e n t , V/ L i n 1 0 0 , The p e r m e a t e i n t h e f e e d i s ) ;

// For more a c c u r a t e c a l c u l a t i o n j = 2; yi_in (1) = 0.5148; x (1) = 0.209; y (1) = 0.5148; L = Lin ; deltaV = []; deltaVybar = []; ybar = []; for i = 0.2: -0.01: xd x(j) = i; A = alpha -1; B = 1 - alpha -1/ R - x ( j ) *( alpha -1) / R ; C = alpha * x ( j ) / R ; yi_in ( j ) = ( -B - sqrt ( B ^2 -4* A * C ) ) /(2* A ) ; ybar (j -1) = ( yi_in (j -1) + yi_in ( j ) ) /2; 142

55 56 57 58 59 60 61 62 63 64 65 66 67 68 69 70 71 72 73 74 75 76 77 78 79 80 81

deltaV ( j ) = L *( x (j -1) -x ( j ) ) /( ybar (j -1) -x ( j ) ) ; V = sum ( deltaV ) ; L = Lin - V ; deltaVybar ( j ) = deltaV (j -1) * ybar (j -1) ; deltaVybarsum = sum ( deltaVybar ) ; y (j -1) = deltaVybarsum / V ; j = j +1; end disp ( y ( $ ) , and p e r m e a t e c o m p o s i t i o n i s , p e r c e n t ,V / Lin *100 , The p e r m e a t e r e c o v e r e d ) ;;

// ( b ) // The membrane a r e a o b t a i n e d from t h e f l u x o f A using // Eq . ( 2 6 . 2 9 ) and ( 2 6 . 1 3 ) // f o r t h e f i r s t i n c r e m e n t x = 0 . 2 0 9 t o x = 0 . 2 deltaybar1 = 1.4856; // [ s c f h ] , f o r L i n = 100 s c f h // At x = 0 . 2 0 9 A1 = 0.209 -0.1*0.5148; // At x = 0 . 2 A2 = 0.2 -0.1*(0.50) ; Aavg = ( A1 + A2 ) /2 QAP1 = 0.2*10; // s c f h / f t 3 // f o r s p e c i f i e d f l o w o f 300 s c f h deltaA = 1/2*1.486/ Aavg *180; // [ f t 2 ] // The c a l c u l a t i o n c o n t i n u e d w i t h i n c r e m e n t s o f 0 . 0 1 A = 211/2.0*180; // [ f t 2 ] disp ( f t 2 ,A , The membrane a r e a n e e d e d i s ) Example 26.4 Example 26.4.sce

1 clear all ; 2 clc ; 3 4 // Example 2 6 . 4 5 // Given 6 F = 10; // [ g a l / day f t 3 ]

143

7 8 9 10 11 12 13 14 15 16 17 18 19 20 21 22 23 24 25 26

Do = 300*10^ -6; // [m] Di = 200*10^ -6; // [m] vi = 0.5; // [ cm/ s ] rho = 1; // [ g /cm 3 ] mu = 0.01; // [ g /cm s ] , assumed f = 0.97; // S o l u t i o n // For 10 g a l / day f t 2 Jw = F *231*16.3871/(24*3600*929) ; // [ cm/ s ] Nre = Do *100* vi * rho / mu ; Ds = 1.6*10^ -5; // [ cm2/ s ] Nsc = mu /( rho * Ds ) ; // U s i n g Eq . ( 1 2 . 6 9 ) , A n a l o g o u s l y t o mass t r a n s f e r Nsh = (0.35+0.56* Nre ^0.52) / Nsc ^ -0.3; kc = Nsh * Ds /( Do *100) ; // [ cm/ s ] // From Eq . ( 2 6 . 4 9 ) gama = Jw * f / kc ; disp ( A c o n c e n t r a t i o n d i f f e r n c e o f 12 p e r c e n t w i l l n o t be s i g n i f i c a n t t i l l good f l o w d i s t r i b u t i o n i s maintained ); Example 26.5 Example 26.5.sce

1 2 3 4 5 6 7 8 9 10 11 12 13

clear all ; clc ; // Example 2 6 . 5 // Given ( from Example 2 6 . 4 ) F = 10; // [ g a l / day f t 2 ] , b a s e d on e x t e r n a l a r e a Do = 300*10^ -6; // [m] Di = 200*10^ -6; // [m] vi = 0.5; // [ cm/ s ] rho = 1; // [ g /cm 3 ] mu = 10^ -3; // [ Pa s ] , assumed f = 0.97; L = 3; // [m] 144

14 15 // S o l u t i o n 16 // ( a ) 17 //Jw b a s e d on a r e a 18 Jw = 4.72*10^ -4* Do / Di *10^ -2; // [m/ s ] 19 dt = 200*10^ -6; // [m] 20 D = dt ; // [m] 21 // From Eq . ( 2 6 . 5 3 ) 22 Vbar = 4*( Jw ) * L / Di ; // [m/ s ] 23 // From Eq . ( 2 6 . 5 6 ) 24 delta_ps = ( Vbar *32* mu * L ) /( D ) ^2*(1/2) /10^5; // [ atm ] 25 disp ( atm , delta_ps , p r e s s u r e d r o p = , m/ s , Vbar ,

e x i t v e l o c i t y = );
26 27 28 29 30 31 32

// ( b ) // I f t h e f i b r e s a r e open a t b o t h ends , t h e e f f e c t i v e l e n g t h i s 1 . 5m and // t h e e x i t v e l o c i t y i s h a l f a s g r e a t . The p r e s s u r e d r o p i s one f o u r t h a s // l a r g e a s i t was : deltaP = delta_ps /4; // [ atm ] disp ( atm , deltaP , p r e s s u r e d r o p ( i f b o t h e n d s a r e open ) = )

145

Chapter 27 Crystallization
27.1 Scilab Code

Example 27.1 Example 27.1.sce


1 2 3 4 5 6 7 8 9 10 11 12 13 14 15 16 17 18 19

clear all ; clc ; // Example 2 7 . 1 // Given T = 60; // [ F ] wA = 0.30; // [ MgSO4 ] wB = 0.70; // [ H2O ] // S o l u t i o n // From F i g . 2 7 . 3 i t i s n o t e d t h a t t h e c r y s t a l s a r e MgSO4 . 7 H2O // and t h a t t h e c o n c e n t r a t i o n o f t h e mother l i q u i d i s xA = 0.245; // [ a n h y d r o u s MgSO4 ] xB = 0.755; // [ H2O ] // B a s e s : F_in = 1000; // [ kg ] H2O_in = F_in * wB ; // [ kg ] H2O_evp = 0.05* H2O_in ; // [ kg ] M1 = 120.4; // [ MgSO4 m o l e c u l a r w e i g h t ]

146

20 M2 = 246.5; // [ MgSO4 . 7 H2O m o l e c u l a r w e i g h t ] 21 M2_in = wA * F_in * M2 / M1 ; // [ kg ] 22 H2O_free = F_in - H2O_evp - M2_in ; // [ kg ] 23 ML = 100; // [ kg ] 24 M2_in100 = ML * xA * M2 / M1 ; // [ kg ] 25 H2O_free100 = ML - M2_in100 ; // [ kg ] 26 M2_ML = M2_in100 / H2O_free100 * H2O_free ; // [ kg ] 27 FC = M2_in - M2_ML ; // [ kg ] 28 disp ( FC , k i l o g r a m s o f c r y s t a l s o b t a i n e d p e r k i l o g r a m

of o r i g i n a l mixture = ) Example 27.2 Example 27.2.sce


1 2 3 4 5 6 7 8 9 10 11 12 13 14 15 16 17 18 19 20 21 22

clear all ; clc ; // Example 2 7 . 2 // Given //A = MgSO4 , B = MgSO4 . 7 H2O and C = H2O T = 120; // [ F ] wA = 0.325; // S o l u t i o n // From F i g 2 7 . 4 // E n t h a l p y c o o r d i n a t e o f t h e p o i n t wA H1 = -33; // [ Btu / l b ] // E n t h a l p y c o o r d i n a t e o f t h e f i n a l magma a t c o n c e n t r a t i o n wA H2 = -78.4; // [ Btu / l b ] // Per hundred pouds o f o r i g i n a l s o l u t i o n t h e c h a n g e in enthalpy F = 100; // [ l b ] delta_H = F *( H1 - H2 ) ; // [ Btu ] // A p p l y i n g c e n t e r o f g r a v i t y p r i n c i p l e t o 70 F isotherm in Fig . 27.3 C_ML = 0.259; C_CRY = 0.488; // C r y s t a l s a r e 147

23 Cry = F *( wA - C_ML ) /( C_CRY - C_ML ) ; // [ l b / 1 0 0 l b s l u r r y ] 24 // The h e a t e v o l v e d p e r t o n o f c r y s t a l s i s 25 H = delta_H / Cry *2000; // [ Btu / t o n ] 26 disp ( Btu / t o n ,H , The h e a t e v o l v e d p e r t o n o f

crystals is ) Example 27.3 Example 27.3.sce


1 2 3 4 5 6 7 8 9 10 11 12 13 14 15 16 17 18 19 20 21 22 23 24 25 26

clear all ; clc ; // Example 2 7 . 3 // Given sigma = 2.5; // [ e r g /cm 3 ] T = 300; // [ K ] N = 6.0222*10^23; R = 8.3134*10^7; // [ e r g / g mol K ] // S o l u t i o n M = 74.56; // [ M o l e c u l a r w e i g h t ] rho = 1.988; // [ g /cm 3 ] nu = 2; VM = M / rho // [ cm3/ g mol ] // U s i n g Eq . ( 2 7 . 1 1 ) // E x p o n e n t i a l term , e x c l u d i n g s A = 16* %pi * VM ^2* N * sigma ^3*10/(3*( T * R ) ^3* nu ^2) B0 = 1; s (1) = sqrt ( - A / log ( B0 /10^25) ) ; // For B0 ; s = s (1) :0.0001:0.029; B0 = exp (57.565) * exp ( - A ./ s .^2) ; plot (s , B0 ) title ( B0 v s s ) xlabel ( s ) ylabel ( B0 ) Example 27.4 Example 27.4.sce

clear all ; 148

2 3 4 5 6 7 8 9 10 11 12 13 14 15 16 17 18 19

clc ; // Example 2 7 . 4 // Given alpha = 1+0.029; // From Example 2 7 . 3 sigma = 2.5; // [ e r g /cm 3 ] T = 300; // [ K ] N = 6.0222*10^23; R = 8.3134*10^7; // [ e r g / g mol K ] M = 74.56; // [ M o l e c u l a r w e i g h t ] rho = 1.988; // [ g /cm 3 ] nu = 2; VM = M / rho ; // [ cm3/ g mol ] // U s i n g Eq . ( 2 7 . 9 ) L = 4* VM * sigma /(2* R * T * log ( alpha ) ) *10^7; // [ nm ] disp ( nm ,L , s i z e o f n u c l u e s ( L ) = ) ; Example 27.5 Example 27.5.sce

1 2 3 4 5 6 7 8 9 10 11 12 13 14 15 16 17

clear all ; clc ; // Example 2 7 . 5 // L e t : A = MgSO4 ; B = MgSO4 . 7 H2O ; C = H2O // Given xA = 0.31; T = 86; // [ F ] Tb = 2; // [ F ] vbys = 0.15; //PB = rho_cr = 105; // [ l b / f t 3 ] rho_ml = 82.5; // [ l b / f t 3 ] // S o l u t i o n // B a s i s : F = 10000; // [ l b / h ] 149

18 19 20 21 22 23 24 25 26 27 28 29 30 31 32 33 34 35 36

// From F i g 2 7 . 1 3 and F i g 2 7 . 4 crbyml = vbys * rho_cr /((1 - vbys ) * rho_ml ) ; ml_prod = F / crbyml ; // [ l b / h ] magma_prod = F + ml_prod // [ l b / h ] xA_avg = ( crbyml *0.488+0.285) /1.224; // The e n t h a l p y o f t h e magam Hmag = ( crbyml *( -149) +( -43) ) /1.224; // [ Btu / l b ] // These a r e t h e c o n c e n r a t i o n s o f t h e p o i n t e . The p o i n t f o r t h e f e e d must // l i e on t h e s t r a i g h t l i n e a e . // The e n t h a l p y o f t h e f e e d Hf = -21; // [ Btu / l b ] // T e m p e r a t u r e o f t h e f e e d Tf = 130; // [ F ] //By COG p r i n c i p l e , t h e e v a p o r a t i o n r a t e evap_rate = magma_prod *( Hf - Hmag ) /(1098 - Hf ) ; // [ l b / h ] Total_feed = magma_prod + evap_rate ; // [ l b / h ] disp ( F ,Tf , T e m p e r a t u r e o f t h e f e e d i s ) ; disp ( l b / h , Total_feed , T o t a l f e e d r a t e ) ; disp ( l b / h , evap_rate , T o t a l e v a p o r a t i o n r a t e ) ; Example 27.6 Example 27.6.sce

// Example 2 7 . 6 // Given G = 0.0018; // [ f t / h ] // S o l u t i o n // S c r e e n o p e n i n g o f 20 mesh s t a n d a r d s c r e e n i s , L = 0.00273; // [ f t ] , Appendix 20 a = 1; // [ Eq . 2 7 . 1 6 ] // From Example 2 7 . 5 // The volume f l o w r a t e o f mother l i q u o r i n t h e p r o d u c t magma 13 Q = 44520/82.5; // [ f t 3/ h ] 14 // S i n c e , when z =3 , 150

1 2 3 4 5 6 7 8 9 10 11 12

clear all ; clc ;

15 Lpr = L ; // [ f t ] 16 // U s i n g Eq . ( 2 7 . 2 8 ) 17 // drawdown t i m e 18 tou = Lpr /(3* G ) ; // [ h ] 19 // volume o f t h e l i q u i d i n t h e c r y s t a l l i z e r 20 Vc = tou * Q ; // [ f t 3 ] 21 // T o t a l magma volume 22 Vmagma = Vc /0.85*7.47; // [ g a l ] 23 disp ( g a l , Vmagma , The magma volume i n t h e 24 25 26 27 28 29 30 31 32 33 34 35 36 37 38 39 40 41 42 43 44 45 46 47 48

c r y s t a l l i z e r be ) ; // U s i n g Eq . ( 2 7 . 4 4 ) // The n u c l e a t i o n r a t e i s C = 10000; // [ l b / h ] rho_c = 105; B0 = 9* C /(2* rho_c * Vc * Lpr ^3) ; // [ n u c l e i / f t 3 h ] disp ( n u c l e i / f t 3 h ,B0 , The n u c l e a t i o n r a t e n e c e s s a r y i s ); // U s i n g Eq . ( 2 7 . 4 0 ) , t h e z e r o s i z e p a r t i c l e d e n s i t y is n0 = B0 /0.0018; // [ n u c l e i / f t 4 ] L1 = (0:8) *10^ -3; // U s i n g Eq . ( 2 7 . 2 7 ) // L e t A = l o g 1 0 ( n ) , B = l o g 1 0 ( n0 ) B = log10 ( n0 ) ; A = B - 1.1*10^3* L1 /(2.3026) ; figure (1) ; plot ( L1 *10^3 , A ) ; xgrid () ; xlabel ( L x 1 0 3 f t ) ; ylabel ( l o g n ) ; title ( P o p u l a t i o n d e n s i t y v s l e n g t h ) ; // From F i g . 2 7 . 1 5 c f o r v a l u e s o f z c o r r e s p o n d i n g t o mesh o p e n i n g s . L1 = [11 ,14 ,16 ,19 ,23 ,27 ,33 ,38 ,46 ,54 ,65 ,78] *10^ -2; z = L1 /( tou * G *100) ; // [mm] t = 0; function f = fun (z , xm ) 151

49 f = z ^3* exp ( - z ) /6; 50 endfunction 51 [ xm ]= ode (0 ,0 ,z , fun ) ; 52 for i =1: length ( xm ) 53 Diff ( i ) = z ( i ) ^3* exp ( - z ( i ) ) /6; 54 end 55 figure (2) ; 56 subplot (2 ,1 ,1) ; 57 plot (z , xm ) ; 58 xgrid () ; 59 xlabel ( z ) ; 60 ylabel ( xm ) ; 61 title ( c u m u l a t i v e mass d i s t r i b u t i o n ) ; 62 subplot (2 ,1 ,2) ; 63 plot (z , Diff ) 64 xgrid () ; 65 xlabel ( z ) ; 66 ylabel ( dxm/ dz ) ; 67 title ( d i f f e r e n t i a l mass d i s t r i b u t i o n ) ;

152

Figure 27.1: Population density vs. length Example 27.6

153

Figure 27.2: Size-distribution relations for Example 27.6

154

Chapter 28 Properties, Handling and Mixing of Particulate Soilds


28.1 Scilab Code

Example 28.1 Example 28.1.sce


1 clear all ; 2 clc ; 3 4 // Example 2 8 . 1 5 // Given 6 rho_p = 0.002650; // [ g /mm 3 ] 7 a = 2; 8 phi_s = 0.571; 9 // S o l u t i o n 10 // ( a ) 11 // For t h e 4/6 mesh i n c r e m e n t , from T a b l e 2 8 . 2 12 x =

[0 ,2.51 ,12.5 ,32.07 ,25.7 ,15.9 ,5.38 ,2.10 ,1.02 ,0.77 ,0.58 ,0.41 ,0.31 ,0. // [ mass f r a c t i o n ] 13 Dp = [4.699 ,3.327 ,2.362 ,1.651 ,1.168 ,0.833 ,0.589 ,0.417 ,0.295 ,0.208 ,0.147 // [mm] 14 Dpbar (1) = 10^ -5;

155

15 for i =2: length ( Dp ) 16 Dpbar ( i ) = ( Dp (i -1) + Dp ( i ) ) /2; 17 end 18 19 // ( a ) 20 // U s i n g Eq . ( 2 8 . 4 ) 21 Aw = 6/( phi_s * rho_p ) * sum ( x (1: $ -1) ./ Dpbar (1: $ -1) ) /(1 -

x ( $ ) ) ; // [mm2/ g ] 22 Nw = 1/( a * rho_p ) * sum ( x (1: $ -1) ./ Dpbar (1: $ -1) ^3) /(1 - x ( $ ) ) ; // [ p a r t i c l e s / g ] 23 disp ( p a r t i c l e s / g ,Nw , Nw = , mm2/ g ,Aw , Aw = ) ;
24 25 26 27

// ( b ) // U s i n g Eq . ( 2 8 . 9 ) Dvbar = (1/ sum ( x (1: $ -1) ./ Dpbar (1: $ -1) ^3) /(1 - x ( $ ) ) ) ^(1/3) ; // [mm] ; 28 disp ( mm , Dvbar , Dvbar = ) ;
29 30 31 32

// ( c ) // U s i n g Eq . ( 2 8 . 6 ) Dsbar = 1/ sum ( x (1: $ -1) ./ Dpbar (1: $ -1) ) /(1 - x ( $ ) ) ; // [ mm] 33 disp ( mm , Dsbar , Dsbar = ) ;

34 35 // ( d ) 36 // U s i n g Eq . ( 2 8 . 8 ) and T a b l e 2 8 . 3 37 Dwbar = sum ( x .* Dpbar ) ; // [mm] 38 disp ( mm , Dwbar , Dwbar = ) ; 39 40 // ( e ) 41 // U s i n g Eq . ( 2 8 . 1 1 ) 42 N2 = x ($ -1) /( a * rho_p * Dpbar ($ -1) ^3) ; // [ p a r t i c l e s / g ] 43 disp ( p a r t i c l e s / g ,N2 , Nt = ) ; 44 fra = N2 / Nw ; 45 disp ( fra , F r a c t i o n o f t h e p a r t i c l e s i n t e t o p 12

i n c r e m e n t s = ); Example 28.2 Example 28.2.sce 156

1 2 3 4 5 6 7 8 9

clear all ; clc ; // Example 2 8 . 2 // Given x = 0.14; xavg = 0.10; t = 3; // [ min ] x =[10.24 ,9.3 ,7.94 ,10.24 ,11.08 ,10.03 ,11.91 ,9.72 ,9.20 ,10.76 ,10.97 ,10

10 11 12 13 14 15 16 17 18 19

// S o l u t i o n mu = xavg ; N =12; xbar = mean ( x ) ; // S u b s t i t u i n g i n Eq . ( 2 8 . 2 0 ) Ip = sqrt (( N -1) * mu *(1 - mu ) /( sum ( x ^2) - xbar * sum ( x ) ) ) ; // U s i n g Eq . ( 2 8 . 1 8 ) s = stdev ( x ) ; disp (s , s = ,Ip , I p = )

157

Chapter 29 Size Reduction


29.1 Scilab Code

Example 29.1 Example 29.1.sce


1 2 3 4 5 6 7 8 9 10 11 12 13 14 15

clear all ; clc ; // Example 2 9 . 1 // Given mdot = 100; // [ t o n / h ] w1 = 0.80; w2 = 0.80; // S o l u t i o n Wi = 12.74; // From T a b l e 2 9 . 1 Dpa = 2*25.4; // [mm] Dpb = 0.125*25.4; // [mm] // U s i n g Eq . ( 2 9 . 1 0 ) P = mdot *0.3162* Wi *(1/ Dpb ^0.5 -1/ Dpa ^0.5) ; // [kW] disp ( kW ,P , Power r e q u i r e d (P) = ) ; Example 29.2 Example 29.2.sce

1 clear all ; 2 clc ; 3

158

4 // Example 2 9 . 2 5 // Given 6 n = 1:7; 7 beeta = 1.3; 8 // From T a b l e 2 9 . 2 9 Dpn = [3.327 ,2.362 ,1.651 ,1.168 ,0.833 ,0.589 ,0.417] ;

// [mm]
10 Dpu = Dpn ; // [mm] 11 xn0 =

[0.0251 ,0.125 ,0.3207 ,0.2570 ,0.1590 ,0.0538 ,0.0210] ;


12 Su (1) = 10*10^ -4; // [ s 1] 13 //B ( 1 ) = 1 ; 14 // S o l u t i o n 15 16 // ( a ) 17 // For t h e 4/6 mesh m a t e r i a l s 18

19 20 21 22 23 // ( b ) 24 25 // Assuming Su v a r i e s w i t h Dp3 26 for i = 1: length ( Dpn ) -1 27 Su ( i +1) = Su ( i ) *( Dpn ( i +1) / Dpn ( i ) ) ^3; // [ s 1] 28 end 29 for i = 1: length ( Dpn ) 30 for j = 1: length ( Dpu ) 31 // U s i n g Eq . ( 2 9 . 1 3 ) 32 if (j < i ) 33 B (i , j ) =0; 34 else 35 B (i , j ) = ( Dpn ( j ) / Dpn ( i ) ) ^ beeta ; 36 end

t h e r e i s no i n p u t from coarser // m a t e r i a l and a p p l y i n g Eq . ( 2 9 . 1 1 ) . At t h e end o f t i m e tT x1 = xn0 (1) *0.9; tT = 1/ Su (1) * log ( xn0 (1) / x1 ) ; // [ s ] disp ( s ,tT , R e q u i r e d t i m e i s ) ;

159

37 38 39 40 41 42 43 44 45 46 47 48 49 50 51 52 53 54 55 56 57 58 59 60 61 62 63 64 65 66 67 68 69 70 71 72 73

end end for i = 1: length ( Dpn ) -1 for j = 1: length ( Dpu ) -1 if (j < i ) delta_B (i , j ) =0; else delta_B (i , j ) = B (i , j ) -B (i , j +1) ; end end end disp ( delta_B , i n d i v i d u a l b r e a k a g e f u n c t i o n s ) ; // ( c ) deltaT = 30; // [ s ] // U s i n g Eq . ( 2 9 . 1 5 ) x =[]; x (: ,1) = xn0 ; for n = 1: length ( xn0 ) for t = 1:720 if ( n ==1) x (n , t +1) = x (n , t ) *(1 - Su ( n ) * deltaT ) ; else x (n , t +1) = x (n , t ) *(1 - Su ( n ) * deltaT ) + deltaT * Su ( n -1) * delta_B (n -1 ,n -1) * x (n -1 , t ) ; end end end time = linspace (0 ,6 ,721) ; for i =1: length ( xn0 ) plot2d ( time , x (i ,:) , style = i ) ; xgrid () ; xlabel ( t i m e ( h ) ) ; ylabel ( mass f r a c t i o n ( xa ) ) ; title ( Mass f r a c t i o n s ) ; legend ( x1 , x2 , x3 , x4 , x5 , x6 , x7 ) ; end 160

Figure 29.1: Mass-fractions of Example 29.2

161

Chapter 30 Mechanical Separations


30.1 Scilab Code

Example 30.1 Example 30.1.sce


1 clear all ; 2 clc ; 3 4 // Example 3 0 . 1 5 // Given 6 // From T a b l e 3 0 . 1 7 Dp =

[4.699 ,3.327 ,2.362 ,1.651 ,1.168 ,0.833 ,0.589 ,0.417 ,0.208 ,0.0000001] // [mm] 8 F = [0 ,0.025 ,0.15 ,0.47 ,0.73 ,0.885 ,0.94 ,0.96 ,0.98 ,1.0] ;
9 O = [0 ,0.071 ,0.43 ,0.85 ,0.97 ,0.99 ,1.00] ; // [ 1 t o 7 ] 10 U = [0.0 ,0.195 ,0.58 ,0.83 ,0.91 ,0.94 ,0.975 ,1.00] ; //

[ 3 to 10]
11 12 13 14 15

// S o l u t i o n plot ( Dp , F ) plot ( Dp (1:7) ,O , r ) plot ( Dp (3: $ ) ,U , g )

162

// Cut p o i n t d i a m e t e r from t h e T a b l e 3 0 . 1 Dcp = 1.651; // [mm] xF = 0.47; xD = 0.85; xB = 0.195; // From Eq . ( 3 0 . 3 ) DbyF = ( xF - xB ) /( xD - xB ) ; BbyF = 1 - DbyF ; // U s i n g Eq . ( 3 0 . 7 ) , o v e r a l l e f f e c t i v e n e s s E = ( xF - xB ) *( xD - xF ) *(1 - xB ) *( xD ) /(( xD - xB ) ^2*((1 - xF ) * xF ) ) ; 32 disp ( r e s p e c t i v e l y , BbyF , DbyF , mass r a t i o o f o v e r f l o w and u n d e r f l o w i s ) ; 33 disp (E , O v e r a l l E f f e c t i v e n e s s (E) = ) ;

16 17 18 19 20 21 22 23 24 25 26 27 28 29 30 31

xgrid () ; xlabel ( Dp mm ) ; ylabel ( C u m u l a t i v e mass f r a c t i o n l a r g e r t h a n Dp ) ; title ( A n a l y s i s f o r Example 3 0 . 1 ) ; legend ( Feed , O v e r s i z e , U n d e r s i z e ) ;

163

Figure 30.1: Analysis for Example 30.1 Example 30.2 Example 30.2.sce
1 2 3 4 5 6 7 8 9

clear all ; clc ; // Example 3 0 . 2 // Given // From T a b l e 3 0 . 2 V = linspace (0.5 ,6 ,12) ; // [ L ] t1 = [17.3 ,41.3 ,72 ,108.3 ,152.1 ,201.7] ; // [ s ] t2 = [6.8 ,19 ,34.6 ,53.4 ,76 ,102 ,131.2 ,163] ; // [ s ] 164

10 t3 = [6.3 ,14 ,24.2 ,37 ,51.7 ,69 ,88.8 ,110 ,134 ,160] ; // [

s]
11 t4 =

12

13 14 15 16 17 18 19 20 21 22 23 24 25 26 27 28 29 30 31 32 33 34 35 36 37 38 39 40

[5 ,11.5 ,19.8 ,30.1 ,42.5 ,56.8 ,73 ,91.2 ,111 ,133 ,156.8 ,182.5] ; // [ s ] t5 = [4.4 ,9.5 ,16.3 ,24.6 ,34.7 ,46.1 ,59 ,73.6 ,89.4 ,107.3] ; // [ s ] figure (1) ; plot ( V (1: length ( t1 ) ) , t1 ./ V (1: length ( t1 ) ) ) ; plot ( V (1: length ( t2 ) ) , t2 ./ V (1: length ( t2 ) ) , r ) ; plot ( V (1: length ( t3 ) ) , t3 ./ V (1: length ( t3 ) ) , g ) ; plot ( V (1: length ( t4 ) ) , t4 ./ V (1: length ( t4 ) ) , k ) ; plot ( V (1: length ( t5 ) ) , t5 ./ V (1: length ( t5 ) ) , y ) ; xgrid () ; xlabel ( V ( L ) ) ; ylabel ( t /V ( s /L ) ) ; legend ( deptaP = 6 . 7 , deptaP = 1 6 . 2 , deptaP = 2 8 . 2 , deptaP = 3 6 . 3 , deptaP = 4 9 . 1 ) ; title ( t /V v s V ) ; deltaP = [965 ,2330 ,4060 ,5230 ,7070] ; // [ l b f / f t 2 ] // From F i g . 3 0 . 1 5 // S l o p e ( Kc / 2 ) slope = [10440 ,5800 ,3620 ,3060 ,2400] ; // [ s / f t 6 ] Kc = slope *2; // [ s / f t 6 ] // I n t e r c e p t ( 1 / q0 ) Inter = [800 ,343 ,267 ,212 ,180] ; // [ s / f t 3 ] // V i s c o s i t y o f w a t e r muw = 5.95*10^ -4; // [ l b / f t s ] , from Appendix 14 // F i l t e r a r e a A = 440/30.48^2; // [ f t 2 ] // c o n c e n t r a t i o n c = 23.5*28.31/454; // [ l b / f t 3 ] gc = 32.14; // U s i n g Eq . ( 3 0 . 2 2 ) Rm = A * gc / muw * deltaP .*( Inter ) /10^10; // [ f t 1 1010] 165

41 42 43 44 45 46 47 48 49 50 51 52 53 54 55 56 57 58 59

// U s i n g Eq . ( 3 0 . 2 4 ) alpha = A ^2* gc /( c * muw ) * deltaP .*( Kc ) /10^11; // [ f t / l b 10 11] figure (2) ; plot2d ( deltaP , Rm ) ; xgrid () ; xlabel ( d e l t a P ( l b f / f t 2 ) ) ; ylabel ( Rm ( f t 1 10 10) ) ; title ( Rm v s d e l t a P ) ; figure (3) ; plot2d ( log ( deltaP ) , log ( alpha ) ) ; xgrid () ; xlabel ( d e l t a P ( l b f / f t 2 ) ) ; ylabel ( a l p h a ( l b / f t 10 11) ) ; title ( a l p h a v s d e l t a P ) ; // Form 3 0 . 1 7 disp ( Rm , Rm ( f t 1 10 10) = ) ; disp ( alpha , a l p h a ( l b / f t 10 11) = ) ; alpha0 = 1.75*10^11/1000^0.26; disp ( a l p h a = 2 . 9 1 0 1 0 d e l t a P 2 . 6 , E m p e r i c a l Equation f o r the cake );

166

Figure 30.2: t/V vs. V for Example 30.2

167

Figure 30.3: Rm vs. deltaP for Example 30.2

168

Figure 30.4: alpha vs. deltaP for Example 30.2 Example 30.3 Example 30.3.sce
1 2 3 4 5 6 7 8 9

clear all ; clc ; // Example 3 0 . 3 // Given f = 0.30; tc = 5*60; // [ s ] n = 1/ tc ; // [ s 1] cF = 14.7; // [ l b / f t 3 ] 169

10 11 12 13 14 15 16 17 18 19 20 21 22

deltaP = 1414; mFbymC = 2 // S o l u t i o n alpha0 = 2.9*10^10; // [ f t / l b ] , From Example 3 0 . 2 s = 0.26; mu = 6.72*10^ -4; // [ l b / f t s ] rho = 62.3; // [ l b / f t 3 ] gc =32.17; // U s i n g Eq . ( 3 0 . 1 9 ) c = cF /(1 -( mFbymC -1) *( cF / rho ) ) ; // [ l b / f t 3 ] mcdot = 10/(60*7.48) *(1/( cF /168.8+1) ) * cF ; // [ l b / s ] // S o l v i n g Eq . ( 3 0 . 3 4 ) AT = mcdot *( alpha0 * mu /(2* c *1414^(1 - s ) * gc * f * n ) ) ^(0.5) ; 23 disp ( f t 2 ,AT , F i l t e r Area (AT) = ) ; Example 30.4 Example 30.4.sce

1 2 3 4 5 6 7 8 9 10 11 12 13 14 15 16 17 18 19

clear all ; clc ; // Example 3 0 . 4 // Given D = 2; // [ cm ] Vbar = 150; // [ cm/ s ] rho = 1; // [ g /cm 3 ] mu = 0.01; // [ g /cm s ] Dv = 4*10^ -7; // [ cm2/ s ] // S o l u t i o n // ( a ) Nre = Vbar * D * rho / mu ; Nsc = mu /( rho * Dv ) ; // U s i n g Eq . ( 2 1 . 5 5 ) Nsh = 0.0096* Nre ^0.913* Nsc ^0.346; kc = Nsh * Dv / D ; // [ cm/ s ] pi = poly ([0 ,4.4*10^ -3 , -1.7*10^ -6 ,7.9*10^ -8] , c , c o e f f ); 170

20 21 22 23 24 25 26 27 28 29 30 31 32 33 34 35 36 37 38 39 40 41 42 43 44 45 46 47 48 49 50 51 52 53 54 55 56

// For c1 = 10; // [ g /L ] v = 10^ -3; // [ cm/ s ] // U s i n g Eq . ( 3 0 . 5 3 ) cs = c1 * exp ( v / kc ) ; // [ g /L ] deltaPi = horner ( pi , cs ) ; Qm = 250/36000; // [ cm/ s atm ] // U s i n g Eq . ( 3 0 . 5 0 ) deltaP = v / Qm + deltaPi ; // [ atm ] // U s i n g Eq . ( 3 0 . 5 3 ) cs = 400; vmax = kc * log ( cs / c1 ) ; // [ cm/ s ] deltaP = vmax / Qm + horner ( pi , cs ) ; // [ tm ] c = [10 ,20 ,40]; V =[]; deltaP =[]; for j = 1: length ( c ) c1 = c ( j ) ; i = 1; vmax = kc * log ( cs / c1 ) *10^4; h = ( vmax -1) /1000; for v = 1: h : vmax cs = c1 * exp ( v *10^ -4/ kc ) ; // [ g /L ] deltaPi = horner ( pi , cs ) ; // [ atm ] deltaP (j , i ) = v *10^ -4/ Qm + deltaPi ; // [ atm ] V (j , i ) = v *10^ -4; i = i +1; end end V = V *36000; for l =1: length ( c ) figure (1) plot2d ( deltaP (l ,:) ,V (l ,:) , style = l ) ; xgrid () ; xlabel ( d e l t a P ( atm ) ) ; ylabel ( Pe rmeate f l u x ( L/m2 h ) ) ; title ( E f f e c t i v e p r e s s u r e d r o p and c o n c e n t r a t i o n on f l u x ) 171

57 58 59 60 61 62 63 64 65 66 67 68

legend ( Cf =10 , , Cf =20 , Cf =40 ) ; end // ( b ) Qmb = Qm /5; // [ cm/ s atm ] vb = 10^ -3; // [ cm/ s ] c = 40; // [ g /L ] c1 = 40; csb = c1 * exp ( vb / kc ) ; deltaPi = horner ( pi , csb ) ; deltaPb = vb / Qmb + deltaPi ; disp ( The l a r g e s t e f f e c t o f t h e l o w e r membrane p e r m e a b i l i t y i s a 30 p e r c e n t r e d u c t i o n i n low p r e s s u r e drop ); i = 1; vmax = kc * log (400/ c1 ) *10^4; h = ( vmax -1) /1000; for vb = 1: h : vmax csb = c1 * exp ( vb *10^ -4/ kc ) ; // [ g /L ] deltaPi = horner ( pi , csb ) ; // [ atm ] deltaPb ( i ) = vb *10^ -4/ Qmb + deltaPi ; // [ atm ] Vb ( i ) = vb *10^ -4; i = i +1; end Vb = Vb *36000; plot2d ( deltaPb , Vb , style = l +1) legend ( Cf =10 , , Cf =20 , Cf =40 , Cf = 4 0 (Qm = 2 5 0 / 5 ) );

69 70 71 72 73 74 75 76 77 78 79 80 81

172

Figure 30.5: Eect of pressure drop and concentration on ux for Example 30.4 Example 30.5 Example 30.5.sce
1 2 3 4 5 6 7 8

clear all ; clc ; // Example 3 0 . 5 // Given D = 1.5; // [ cm ] Nre = 25000; Qm = 40; // [ L/m62h ] 173

9 10 11 12 13 14 15 16 17 18 19 20 21 22 23 24 25 26 27 28 29 30 31 32 33 34 35 36 37 38 39 40 41 42 43 44 45 46

Mw = 30000; Dv = 5*10^ -7; // [ cm2/ s ] R = 0.75; // S o l u t i o n // ( a ) // Base c a s e : v = Qm *2.78*10^ -5; // [ cm/ s ] Nsc = 0.01/ Dv ; // U s i n g Eq . ( 2 1 . 5 5 ) Nsh = 0.0096* Nre ^0.913* Nsc ^0.346; kc = Nsh * Dv / D ; // [ cm/ s ] // L e t A = K/(1 K) A = (1 - R ) / R * exp ( - v / kc ) ; K = A /(1+ A ) ; // I f t h e f l u x i s r e d u c e d t o 0 . 5 5 6 1 0 3 cm/ s // L e t B = (1 R) /R B = K /(1 - K ) * exp (0.556*10^ -3/ kc ) ; R = 1/(1+ B ) ; // As f l u x a p p r o a c h e s z e r o R a p p r a o c h e s 1K : Rmax = 1 - K ; disp (R , f r a c t i o n r e j e c t e d (R) = ) ; disp ( Rmax , maximum r e j e c t i o n ( Rmax ) = ) ; // ( b ) // U s i n g F i g . ( 3 0 . 2 4 ) kc1 = kc ; M2 = 10000; R2 = 0.35; K1 = K ; lambda1 = 1 - K1 ^0.5; lambda2 = lambda1 *(10000/ Mw ) ^(1/3) ; K2 = (1 - lambda2 ) ^2; kc2 = kc1 *3^0.22; // [ cm/ s ] // L e t B2 = (1 R2 ) /R2 B2 = K2 /(1 - K2 ) * exp ( v / kc2 ) ; R2 = 1/(1+ B2 ) ; disp ( R2 , f r a c t i o n r e j e c t e d ( R2 ) = ) ; 174

47 48 49 50 51 52 53 54 55 56 57 58 59

// ( c ) Dpore = 10^ -7; // [ cm2/ s ] eps = 0.5; tou = 2; De = 2.5*10^ -8; // [ cm2/ s ] L = 2*10^ -5; // [ cm ] v = 5.56*10^ -4; // [ cm/ s ] vLbyDe = v * L / De ; // U s i n g Eq . ( 3 0 . 6 3 ) K = 0.101; c2bycs = K * exp ( vLbyDe ) /( K -1+ exp ( vLbyDe ) ) ; disp ( D i f f u s i o n i n t h e membrane makes t h e p r e m e a t e c o n c e n t r a t i o n s a b o u t t w i c e a s h i g h a s i t would be i f c 2=Kcs = 0 . 1 0 1 c s , i n d i c a t i n g t h a t t h e p a r t i t i o n c o e f f i c i e n t i s l o w e r than t h a t e s t i m a t e d i n p a r t ( a ) );

175

You might also like